X



トップページ数学
1002コメント476KB
面白い問題おしえて〜な 32問目
レス数が1000を超えています。これ以上書き込みはできません。
0390132人目の素数さん
垢版 |
2020/05/23(土) 20:57:15.99ID:KUlK5hoA
>>384
必ずしも成り立たない。

Gとして整数全体 Z を考える(加法群としてのZ)。
正規部分群Hとして、偶数全体の集合を考える。
もちろん、ここでの「偶数」は負の数も込めている。

Z = { g_i|i∈N } と表示できる任意の g:N → Z に対して

#(Z/H) = lim[n→∞] #{ g_i|i<n } / #(H∩{ g_i|i<n })

が成り立つかどうかを考える。以下では、g:N → Z が全単射のときを考える。
#{ g_i|i<n } = n であり、#(Z/H)= 2 であるから、

2 = lim[n→∞] n / #(H∩{ g_i|i<n })

が成り立つかどうかを考えればよい。そのためには

1/2 = lim[n→∞] #(H∩{ g_i|i<n }) / n

が成り立つかどうかを考えればよい。
0391132人目の素数さん
垢版 |
2020/05/23(土) 20:58:17.85ID:KUlK5hoA
全単射 g:N → Z を、以下の性質を満たすように作る。

・ g_n が偶数になるのは、3^{2s} ≦ n < 3^{2s+1} (s=0,1,2,…) のとき、かつそのときのみ

このような g が存在することは後で見ることにして、先にこのような g に対して

1/2 = lim[n→∞] #(H∩{ g_i|i<n }) / n

が成り立たないことを示す。というか、この g に対しては
そもそも lim[n→∞] #(H∩{ g_i|i<n }) / n が存在しない。
0392132人目の素数さん
垢版 |
2020/05/23(土) 20:59:40.49ID:KUlK5hoA
実際、k≧0 を任意に取る。1≦n<3^{2k+1}の範囲内で g_n が偶数になるのは、
3^{2s} ≦ n < 3^{2s+1} (0≦s≦k) のとき、かつそのときのみだから、

#(H∩{ g_i|i<3^{2k+1} }) = Σ[s=0〜k](3^{2s+1}−3^{2s}) = (9^{k+1}−1) / 4

であり、よって #(H∩{ g_i|i<3^{2k+1} }) / 3^{2k+1} = 3(1−9^{−(k+1)}) / 4 である。
特に、limsup[n → ∞] #(H∩{ g_i|i<n }) / n ≧ 3/4 である。

次に、1≦n<3^{2k+2}の範囲内で g_n が偶数になるのは、
やはり 3^{2s} ≦ n < 3^{2s+1} (0≦s≦k) のときのみだから、

#(H∩{ g_i|i<3^{2k+2} }) = Σ[s=0〜k](3^{2s+1}−3^{2s}) = (9^{k+1}−1) / 4

であり、よって #(H∩{ g_i|i<3^{2k+2} }) / 3^{2k+2} = (1−9^{−(k+1)}) / 4 である。
特に、liminf[n → ∞] #(H∩{ g_i|i<n }) / n ≦ 1/4 である。

以上より、そもそも lim[n→∞] #(H∩{ g_i|i<n }) / n が存在しない。
0393132人目の素数さん
垢版 |
2020/05/23(土) 21:00:12.23ID:KUlK5hoA
あとは、上記のような g が存在することを言えばよい。

偶数全体の集合を Z_0 とする(集合としては H = Z_0 である)。
奇数全体の集合を Z_1 とする。以下の4つの集合

Z_0, Z_1, ∪[s=0〜∞] { n∈N|3^{2s} ≦ n < 3^{2s+1} }, ∪[s=0〜∞] { n∈N|3^{2s+1} ≦ n < 3^{2s+2} }

はどれも可算無限集合であるから、どの2つの間にも全単射が取れる。特に、
全単射 f_0:∪[s=0〜∞] { n∈N|3^{2s} ≦ n < 3^{2s+1} } → Z_0 と
全単射 f_1:∪[s=0〜∞] { n∈N|3^{2s+1} ≦ n < 3^{2s+2} } → Z_1 を
何でもいいから取っておく。

g:N → Z を以下のように定義する。

g_n = f_0(n) (3^{2s} ≦ n < 3^{2s+1}, s=0,1,2…),
g_n = f_1(n) (3^{2s+1}≦ n < 3^{2s+2}, s=0,1,2…)

明らかに g:N → Z は全単射である。また、この g は明らかに

・ g_n が偶数になるのは、3^{2s} ≦ n < 3^{2s+1} (s=0,1,2,…) のとき、かつそのときのみ

という性質を満たす。
0394132人目の素数さん
垢版 |
2020/05/23(土) 21:27:42.93ID:9Nc0PEW8
>>384
右辺の極限は g_i の順番(添え字の取り方)に依存するので成立しない

【反例】
G を整数全体 Z がなす加法群 (Z, +) とし、 H = 2 * Z (偶数全体)とする。
このとき、 #(G/H) = 2 である。
n = 1, 2, … に対し、右辺の n のときの値を a_n とする:
a_n := #{g_i | i < n}/#(H∩{g_i | i<n}) = n / b_n
ここで、 g_i は i = 0, 1, 2, … で定められているものとし、
b_n := #(H∩{g_i | i<n}) ≠ 0 となる n について考えるものとする。
(1) g_i = 0, 1, -1, 2, -2, 3, -3, … (i = 0, 1, 2, … ) のとき

n = 1, 2, 3, … に対して、
b_n = 1, 1, 1, 2, 3, 3, 3, 4, 5, 5, 5, 6, 7, 7, 7, 8, 9, 9, 9, 10, …
より、 b_n = floor(n/4) + ceil(n/4) = floor(n/4) - floor(-n/4)
となる。ここで floor, ceil はそれぞれ床および天井関数である。このとき、
(n/2) - 1 < b_n < (n/2) + 1
であるので、 a_n → 2 (n → ∞)


(2) g_i = 0, 1, -1, 3, -3, 2, 5, -5, 7, -7, -2, 9, -9, 11, -11, 4, 13, -13, 15, -15, -4, …
(i = 0, 1, 2, … ) のとき

n = 1, 2, 3, … に対して、
b_n = 1, 1, 1, 1, 1, 2, 2, 2, 2, 2, 3, 3, 3, 3, 3, …
であるので、 n ≡ 0 (mod 5) のとき、常に a_n = 5 となる。
ゆえに a_n は 2 に収束しない。
0395132人目の素数さん
垢版 |
2020/05/23(土) 21:49:44.29ID:wOOSm8sv
>>375
ワイみたいなチンピラの書き込みが
建設的な話に発展して嬉しいわ

カスみたいな書き込みから
こういうまともな問題を思いつく人って
頭良さそう…とボクは思いました (^〜^)
0396132人目の素数さん
垢版 |
2020/05/23(土) 22:17:01.97ID:9Nc0PEW8
全単射の取り方によっては成立する可能性もあるので、>>384を次のように変更した問題を考えてみた

#A を 集合 A の濃度とする。

可算無限濃度の群 G とその正規部分群 H に対して、適切な全単射
N ∍ i |→ g_i ∊ G (ここで矢印 |→ は元の対応を表す)
を選べば、

#(G/H) = lim[n→∞] #{g_i | i < n} / #(H∩{g_i | i < n})

は常に成立するか?成立しないのならば反例を挙げよ。
ここで、右辺の極限は #(H∩{g_i | i<n}) ≠ 0 となるように十分大きな n の範囲で考えることとする。
0397132人目の素数さん
垢版 |
2020/05/24(日) 00:19:19.52ID:ZXQjARV3
どうせならこんな感じに、もうちょい自明でなさそうな問題を考えたいなあ
答えはわからないけど

任意の可算な群Gについて、次を満たす全単射f:N→Gは存在するか:
Gの任意の部分群Hについて、(#(G/H))^(-1) = lim_(n→∞) (1/n)*#{1≦k≦n | f(k)∈H})
0398132人目の素数さん
垢版 |
2020/05/24(日) 00:33:13.78ID:7dB8qUo6
>>397
#(G/H) = ∞ のときは、左辺は 0 と解釈するってこと?
あと、全単射 f を固定して部分群 H を動かすってこと?
全単射 f が部分群 H に依存せずに決まるとは考えにくいな
0399132人目の素数さん
垢版 |
2020/05/24(日) 00:47:27.03ID:ZXQjARV3
>>398
そうそう、fをGだけに依存してとれるかという問題
例えば加法群としてのZはこの条件を満たす。(fの値を0,1,-1,2,-2,…とすればOK)
反例が存在してもおかしくないくらいには強い命題だけど、
存在したとしてもそんなに自明なものにはならない気がする
0400132人目の素数さん
垢版 |
2020/05/24(日) 06:05:09.43ID:4W+6Vb4n
>>371 最後の933379288600368294785169967190258422519522243538669103040838466576871923901は合成数
10^100+3 = 7×157×769×2593×4888946572366141×220030935994058489226133×4242036622639156527888055237578804493024993216233097
0402132人目の素数さん
垢版 |
2020/05/24(日) 07:26:13.32ID:FbGoVtS3
>>390-394
素晴らしい
大正解です

こちらが用意していたものは、ほとんど>>394と同じですが、
G=Z,H=2Zに対して、

g_k=0,1,-1,2,3,-3,4,5,-5,...
= (2k)*(1+ω^k+ω^{2k})/9+(2k+1)*(1+ω^{k+2}+ω^{2k+1})/9-(2k-1)*(1+ω^{k+1}+ω^{2k+2})/9
(ω=(-1+√(-3))/2,k=0,1,2,...)

とすれば、lim(n→∞) #{g_k | k<n}/#(H∩{g_k | k<n})
=lim(n→∞)n/([n/3]+1)=3
([ ]はガウス記号)
より、#(G/H)=2とは異なる

というものでした
0403132人目の素数さん
垢版 |
2020/05/24(日) 07:27:41.71ID:FbGoVtS3
>>396-397
なるほどこれは無茶苦茶難しそうだな...

少なくともGが有限生成アーベル群なら正しいかな?
0404132人目の素数さん
垢版 |
2020/05/24(日) 07:41:49.28ID:FbGoVtS3
>>402
ということで、これで何が言いたかったというと
無限集合における「割合」を有限集合で区切って割合を求めて極限を出す、という定義にすると、

集合の番号の取り方によっては自然数における偶数の「割合」は1/3とも1/2とも解釈出来てしまって、定まらない
ということです

うまい具合に集合の割り算を定義してその濃度で定義してもいいのかもしれないけど群以外で集合の割り算を定義する方法を知りません(あるかもしれんけど)
0405イナ ◆/7jUdUKiSM
垢版 |
2020/05/24(日) 10:11:06.05ID:MJ8ChL8l
>>357
>>359
A.「整数空間Z の 約5割が 自然数空間N で占められている」

B.「実数空間R の 約0割が 有理数空間Q で占められている」

C.「素数の空間のうち、約10割が奇数で占められている」

D.「半素数の空間のうち、約d割が偶数で占められている」

E.「偶数空間のうち、約e割が 矩形数で占められている」

>>359半素数と矢巨形数がなにかによる。
0406132人目の素数さん
垢版 |
2020/05/24(日) 10:40:55.66ID:ZXQjARV3
>>403
多分正しい。以下の通り

補題
群Gの有限部分集合属 {S_i}_(i∈N) が以下を全て満たすとする。
・S_k⊂S_(k+1)
・∪_(k∈N) S_k = G
・lim_(k→∞) (#S_k)/(#S_(k+1)) = 1
・Gの任意の部分群Hについて #(G/H)^(-1) = lim_(k→∞) #(S_k∩H)/#S_k
この時、Gは>>397の性質を満たす。
(証明)
便宜的にS_0を空集合とする。
全単射f:N→Gを、全てのk∈Nについて
(#S_(k-1)<n≦#S_k ならば f(n)∈S_k\S_(k-1))
が成り立つように任意に定める。
変数n∈Nに対し、k を常に #S_(k-1)<n≦#S_k を満たす整数とすると、
S_kについての仮定から lim_(n→∞) (1/n)*#S_k = lim_(n→∞) (1/n)*#S_(k-1) = 1.
また、HをGの任意の部分集合とすると、
#(S_(k-1)∩H)/#S_k ≦ #{1≦i≦n|f(i)∈H}/#S_k ≦ #(S_k∩H)/#S_k
であり、n→∞の時の右左辺の極限が #(G/H)^(-1) であるから、中辺の極限も同じ値になる。
以上より lim_(n→∞) (1/n)*#{1≦i≦n|f(i)∈H} = #(G/H)^(-1).□

これより、Gが有限生成アーベル群なら
G = Z^m (+) T (ただし(+)は直和、Tはねじれ)
と分解できるので、
S_k = { g∈G | gのZ^m部分の各成分の絶対値はk以下 }
と定めればこれは補題の条件を満たすため、Gは397の性質を満たす。
0407132人目の素数さん
垢版 |
2020/05/24(日) 11:06:26.19ID:7yrUqYW3
>>406
自明じゃないし面白みあるから学会発表してみたら銅かな
0408132人目の素数さん
垢版 |
2020/05/24(日) 11:21:23.41ID:lJXSDwC8
それなりにむずかしい問題を投下します
チャレンジャー求む できたら天才BOY-KEN
「a,b,cの最小公倍数とa+1,b+1,c+1の最小公倍数が一致する」
そのような自然数a,b,c(a≦b≦c)の組をすべて求めよ

たとえば a=3,b=4,c=5 とすれば
3,4,5の最小公倍数は60, 4,5,6の最小公倍数も60 で条件を満たす
0409132人目の素数さん
垢版 |
2020/05/24(日) 11:33:36.46ID:AROWWaeg
>>359 の者ですが、質問です。

番号の取り方で前者を採用して、
偶数の割合を 1/3 と解釈した場合…、
この時、残りの 2/3 はいったい何なのですか?
残りの 2/3 はすべて奇数?
0413132人目の素数さん
垢版 |
2020/05/24(日) 18:50:21.11ID:FbGoVtS3
>>406
なるほど、これは素晴らしいですね
ということは同様に、非アーベル群でもZ^r × G(Gは有限群)みたいな形の群なら言えるということですか

2chでここまで有意義な議論が出来るとは...

あとはQ(√2)>Qみたいな有限生成じゃない群だとどうなるんだろ

>>409
この順番で「割合」を定義した場合、当然残りの2/3は奇数の集合です

>>411
あーすみません>>412の間違いですね
0414132人目の素数さん
垢版 |
2020/05/24(日) 19:28:23.94ID:7dB8qUo6
有限生成な群の部分群でも有限生成になるとは限らないらしいけど、そのあたりはどうなんだろう

問. 可算無限濃度の群 G に対し、 G が有限生成かつ有限生成でない G の部分群 H が存在するような例は存在するか?
0416132人目の素数さん
垢版 |
2020/05/24(日) 21:31:43.35ID:ra0ZpDC7
>>408
答だけ。
(a, b, c; L)=(3, 4, 5; 60)
      (3, 14, 20; 420)
      (5, 6, 14; 210)
      (9, 20, 35; 1260)
0418132人目の素数さん
垢版 |
2020/05/25(月) 00:03:14.06ID:Rb6oNfF2
一瞬、間違っているように見えるが正しい問題。解答はまあ普通。

問題:{a_n}_n, {b_n}_n, {c_n}_n は実数列で

・ a_n ≦ b_n ≦ c_n (n=1,2,3,…)

・ Σ[n=1〜∞] a_n と Σ[n=1〜∞] c_n が'存在する

を満たすとする。このとき、Σ[n=1〜∞] b_n も存在することを示せ。
0420132人目の素数さん
垢版 |
2020/05/25(月) 01:36:33.43ID:sJy5IDc3
>>415
自由群<a,b>の部分群でなんかできないかな
0422132人目の素数さん
垢版 |
2020/05/25(月) 06:05:48.16ID:tJYEuSNi
>>416
それだけしかないことの証明できそう?
技術的に かなり難しいとおもうんだよね
答えだけみると簡単にみえる分 たちがわるい
0424132人目の素数さん
垢版 |
2020/05/25(月) 08:36:57.80ID:sJy5IDc3
>>420
交換子群はどう?aba-b-, bab-a-ではa…aba-…a-b-作れないだろ
0425132人目の素数さん
垢版 |
2020/05/25(月) 08:53:40.08ID:sJy5IDc3
でも自由群の部分群は自由群だから交換子群も自由群だけど
ランクは∞か
0426132人目の素数さん
垢版 |
2020/05/25(月) 11:10:32.67ID:as7r/XH1
>>418
部分和を
 A_n = a_1 + a_2 + ・・・・ + a_n,
 B_n = b_1 + b_2 + ・・・・ + b_n,
 C_n = c_1 + c_2 + ・・・・ + c_n,
とおくと
 0 ≦ B_n - A_n ≦ C_n - A_n,

題意により C_n - A_n は収束するから有界である。
∴ B_n - A_n も有界である。
一方 a_n≦b_n から
 B_n - A_n は(広義の)単調増加列である。
よって
 B_n - A_n は有界単調列だから収束する。(定理6)

あるいは、題意により、
C_n - A_n は収束するのでコーシー列である。
∴ B_n - A_n もコーシー列だから収束する。(定理8)  >>421

高木:「解析概論」改訂第三版、岩波書店(1961)
  第1章 §4.定理6.有界なる単調数列は収束する。p.7-8
  第1章 §6.定理8.収束の条件、Cauchyの判定法 p.11-12
0427132人目の素数さん
垢版 |
2020/05/25(月) 12:32:19.35ID:mD76mqzp
>>413
1/3 が偶数で 2/3 が奇数 って
何か変な感じですね、

この論理で展開したら
集合のとり方によって結論が変わる。

A. 1/3 が「偶数」で 2/3 が奇数
B. 1/3 が奇数で 2/3 が「偶数」
C. 1/2 が「偶数」で 1/2 が奇数

この3通りのどれかに着地するんですね。
0428132人目の素数さん
垢版 |
2020/05/25(月) 15:59:43.02ID:6NWpRXTA
>>427
いやその3通りだけではもちろんない

>>394の(2)のように取れば偶数の割合は1/5になってしまうし

もちろん1/5以外もいくらでもなりうる

>>391-393
のように取ればそもそも振動して極限が存在しない場合もある

なので、有限で区切って割合をとっても意味がないということが言いたいことです。
(順序が最初から決められているのならばいいかもしれない)
0429132人目の素数さん
垢版 |
2020/05/25(月) 16:22:40.06ID:6NWpRXTA
g_i=0,-1,1,2,3,-3,5,-5,-2,7,-7,9,-9,11,-11,4,13,-13,15,-15,17,-17,19,-19,-4,...

みたいに全単射を取れば
偶数の割合は0%とも解釈できてしまう
0431132人目の素数さん
垢版 |
2020/05/25(月) 17:39:28.74ID:as7r/XH1
>>364
・N(自然数), Z(整数), 2Z(偶数), PP'(半素数), 矩形数, それらの共通要素の集合
の要素には1,2,・・・・の番号が付いているとする。

(A)Nのa(n-1)+1番〜an番と、Z-Nの(10-a)(n-1)+1番〜(10-a)n番
 を合わせて第n組とする。

(D)偶数の半素数のd(n-1)+1番〜dn番と、奇数の半素数の(10-d)(n-1)+1番〜(10-d)n番
 を合わせて第n組とする。

(E)矩形数の e(n-1)+1番〜en番と、矩形でない偶数の(10-e)(n-1)+1番〜(10-e)n番
 を合わせて第n組とする。

どれも余らずに対応する。
0433132人目の素数さん
垢版 |
2020/05/25(月) 20:13:18.46ID:as7r/XH1
コーシーの条件を使えば A_n ≦ B_n ≦ C_n だけでも解ける。
コーシーなしの場合は無理か?(a_n≦b_n≦c_nを使わざるを得ない?)
コーシーの条件は実数の連続性に基づくから、その影響かな?
0434132人目の素数さん
垢版 |
2020/05/25(月) 20:27:07.78ID:Rb6oNfF2
>>433
>コーシーの条件を使えば A_n ≦ B_n ≦ C_n だけでも解ける。

ん?それはどういう意味?

問題:{a_n}_n, {b_n}_n, {c_n}_n は実数列で

・ Σ[i=1〜n] a_i ≦ Σ[i=1〜n] b_i ≦ Σ[i=1〜n] c_i (n=1,2,3,…)

・ Σ[n=1〜∞] a_n と Σ[n=1〜∞] c_n が存在する

を満たすとする。このとき、Σ[n=1〜∞] b_n も存在することを示せ。

↑この問題に差し替えても解ける、という意味かな?
こちらの問題は普通に反例があって、

Σ[i=1〜n] a_i = 0 (n=1,2,3,…)
Σ[i=1〜n] c_i = 1 (n=1,2,3,…)
Σ[i=1〜n] b_i = 0 (nが偶数), 1 (nが奇数)

となるように適当に a_i, b_i, c_i を決めれば反例になる。
もちろん、この反例では a_n ≦ b_n ≦ c_n (n=1,2,3,…) が
成り立つようにできないので、もともとの問題の反例ではない。
0435132人目の素数さん
垢版 |
2020/05/25(月) 20:34:30.87ID:Rb6oNfF2
ああ、だから、>>426の解答のうち、前半は正しい解答だけど、
後半はこのままだと意味が確定しないね。前半だけ読んで満足してたわ。

> あるいは、題意により、
> C_n - A_n は収束するのでコーシー列である。
> ∴ B_n - A_n もコーシー列だから収束する。(定理8)  >>421

C_n−A_n がコーシー列で 0 ≦ B_n - A_n ≦ C_n - A_n であっても、
それだけでは B_n−A_n がコーシー列だとは言えない。

A_n,B_n,C_n は

A_n = a_1 + a_2 + ・・・・ + a_n,
B_n = b_1 + b_2 + ・・・・ + b_n,
C_n = c_1 + c_2 + ・・・・ + c_n,

という形をしていて、しかも a_n≦b_n≦c_n なので、これらを全て組み合わせれば、
B_n−A_n もコーシー列だと言えるだろうけど、いずれにしても a_n≦b_n≦c_n は必須なので、

>コーシーの条件を使えば A_n ≦ B_n ≦ C_n だけでも解ける。

がどういう意味なのか判断しかねる。
0437132人目の素数さん
垢版 |
2020/05/25(月) 20:58:47.85ID:Rb6oNfF2
>>436
何が言いたいのか分からない。

「 Σan、Σcnが収束するならlim an = lim cn = 0 である 」

これは正しい。それで?
0438132人目の素数さん
垢版 |
2020/05/25(月) 20:59:15.95ID:dE6ck3kC
>>436
それで lim[n→∞] b_n = 0 だから、 b_n の和も収束するってこと?
そうとは限らないぞ
有名な反例として、調和級数
1 + 1/2 + 1/3 + … = ∞
がある
0440132人目の素数さん
垢版 |
2020/05/26(火) 00:17:50.15ID:moXRUVS0
一瞬、自明でナンセンスな問題に見えるが、実はそうではない問題。

問題:写像 f:N → {0,1} が極限値 lim[n→∞] (f(1)+f(2)+…+f(n)) / n を持つとき、
その値を L(f) と書くことにする。次に、任意の写像 f,g:N → {0,1} に対して、
新しい写像 f*g:N → {0,1} を以下のように定義する。

(f*g)(n) = 0 ( (f(n),g(n))=(0,0),(1,1)のとき )
(f*g)(n) = 1 ( (f(n),g(n))=(0,1),(1,0)のとき )

さて、実数αは次を満たすとする。

∀f,g:N → {0,1}, ∃h:N → {0,1} s.t. L(f*h) と L(g*h) が存在して L(f*h)=α, L(g*h)=α.

このような性質を満たすαを全て求め、そのことも証明せよ。
0441132人目の素数さん
垢版 |
2020/05/26(火) 01:16:55.63ID:DAH9orLP
α=1/2.
f≡0, g≡1 とすればL(f*h)=L(g*h)の時の可能な値は1/2のみ。
以下で逆を証明する。

fとgの両方にfを*することにより、f≡0 として良い。この時、関数hを
h(n)=0 (nがg(n)=0を満たす最小の整数であるか、g(n)=1を満たす最小の整数である時)
h(n)=1-h(m) (g(n)=g(m)を満たすn未満の整数のうち最大の整数mが存在する時)
と定めればL(f*h)=L(g*h)=1/2となる。
0442132人目の素数さん
垢版 |
2020/05/26(火) 01:25:01.49ID:tZAgiR8E
>>440
α = 1/2 のみ

なぜなら、 f, g が任意であることから、 Im(f) = {0}, Im(g) = {1} となる f, g をとることができる。
この f, g に対して定まる h について、
H(n) := #{k | h(k) = 1 (1 ≦ k ≦ n)}
とすると、仮定から
α = L(f*h) = lim[n→∞] H(n) / n が存在し、また、明らかに H(n) ≦ n であることから、
α = L(g*h) = lim[n→∞] |n - H(n)| / n = lim[n→∞] (1 - H(n)/n) = 1 - lim[n→∞] H(n) / n = 1 - α
すなわち、
α = 1 - α
となるので、 α = 1/2 である。
0443132人目の素数さん
垢版 |
2020/05/26(火) 01:35:01.22ID:wFm4aDoj
>>440
f=0g=1の時も成立せねばならないが、このとき
L(f*h)=L(h), L(g*h)=1-L(h)
となるから、コレが等しくなるにはL(h)=1/2である事が必要。
α=1/2が条件を満たすことを示す。
まずg=0の場合を考える。
この時は任意のNについて
| #{x≦N | h(x) = 1 & f(x)=1} - #{x≦N | h(x) = 0& f(x)=1}|,
| #{x≦N | h(x) = 1 & f(x)=0} - #{x≦N | h(x) = 0& f(x)=0}|,
のいずれも高々1しか違わないようにとる。
(すなわちf(x)=1,f(x)=0となるxを半分±1/2になるものを選びそこでh(x)=1と割り当てる)
この時L(f*h)=L(h)=L(g*h)=1/2となる。
一般の場合には
L(f*g*k)=L(k)=1/2
となるkをとってh=g*kとすればよい。
0444132人目の素数さん
垢版 |
2020/05/26(火) 01:49:36.27ID:tZAgiR8E
あ、そうか
求めた α が条件を満たすことを確認しないといけないのか
しまった
0445132人目の素数さん
垢版 |
2020/05/26(火) 04:17:50.92ID:moXRUVS0
>>441-444
こちらが想定していた解法と違ってるゥ!(α=1/2を導くところは想定解法と同じだが)
確かに、fとgのうち片方は0としても一般性を失わず、そうすると割と簡単にできるっぽい。

今回はfとgの2つだったが、実は可算無限個のf_1,f_2,f_3,…にした問題でも、
こちらの想定していた解法だと通用する。ちょっとオーバーかなと思って
書かなかった問題なのだが、まあ興味があったらどうぞ↓

問題:写像 f:N → {0,1} が極限値 lim[n→∞] (f(1)+f(2)+…+f(n)) / n を持つとき、
その値を L(f) と書くことにする。次に、任意の写像 f,g:N → {0,1} に対して、
新しい写像 f*g:N → {0,1} を以下のように定義する。

(f*g)(n) = 0 ( (f(n),g(n))=(0,0),(1,1)のとき )
(f*g)(n) = 1 ( (f(n),g(n))=(0,1),(1,0)のとき )

さて、実数αは次を満たすとする。

∀f_n:N → {0,1} (n=1,2,3,…), ∃h:N → {0,1} s.t. L(f_n*h) (n≧1) が全て存在して L(f_n*h)=α (n≧1).

このような性質を満たすαを全て求め、そのことも証明せよ。
0447132人目の素数さん
垢版 |
2020/05/26(火) 10:12:41.24ID:DAH9orLP
>>445
【α=1/2 が条件を満たすこと】
S={f:N→{0,1}} とおき、関数 F:[0,1)→S を
(F(x))(n) = ([x・2^n]を2で割った余り)
と定める。
(つまり 0.(F(x))(1) (F(x))(2) (F(x))(3)… と並べてできる二進小数はxと一致する。)
区間[0,1)を連続一様分布でとる確率変数xについて、
各n∈Nについて『L(F(x)*f_n)=1/2 である確率は1』が成り立つため、
『全てのn∈Nについて L(F(x)*f_n)=1/2 が成り立つ』確率も1.
ゆえにそのようなxが少なくとも一つ存在するので、h=F(x) とすれば良い。

【条件を満たすαが1/2のみであること】
f_1≡1, f_n≡0 (n≧2) の場合を考えれば良い。
0448132人目の素数さん
垢版 |
2020/05/26(火) 11:13:54.65ID:4wdG72bX
>>446
いくら近づけようがさらにそれを凌駕する値をいくらでももってくることができる。
という意味合いを量化子と二階論理で記述しただけの話だ。

詭弁の方の定義すら意を解してなさそうなのはおまえ。
0449132人目の素数さん
垢版 |
2020/05/26(火) 14:10:38.88ID:aYF++qy3
>>447
> 区間[0,1)を連続一様分布でとる確率変数xについて、
> 各n∈Nについて『L(F(x)*f_n)=1/2 である確率は1』が成り立つため、

コレはなぜ?
0451132人目の素数さん
垢版 |
2020/05/26(火) 14:21:01.82ID:tZAgiR8E
確率論の話は全然分からないんだが、 α = 1/2 であることの有効性はどの辺に現れるんだろう
条件を満たす α が 1/2 に限る以上、どこかに 1/2 特有の議論が含まれるはずだが

>(つまり 0.(F(x))(1) (F(x))(2) (F(x))(3)… と並べてできる二進小数はxと一致する。)

ここが重要なの?
0452132人目の素数さん
垢版 |
2020/05/26(火) 14:45:14.94ID:/orrRVlJ
>>451
> 各n∈Nについて『L(F(x)*f_n)=1/2 である確率は1』が成り立つため、

多分この辺なのかな
F(x)のk番目を確率1/2ずつでランダムに決めれば
F(x)*f_n のk番目も同じく確率1/2ずつランダムに決まることになり、
大数の法則みたいな議論がうまいこと機能する。

F(x)の各項の値が1である確率を1/3、0である確率を2/3等としても、
F(x)*f(n) の各項が1である確率が1/3と2/3でばらばらになって、同じ議論はできない。
0453132人目の素数さん
垢版 |
2020/05/26(火) 16:02:29.37ID:aYF++qy3
あ、いや、やっぱり当たり前じゃないな。

実数xの2進展開のn桁目をxnとするときに
L(xn)=1/2
となるxは一様分布で確率1で成立。

これなぜ?
一様分布定理だけじゃ無理だよね?
0454132人目の素数さん
垢版 |
2020/05/26(火) 16:53:01.44ID:/orrRVlJ
>>453
確率論の細かい定理は自分もうろ覚えな所あるからちゃんと示してみようかな

整数mに対して定まるxについての事象
| #{k∈{1,2,…,m} : xk=1} - m/2 | ≦ m^(3/4)
が起きる確率P(m)は 1-O(m^(-1/4)). (as m→∞)

ここで、Mを正の数とすると
『全ての8乗数 m≧M について上の事象が起こる』…(A)
ことは L(xn)=1/2 が成り立つための十分条件であるから、後者が成り立つ確率は
≧ Π_(m≧M, mは8乗数) P(m)
= 1-O(M^(-1/8)) (as M→∞)
であり、つまり1と等しい。
0455132人目の素数さん
垢版 |
2020/05/26(火) 17:08:47.25ID:moXRUVS0
>>447
正解!こちらが想定していた解法と本質的に同じです。
こちらが想定していたのは、次のようなやり方。

・α=1/2 に絞られるのはいつも通り。
 あとは、α=1/2のときに問題の性質が実際に成り立つことを示せばよい。

・{0,1}上の離散位相をθとして、{0,1}^N 上の積位相をΠθと書くことにする。
 Nから{0,1}への写像は{0,1}^Nの元と同一視できるので、

 ∀y_n∈{0,1}^N (n=1,2,3,…), ∃x∈{0,1}^N s.t. L(y_n*x) (n≧1) が全て存在して L(y_n*x)=1/2 (n≧1)

 を示せばよい。

・Πθから生成されるボレル集合体を B と書くことにする。可測空間 ({0,1}^N, B) の上には、

 ∀n≧1, ∀v_1,v_2,…,v_n∈{0,1} s.t. P( { x∈{0,1}^N|x_1=v_1, …, x_n=v_n } ) = 1/2^n

 を満たすただ1つの確率測度 P が定義できる。
 確率空間({0,1}^N, B, P)の完備化を ({0,1}^N, F, Q) と書くことにする。
 直観的に言えば、コイントスを可算無限回繰り返したときの各種の確率を
 ({0,1}^N, F, Q) によって論じることが可能になる。
0456132人目の素数さん
垢版 |
2020/05/26(火) 17:10:10.96ID:moXRUVS0
・L(x)=1/2 Q.a.e. x∈{0,1}^N が成り立つことが知られている(大数の強法則)。
 特に、M = { x∈{0,1}^N|L(x)が存在しない、または存在するが L(x)≠1/2 } と置けば、
 M はゼロ集合である。すなわち、M∈F かつ Q(M)=0 である。

・A⊂{0,1}^N と x∈{0,1}^N に対して x*A={ x*a|a∈A } と定義すると、
 A∈F に対しては x*A∈F が成り立ち、しかも Q(x*A)=Q(A) が成り立つことが証明できる。
 特に、A∈F がゼロ集合なら x*A もまたゼロ集合である。

・y∈{0,1}^N ごとに M_y={ x∈{0,1}^N|L(y*x)が存在しない、または存在するが L(y*x)≠1/2 }
 と置けば、M_y=y*M となることが示せるので、M_y はゼロ集合である。

・特に、可算無限個の y_n∈{0,1}^N (n=1,2,3,…) に対しても M_{y_n} は全てゼロ集合なので、
 ∪[n=1〜∞]M_{y_n} もゼロ集合である。よって、{0,1}^N−∪[n=1〜∞]M_{y_n} は空でない。
 そこで、x∈{0,1}^N−∪[n=1〜∞]M_{y_n}を1つ取れば、
 L(y_n*x) (n≧1) が全て存在して L(y_n*x)=1/2 (n≧1)となり、目標に達する。
0457132人目の素数さん
垢版 |
2020/05/26(火) 17:14:29.93ID:moXRUVS0
要するに、

・ 任意の y_n∈{0,1}^N (n=1,2,3,…) に対して、
  コイントスを無限回繰り返したときの出目を順番に並べて
  x=(x_1,x_2,x_3,…)∈{0,1}^N とすれば、この x が確率1で

 「 L(y_n*x) (n≧1) が全て存在して L(y_n*x)=1/2 (n≧1) 」

  を満たす

と言っていることになる。
具体的に x を構成しようとすると闇が深すぎて無理ゲーなのに、
確率論の観点からは「確率1で成り立つので、x は大体何を取ってきても通用する」
という状況になっている。
0458132人目の素数さん
垢版 |
2020/05/26(火) 17:18:40.23ID:moXRUVS0
ちなみに、この問題に関してよくある誤答は、

「 任意のα∈[0,1] がその性質を満たす。証明は自明。問題としてナンセンス!」

というもの。むかし、どこかのスレにこの問題を書き込んだときに、こういう反応があった。
知人にリアルの場面で出題したときも同じ反応だった。

「この問題、どうやら初見では勘違いしやすいらしい」と悟ったw
0459132人目の素数さん
垢版 |
2020/05/26(火) 18:37:25.04ID:aYF++qy3
おお、なるほど。
大数の法則使うのか。
弱法則だけでもいけるな。
0460132人目の素数さん
垢版 |
2020/05/26(火) 18:47:41.06ID:aYF++qy3
あ、いや強法則までいるのか。Lが確率1で収束する事は強法則までいるな。
0461132人目の素数さん
垢版 |
2020/05/26(火) 19:30:30.89ID:/orrRVlJ
リーマン予想は、メビウス関数μが任意のε>0に対して
Σ_(n=1,N) μ(n) = O(N^(1/2+ε))
を満たすことと同値であることが知られているけど、
例えば関数 f:N→{-1,1} の各項をコイントスのようにランダムに決めれば
Σ_(n=1,N) f(n) = O(N^(1/2)+ε)
は確率1で成り立ってしまうんだよな

リーマン予想は、メビウス関数がこのような『ありふれた』関数であるかどうかを問う問題とも言える
しかし >>457 の通り、確率論的にありふれているという性質について具体的に扱おうとすると、
想像以上の闇の深さに直面することになる…
0462132人目の素数さん
垢版 |
2020/05/27(水) 03:03:30.03ID:sG8aLkL9
>>461
> 例えば関数 f:N→{-1,1} の各項をコイントスのようにランダムに決めれば
> Σ_(n=1,N) f(n) = O(N^(1/2)+ε)
> は確率1で成り立ってしまうんだよな

これは何故?
0464132人目の素数さん
垢版 |
2020/05/27(水) 07:40:05.46ID:ubqnv61v
>>463

x=2cosθ
y=2sinθ

xy=1より4cosθsinθ=1

2倍角の公式よりsin2θ=1/2

よって2θ=30度,150度つまりθ=15度,75度

2つの交点と原点のなす角度が75-15=60度
0465132人目の素数さん
垢版 |
2020/05/27(水) 09:07:48.25ID:6/JW3mMG
>>462
α>1/2 として、((x^(-1)+x)/2)^n の x^O(n^α) の項の係数の和が
1-O(n^(-α+1/2)) であることを使って >>454 と同じように示せるんじゃないかな
0466132人目の素数さん
垢版 |
2020/05/28(木) 06:24:30.20ID:BNrN/28c
inverf(x)を関数
erf(x)=2/√π∫[0,x]exp(-t^2)dt
の逆関数とし、inverf(x)=Σ[t=0,∞]a(n)x^(2n+1)とおく。
a(n)は狭義単調減少である事を示せ。
0467132人目の素数さん
垢版 |
2020/05/28(木) 09:57:54.09ID:BJbMJzAu
>>463
 円の中心をO、円と双曲線の交点を P,Q とおく。
 対称性により P(a,b) Q(b,a)
 題意により aa+bb = 4 = 4ab,
 PQ^2 = 2(b-a)^2 = 2(aa+bb)- 4ab = aa+bb
  = OP^2 = OQ^2,
∴ △OPQ は3辺が等しいから正三角形。
0468132人目の素数さん
垢版 |
2020/05/28(木) 22:54:31.69ID:UME+jaT/
>>429
偶数が0%ってどう考えてもおかしくないですか?
だって偶数自体は必ず存在してるじゃないですか
ありえないですよ

ここにいる人たちはただの詭弁しか言ってないな
0473132人目の素数さん
垢版 |
2020/05/29(金) 04:53:12.66ID:cO4rYgZj
>>466

y = ∫[0,x] e^(-tt) dt
 = Σ[k=0,∞] ∫[0,x] (1/k!)(-tt)^k
 = Σ[k=0,∞] (-1)^k /((2k+1)・k!) x^(2k+1)
 = x - (x^3)/3 + (x^5)/10 - (x^7)/42 + (x^9)/216 - ・・・・

x = y + (1/3)y^3 + (7/30)y^5 + (127/630)y^7 + (4369/22680)y^9 + ・・・
0475132人目の素数さん
垢版 |
2020/05/29(金) 13:14:15.04ID:782fR9LW
>>466
wolframのseriesって数値表示はどうやるんだろ?

series [inverf(x),{x,0,30}]

https://www.wolframalpha.com/input/?i=series+%5B%2Binverf%28x%29%2C%7Bx%2C0%2C30%7D%5D&;lang=ja

無理クリコレ↓で確かめはできるんだけど

series [1.00000x/(1-x^2)+inverf(x),{x,0,30}]

https://www.wolframalpha.com/input/?i=series+%5B1.00000x%2F%281-x%5E2+%29+%2Binverf%28x%29%2C%7Bx%2C0%2C30%7D%5D&;lang=ja
0476132人目の素数さん
垢版 |
2020/05/29(金) 13:30:35.02ID:cO4rYgZj
>>473
 e^tt ≧ 1 + tt,
y < ∫[0,x] 1/(1+tt) dt
 = arctan(x)
 = x - (x^3)/3 + (x^5)/5 - (x^7)/7 + (x^9)/9 - ・・・・

x > tan(y)
 = y + (1/3)y^3 + (2/15)x^5 + (17/315)x^7 + (62/2835)x^9 + ・・・・
 > y/(1 - yy/3)
 = y + (1/3)y^3 + (1/9)y^5 + (1/27)y^7 + (1/81)y^9 + ・・・・
からは出ないだろうな・・・・

(*)
(1-yy/3)sin(y) - y・cos(y) = (1/3) ∫[0,y] t{sin(t) - t・cos(t)} dt
 = (1/3)∫[0,y] t・cos(t){tan(t) - t} dt > 0,
0477132人目の素数さん
垢版 |
2020/05/29(金) 14:19:52.69ID:SwC2PS5p
> y = Σ[k=0,∞] (-1)^k /((2k+1)・k!) x^(2k+1)

ここまでわかったなら、Lagrange inversion theoremを使えばいいんじゃないかな
0478132人目の素数さん
垢版 |
2020/05/29(金) 15:53:20.35ID:iHrnwgOZ
[問題]
lcm(a,b,c) = lcm(a+1,b+1,c+1)
を満たす正の整数a,b,c(a<b<c)の組は以下の4つに限ることを証明せよ:
(a,b,c)=(3,4,5),(3,14,20),(5,6,14),(9,20,35)
0479132人目の素数さん
垢版 |
2020/05/30(土) 21:52:54.01ID:YBzBFQMY
>>466
問題作り損なってた。
こうです。

inverf(x)を関数
erf(x)=2/√π∫[0,x]exp(-t^2)dt
の逆関数とし、inverf(x)=Σ[t=0,∞]a(n)x^(2n+1)とおく。
a(n+2)<4/π a(n)
を示せ。

でした。
ヒントはy=(x^2-π/4)inverf(x)とおくと
y''=(xとyとinverf(x)の式)
が作れます。
それをじっとよく見る。
0481132人目の素数さん
垢版 |
2020/05/30(土) 23:03:44.00ID:YBzBFQMY
>>480
再訂正
やはり>>466そのままで行けます。
ヒントはやはりy=(x^2-1)inverf(x)を考えますが、これのn階微分を一気に考えます。
それの定数項がどうなるかを考えるといけるようです。
0482132人目の素数さん
垢版 |
2020/05/31(日) 15:57:16.86ID:2uiZOpZM
>>478
2桁の数字で総当たりしてみた。

library(numbers)
N=100
abc=NULL
for(a in 1:(N-2)){
for(b in (a+1):(N-1)){
for(c in (b+1):N){
abc<-rbind(abc,c(a,b,c))
}
}
}
saveRDS(abc,('abd.rds'))
nrow(abc)
f <- function(x){
mLCM(x)==mLCM(x+1)
}
abc[apply(abc,1,f),]

> abc[apply(abc,1,f),]
[,1] [,2] [,3]
[1,] 3 4 5
[2,] 3 14 20
[3,] 5 6 14
[4,] 9 20 35

解析解は賢者にお任せ。
0483132人目の素数さん
垢版 |
2020/05/31(日) 16:15:16.37ID:oVdBH7pL
2桁どころか4桁の範囲でもそれしか見つからないが
3つの最小公倍数の意味的には無限個あってもおかしくない気はする...
0484132人目の素数さん
垢版 |
2020/06/01(月) 23:03:29.76ID:COtsZQEu
整数二つだとどうなんだろう
lcm(a,b)=lcm(a+1,b+1) が成り立つ(a,b)の組はどのくらい?
0485132人目の素数さん
垢版 |
2020/06/02(火) 02:09:03.15ID:TPydHgX/
a,a+1 は互いに素、 b,b+1 は互いに素。
lcm(a,b) = lcm(a+1,b+1) から
a|(b+1)|a ⇒ a=b+1
b|(a+1)|b ⇒ b=a+1
∴ (a,b) はない。
0487132人目の素数さん
垢版 |
2020/06/02(火) 09:21:09.76ID:ZPOwdRdb
条件を満たすa,b,cの上限が突き詰められば、あとは計算機任せにできるんだがなぁ。
0488132人目の素数さん
垢版 |
2020/06/02(火) 10:07:25.57ID:WKXJXMmA
自分は無限個あるとおもってるんだけど?
lcmを1つ処理するときに3つのgcd=1であっても6個も文字がでてくるから
つまり a=stp, b=tuq, c=usr とかくことができて lcm(a,b,c)=stupqr
ただし gcd(s,tuq)=gcd(t,usr)=gcd(u,stp)=gcd(p,qr)=gcd(q,rp)=1
ということで自由度が高いとおもう ただ問題はlcmが2つもあるのでそこの処理が困難
0489132人目の素数さん
垢版 |
2020/06/03(水) 00:49:59.38ID:Vj2o+qIA
>>486
すみません。

整数A,B (A≠0) について
A | B
B/A ∈ Z
BはAで割り切れる。余りが0
B ≡ 0 (mod A)
は同じ意味です。
0490132人目の素数さん
垢版 |
2020/06/03(水) 01:15:34.97ID:TvwTZ/i5
>>466
あんまり盛り上がらないので背景と方針を。
コレいわゆる“z値”、すなわち与えられたαに対して

 1/√(2π)∫[-z,z] exp(-t^2/2) dt = α

を満たすzを求める問題です。コレの答えが

 z = √2 inverf(α)

になります。
で話は、コレをマクローリン展開で気分良く一発で計算したいなと思って思いつきました。
inverf(x) = Σ[n=0,∞]a[n]x^(2n+1)
と展開するときのanの漸化式自体は探せばいくらでも見つかるのですが(例えば[1])このa[n]は全部正なので打ち切りの近似値は必ず“下から”になってしまいます。
そこでテーマはf(x)=1/(1+x^2)inverfx)とおいたとき、コレのマクローリン展開は交代級数になるのか?すなわちa[n]は単調減少になるのか?です。
ソレを[1]の漸化式で示せないかなというお話でした。
漸化式を認めると割とスッと示せると思います。

[1] https://mathworld.wolfram.com/InverseErf.html
0491イナ ◆/7jUdUKiSM
垢版 |
2020/06/03(水) 14:15:31.62ID:UPuHTaSO
>>405
>>408
3 4 5 [3 2^2 5☆2^2 5 2×3]
3 14 20[3 2×7 2^2×5☆2^2 3×5 3×7]
5 6 14[5 2×3 3×7☆2×3 7 3×5]
9 20 35[3^2 2^2×5 5×7☆2×5 3×7 2^2×3^3]
たまたまだよ。この書きこみでいいです。
lucky limit
chance cover
magic major
0492132人目の素数さん
垢版 |
2020/06/06(土) 04:31:13.88ID:5FNAzWdc
2人でじゃんけん勝負をし、n回勝ち越した方を勝者とします。勝敗が決するまでに何回勝負が行われるか、その期待値を求めてください。ただしあいこは回数に含みません。
0493 【菊】
垢版 |
2020/06/06(土) 13:40:35.61ID:l5CltTkc
>>491
>>492
n=1のとき、1/2+3/8+5/32+7/128+9/512+11/2048+…
n=2のとき、
0495 【菖蒲】
垢版 |
2020/06/06(土) 14:12:45.67ID:l5CltTkc
>>493
>>492
n=2のとき、(1/2)^2+2(1/2)^4+4(1/2)^6+8(1/2)^8+16(1/2)^10+32(1/2)^12+…
n=3のとき、
まだ予想がつかない。
0496 【凶】
垢版 |
2020/06/06(土) 14:27:14.03ID:l5CltTkc
>>495
>>493訂正。
n=1のとき、1/2の確率で1回勝ち越せるから、
1/2
残りの1/2は相手が一回勝ち越しでこっちは再起不能じゃねえか。
1/2+0=1/2
0497 【だん吉】
垢版 |
2020/06/06(土) 16:17:06.95ID:l5CltTkc
>>496
>>492
n=3のとき、3/2^3+5(3C1)/2^5+7(5C2)/2^7+9(7C3)/2^9+11(9C4)/2^11+13(11C5)/2^13+…
0498132人目の素数さん
垢版 |
2020/06/06(土) 17:19:39.76ID:RyPojoqR
k回目で決着する確率を P_k とおく。(n≦k≦2n-1)
・n≦k≦2n-2
 k-1回まで (n-1) > (k-n) でk回目に(n-1)が勝てば決着。(逃げ切り)
・k=2n-1
 k-1回まで (n-1):(n-1) のデュースですがあと1回勝てばよく、どちらが勝っても決着。
いずれの場合も
 P_k = C[k-1,n-1] / 2^(k-1),
よって
E(k) = Σ[k=n,2n-1] k・P_k = 2n{1 - C[2n,n] /(4^n)},
0499132人目の素数さん
垢版 |
2020/06/06(土) 17:44:33.83ID:YE2mxaSc
日本語の問題ですかね
「n回勝ち越す」は「白星が黒星よりn個多い」を意味します
n=1で早速引っかかってる方がいますが、これはどちらかが1回勝てばその瞬間に決着がつくので期待値は1ですよ
0500132人目の素数さん
垢版 |
2020/06/06(土) 17:46:18.51ID:YE2mxaSc
因みに答えは無限級数が残ったり偶奇で場合分けされたりしません
とても綺麗な答えになりますよ
0501132人目の素数さん
垢版 |
2020/06/06(土) 18:39:38.91ID:dnuHAH8y
シミュレーションしてみた。

sim <- function(n){
flg=FALSE # n回勝ち越しflag
i=0 # カウンタ初期値
while(flg==FALSE){ # どちらかがn回勝ち越しでなければ
i=i+1 # カウンタを増やす
y=rbinom(i,1,0.5) # i回ジャンケンして
flg <- abs(sum(y==1)-sum(y==0)) == n # n回勝ち越しか判定
}
return(i) # 勝負がついたジャンケン回数を返す
}
fn <- function(n,k=1e4) mean(replicate(k,sim(n)))
n=1:20
y=sapply(n,fn)
plot(n,y,bty='l',ylab='Expected Value',pch=19)
data.frame(Ex=y)

https://i.imgur.com/JK8s2F8.png

> data.frame(Ex=y)
Ex
1 1.0000
2 4.1632
3 7.6500
4 11.9086
5 16.7554
6 21.4310
7 26.7498
8 33.1558
9 39.0942
10 45.8874
11 53.2618
12 61.0158
13 68.9670
14 78.1836
15 85.9006
16 95.8346
17 106.2452
18 115.5166
19 125.7312
20 136.7028

解析解は賢者にお任せ。
0502132人目の素数さん
垢版 |
2020/06/06(土) 19:09:08.77ID:YE2mxaSc
困りました、シミュレーション結果と想定解がズレてますね
一応手元のpythonでシミュレーションした結果想定解とほぼ一致しました、何かミスってませんか?
0504132人目の素数さん
垢版 |
2020/06/06(土) 19:54:01.46ID:dnuHAH8y
>>501
i 回のジャンケンでn回勝ち越しでない場合にそのままジャンケンを追加するのでなくて、新たに i+1回 やり直すアルゴリズムなのが間違っているのだと思います。

>501は撤回します。
0505132人目の素数さん
垢版 |
2020/06/06(土) 20:14:04.31ID:dnuHAH8y
デバックしたつもり。

sim <- function(n){
flg=FALSE # n回勝ち越しflag
i=0 # カウンタ初期値
y=NULL # 勝敗配列
while(flg==FALSE){ # どちらかがn回勝ち越しでなければ
i=i+1 # カウンタを増やす
y=c(y,rbinom(1,1,0.5)) # 1回ジャンケンして結果をyに追加
flg <- abs(sum(y==1)-sum(y==0)) == n # n回勝ち越しか判定
}
return(i) # 勝負がついたジャンケン回数を返す
}

fn <- function(n,k=1e4) mean(replicate(k,sim(n)))
n=1:20
y=sapply(n,fn)
data.frame(Ex=y)

> data.frame(Ex=y)
Ex
1 1.0000000000000000
2 3.9952000000000001
3 9.0063999999999993
4 16.0839999999999996
5 24.7911999999999999
6 36.7032000000000025
7 49.3744000000000014
8 64.5473999999999961
9 81.6077999999999975
10 100.3460000000000036
11 120.0926000000000045
12 142.3019999999999925
13 169.5545999999999935
14 197.2299999999999898
15 225.0375999999999976
16 254.5725999999999942
17 289.1537999999999897
18 324.0305999999999926
19 356.2748000000000275
20 404.5817999999999870
0507 【菊】
垢版 |
2020/06/06(土) 20:56:16.73ID:l5CltTkc
>>497たったの20勝やそこら勝ち越すために400試合もすんのかよ?
スパーでもそうとう消耗してるな。松葉相撲ならサドンデスだけどね。
0508132人目の素数さん
垢版 |
2020/06/06(土) 21:21:33.65ID:dnuHAH8y
こうするとどうなるんだろ?

2人でじゃんけん勝負をし、n回勝ち越した方を勝者とします。勝敗が決するまでに何回勝負が行われるか、その期待値を求めてください。あいこも回数に含みます。
0510132人目の素数さん
垢版 |
2020/06/06(土) 22:32:09.93ID:I/Bajz2G
>>492

k 回勝ち越している状態から、確率1/2の勝負を行い、勝ち越し数が0になるか、2nになるかまでに
要する勝負回数の期待値をA(k)とすと、

A(k) = (1/2)*A(k+1) + (1/2)*A(k-1) + 1 ; 1≦k≦2n-1
A(0) = A(2n) = 0

という漸化式が成立する。

この漸化式の解は、A(k)=(2n-k)*k で与えられ、求めたいものは、A(n)=n^2 
0512132人目の素数さん
垢版 |
2020/06/07(日) 06:40:42.08ID:k5ENzaHc
>>510
正解です。
想定解はn-1回勝ち越している状態からもう1回勝ち越すまでにかかる回数の期待値が2n-1であることを帰納法で示す手法でした。
0513132人目の素数さん
垢版 |
2020/06/07(日) 06:44:44.68ID:k5ENzaHc
厳密には、期待値が発散しないことを示してから漸化式を解く必要がありますがこれは殆ど自明なので良いと思います。
0514132人目の素数さん
垢版 |
2020/06/07(日) 06:51:48.20ID:k5ENzaHc
n-1回勝ち越した状態から初めてn回勝ち越す状態に移行するまでに平均2n-1回かかると仮定します。
初めてn+1回勝ち越すまでにかかる回数の期待値E_(n+1)が1/2+(2n-1+1+E_(n+1))/2であるので、解いてE_(n+1)=2n+1となります。
よって帰納法でE_n=2n-1が示せて、E_(n-1),E(n-2),……も芋づる式に書けて、n^2を得るというのが想定解でした。
因みに元ネタは私の大学のレポート問題です。
0515132人目の素数さん
垢版 |
2020/06/07(日) 09:34:14.02ID:AXLE38pC
期待値=n^2を仮定して数学的帰納法での証明は難しいのかなぁ。私にはできません。
0516132人目の素数さん
垢版 |
2020/06/07(日) 12:03:01.20ID:+2q6vZNG
A(k+1)-A(k)=A(k)-A(k-1)-2=A(k-1)-A(k-2)-2*2=...=A(1)-A(0)-2*k = a-2k ; a=A(1)
A(k)=A(0)+Σ[i=0,k-1](a-2i)=0+a*k-k(k-1)=(a-k+1)*k
ここで、もう一つの境界条件 A(2n)=0を用いると、演繹的に a=2n-1 は出てきます。

あるいは、A(k+1)-A(k)=A(k)-A(k-1)-2 を
A(k)-A(k-1)=A(k+1)-A(k)+2=...=A(2n)-A(2n-1)+2*(2n-k)
と、上方向への漸化式として利用し、最初の結果、A(k)-A(k-1)=A(1)-A(0)-2*(k-1)と
A(0)=A(2n)=0、A(1)=A(2n-1) を組み合わせても、A(1)=2n-1 は出てきます。

A(k)=A(2n-k) は 対称性から自明なのですが、B(k)=A(k)-A(2n-k) が従う漸化式を解くことでも示せます。


>>513
確かにその指摘は、正当です。510の結果には、「期待値が収束するならば、」をつけておくべきでした。
0518132人目の素数さん
垢版 |
2020/06/07(日) 18:08:59.65ID:AXLE38pC
>>517
20勝勝ち越すときのジャンケンの回数(あいこは含まず)の分布がどうなるか、1万回シミュレーションしてみた。
https://i.imgur.com/PqjpGvR.png
最頻値は150程度だから、85勝65敗の戦績が最頻みたい。
この分布ってなんだろうな?
0520イナ ◆/7jUdUKiSM
垢版 |
2020/06/07(日) 20:15:57.61ID:vQmCJpRB
>>517
20勝勝ち越すには20^2=400回試合が行われるって計算したじゃん?
だったら210勝190敗したんじゃないの?
0521132人目の素数さん
垢版 |
2020/06/07(日) 20:57:56.63ID:AXLE38pC
>>520
最初から20連勝なら20回ジャンケンですむけど、その確率は2の20乗(1048576)分の1
400回は平均値(=期待値) 95%信頼区間はシミュレーションだと32〜1050にわたる。
400回ジャンケンのときは210勝190敗で400回目は勝ちだね。
0522132人目の素数さん
垢版 |
2020/06/07(日) 20:59:15.80ID:AXLE38pC
成功確率が0.5のとき成功数と失敗数の差(どちらが大きくてもよい)がnになるまでの試行回数の分布、ってことになる
0523132人目の素数さん
垢版 |
2020/06/07(日) 21:19:16.89ID:AXLE38pC
20勝勝ち越しで400回ジャンケンが行われるときの勝ち負けの配列って何種類あるんだろう?
0525132人目の素数さん
垢版 |
2020/06/07(日) 22:27:32.62ID:AXLE38pC
数が少ないと総当たりで列挙できたけど、400回ジャンケンは無理だな。



3回勝ち越しで勝敗決定のとき7回ジャンケンで勝敗が決したときの
勝ち負けの配列(勝敗差3回が7回目に初めて確定)

> J(3,7)
[,1] [,2] [,3] [,4] [,5] [,6] [,7]
[1,] 0 0 1 1 1 1 1
[2,] 0 1 0 1 1 1 1
[3,] 0 1 1 0 1 1 1
[4,] 0 1 1 1 0 1 1
[5,] 1 0 0 1 1 1 1
[6,] 1 0 1 0 1 1 1
[7,] 1 0 1 1 0 1 1
[8,] 1 1 0 0 1 1 1
[9,] 1 1 0 1 0 1 1
[10,] 1 1 0 0 0 0 0
[11,] 1 0 1 0 0 0 0
[12,] 1 0 0 1 0 0 0
[13,] 1 0 0 0 1 0 0
[14,] 0 1 1 0 0 0 0
[15,] 0 1 0 1 0 0 0
[16,] 0 1 0 0 1 0 0
[17,] 0 0 1 1 0 0 0
[18,] 0 0 1 0 1 0 0
0526イナ ◆/7jUdUKiSM
垢版 |
2020/06/07(日) 22:39:07.83ID:vQmCJpRB
>>517
なにを思って400回も試合するかだよね。
パーネル・ウィテカーにはパンチをすべて躱すっていう確固たる答えがあった。
じゃんけんなんか対戦者のあいだに差があるとは思えないのに、なんで20勝も差が出るのか実感できない。
なんで20勝どまりなのかという感じもする。多すぎず、少なすぎずもっともありうる勝ち数の差が、
√試合数ってことか。
0528132人目の素数さん
垢版 |
2020/06/08(月) 02:27:17.40ID:BZyerOGC
In[1]:= A[n_,t_]:=A[n,t]=If[t==0,If[n==0,1,0],If[n==0,2*A[1,t-1],If[n>18,A[n-1,t-1],A[n+1,t-1]+A[n-1,t-1]]]]
In[2]:= Table[A[20,t],{t,20,300,2}]
Out[2]= {1, 20, 230, 2000, 14625, 95004, 566370, 3164480, 16811300, 85795600, 423830264, 2038362560,
> 9586673915, 44246187300, 200966926350, 900331830048, 3985844039275, 17464206951100,
> 75831424919250, 326658445806000, 1397281501935164, 5939663527389840, 25108577638509960,
> 105613430714022400, 442256241114424425, 1844503362933849060, 7664867653576145950,
> 31746685883395530800, 131096805536410875625, 539886351785258307500, 2217853132475244283162,
> 9090251922987325452960, 37180441420243010660340, 151782933822040596272400,

In[3]:= Sum[2*t*2^(-t)*A[20,t],{t,1,2000}]//N
Out[3]= 393.852
In[7]:= Table[10000*2^(-t)*A[20,t],{t,20,250,2}]//N
Out[7]= {0.00953674, 0.0476837, 0.137091, 0.298023, 0.544824, 0.884794, 1.31868, 1.84197, 2.44637, 3.12123, 3.85471, 4.6347, 5.44939, 6.28776,
> 7.13978, 7.99655, 8.85035, 9.69458, 10.5237, 11.3332, 12.1195, 12.8796, 13.6114, 14.3133, 14.9842, 15.6236, 16.231, 16.8066, 17.3505, 17.8633,
> 18.3457, 18.7982, 19.2218, 19.6175, 19.9861, 20.3287, 20.6464, 20.9401, 21.2108, 21.4597, 21.6877, 21.8958, 22.085, 22.2562, 22.4104, 22.5483,
> 22.6708, 22.7788, 22.873, 22.9542, 23.023, 23.0802, 23.1264, 23.1623, 23.1883, 23.2051, 23.2133, 23.2132, 23.2054, 23.1904, 23.1686, 23.1404,
> 23.1061, 23.0662, 23.021, 22.9708, 22.916, 22.8568, 22.7935, 22.7264, 22.6557, 22.5817, 22.5046, 22.4247, 22.342, 22.2568, 22.1693, 22.0796,
> 21.988, 21.8945, 21.7993, 21.7025, 21.6043, 21.5047, 21.404, 21.3021, 21.1993, 21.0955, 20.991, 20.8857, 20.7797, 20.6731, 20.5661, 20.4586,
> 20.3507, 20.2425, 20.1341, 20.0254, 19.9166, 19.8076, 19.6986, 19.5895, 19.4805, 19.3715, 19.2626, 19.1537, 19.045, 18.9365, 18.8282, 18.7201,
> 18.6122, 18.5046, 18.3973, 18.2903, 18.1836, 18.0772}
0529132人目の素数さん
垢版 |
2020/06/08(月) 05:47:57.56ID:HhtoQFdr
>>524
こっちの方が星取り表みたいでいいな。

> J(3,7)
[,1] [,2] [,3] [,4] [,5] [,6] [,7]
[1,] ● ● ○ ○ ○ ○ ○
[2,] ● ○ ● ○ ○ ○ ○
[3,] ● ○ ○ ● ○ ○ ○
[4,] ● ○ ○ ○ ● ○ ○
[5,] ○ ● ● ○ ○ ○ ○
[6,] ○ ● ○ ● ○ ○ ○
[7,] ○ ● ○ ○ ● ○ ○
[8,] ○ ○ ● ● ○ ○ ○
[9,] ○ ○ ● ○ ● ○ ○
[10,] ○ ○ ● ● ● ● ●
[11,] ○ ● ○ ● ● ● ●
[12,] ○ ● ● ○ ● ● ●
[13,] ○ ● ● ● ○ ● ●
[14,] ● ○ ○ ● ● ● ●
[15,] ● ○ ● ○ ● ● ●
[16,] ● ○ ● ● ○ ● ●
[17,] ● ● ○ ○ ● ● ●
[18,] ● ● ○ ● ○ ● ●
0531イナ ◆/7jUdUKiSM
垢版 |
2020/06/08(月) 20:55:06.88ID:Dtpzwa4D
>>526
なんでか急に表示システムが変わったね。
ほかのスレが見あたらないんだが。
ギガ数の関係なら今月はもう無理だと思う。
0532132人目の素数さん
垢版 |
2020/06/08(月) 21:55:10.75ID:BZyerOGC
>>52/8のOut[2]は、次の式で、表現できるようです。

In[28]:= Table[Sum[Binomial[20+2i-1,i+40k]-Binomial[20+2i-1,i-1+40k],{k,0,(20+2i)/40}],{i,0,40}]
Out[28]= {1, 20, 230, 2000, 14625, 95004, 566370, 3164480, 16811300, 85795600, 423830264, 2038362560, 9586673915, 44246187300, 200966926350,
> 900331830048, 3985844039275, 17464206951100, 75831424919250, 326658445806000, 1397281501935164, 5939663527389840, 25108577638509960,
> 105613430714022400, 442256241114424425, 1844503362933849060, 7664867653576145950, 31746685883395530800, 131096805536410875625,
> 539886351785258307500, 2217853132475244283162, 9090251922987325452960, 37180441420243010660340, 151782933822040596272400,
> 618540254546926375123800, 2516575194382944375198528, 10223586645582059997655395, 41476057972670469339852180, 168049542131127131360839350,
> 680085742675872390532987200, 2749246507570540057967664162}

528は、パスカルの三角形に於いて、端(n=20近辺)での生成式を無理矢理変更して、計算させてものですが、
初期条件を、...,A[0,0]=1、A[40,0]=-1、A[80,0]=1,A[120,0]=-1,...
のようにすれば、恒等的にA[20,t]=0 となるので、生成式は、全て同一にできます。
肝心のn=20での値が消えますが、これは、A[19,t-1]に保存されているものを用います。
0533132人目の素数さん
垢版 |
2020/06/08(月) 21:56:37.47ID:BZyerOGC
×:>>52/8のOut[2]は、次の式で、表現できるようです。
○:>>528のOut[2]は、次の式で、表現できるようです。
0534132人目の素数さん
垢版 |
2020/06/12(金) 15:16:33.72ID:h4pf/tcK
>>532は大きなところで、ずれていることに気づきました。修正します。

k 試合目で、初めて n 回勝ち越す勝敗パターンの数は、kとnの偶奇が一致しないときは 0 で、一致するときは、

Sum[(Binomial[k-1,(k-(2i+1)n)/2]-Binomial[k-1,(k-(2i+1)n)/2-1])*(-1)^i,{i,0,k/n-1}]

で与えられます。
528で確認できる最も大きなものは、A[20,86]=151782933822040596272400ですが、
上の式を使えば、C[85,33]-C[85,32]-(C[85,13]-C[85,12]) で計算できます。
0536132人目の素数さん
垢版 |
2020/06/12(金) 20:10:25.45ID:h4pf/tcK
非常に適切な表現ですね。私は勝手に「ガター付きランダムウォーク」って命名してました。

「ランダムウォーク 鏡像法」でググると、「kステップ後に、初めて吸収壁nに到達する経路の数」が

(n/k) C[k,(n+k)/2]

で与えられるという記述を見つけました。(同じ内容になるよう、文字は変更してます。)

あれ?異なる! と思いましたが、この問題はルートの途中で -n 勝になってもいいようで、
微妙に問題設定が異なっていて安心しました。
0537132人目の素数さん
垢版 |
2020/06/12(金) 21:03:38.62ID:oLgfPUZm
おっしゃる通りで、吸収壁が左右に二つある場合にあたります
詳しくは
 臨時別冊数理科学 SGCライブラリ 47 「現代物理数学への招待」ランダムウォークからひろがる多彩な物理と数理 2006年 05月号
にあります
わたしはこの本で知りました
0538132人目の素数さん
垢版 |
2020/06/13(土) 01:45:11.65ID:Gyrz8+tN
高校数学の範囲で証明できる不等式で、難解なものを教えてください。
0539132人目の素数さん
垢版 |
2020/06/13(土) 04:05:00.27ID:HTMGcHIc
x^4 + x^3 - 2x + 1 ≧ 0,

[高校数学の質問スレPart404.051,070] ← 本来はこちらへ
[不等式スレ10.381-384]
0540イナ ◆/7jUdUKiSM
垢版 |
2020/06/13(土) 13:02:47.66ID:CPLScqnr
>>526
>>539
f(x)=x^4 + x^3 - 2x + 1とおくと、
f'(x)=4x^3+3x^2-2
f(0)=f(1)=1
f'(0)=-2
f'(1/4)=1/16+3/16-2=-7/4
f'(1/2)=1/2+3/4-2=-3/4
f'(3/4)=4(27/64)+27/16-2=27/8-2=11/8
f'(2/3)=4(8/27)+4/3-2=32/27+36/27-2=(68-54)/27=14/27
f"(3/5)=4(27/125)+27/25-2=(108+135)/125-2=243/125-2=-7/125
f(2/3)=16/81+8/27-4/3+1=40/81-1/3=13/81>0
f(3/5)=81/625+27/125-6/5+1=216/625-1/5=91/625>0
f'(0.61)=
f'(0.62)=
0541132人目の素数さん
垢版 |
2020/06/13(土) 16:19:51.54ID:lPWHBZNf
次の漸化式で表される数列の一般項を積分、総和記号を用いて表現せよ.
a(1) = 1
a(n+1) = (n^2+3n+1)a(n) + n
0542イナ ◆/7jUdUKiSM
垢版 |
2020/06/13(土) 16:52:33.34ID:CPLScqnr
>>540
>>539
f(x)=x^4+x^3-2x^2+1とおくと、
f(0)=f(1)=1
x→-∞のときf(x)→+∞
x→+∞のときf(x)→+∞
いずれも単調増加でf(x)>1
f'(x)=4x^3+3x^2-2
f'(0.607007295624696)=0
f(x)はx=0.607007295624696のとき最小値をとる。
f(0.607007295624696)= 0.14540320838>0
∴示された。
0543132人目の素数さん
垢版 |
2020/06/13(土) 17:31:23.41ID:3lrNZ+U5
] 定係数線形2階微分方程式、
a2x′′(t) + a1x′(t) + a0x(t) = 0
の一般解は、x(t) = eλt と仮定し、λ に対する2次方程式を解くことによっ て
求めることができる。ここで、x′′(t) = d2x(t) dt2 、x′(t) = dx(t) dt である。λ が重解 λ0 のときは、x(t) = C(t)eλ0t と仮定することによって解を求めるこ とができる。C(t) が、 C′′(t) = 0
を満たすことを示せ。
0544132人目の素数さん
垢版 |
2020/06/13(土) 17:43:17.46ID:3lrNZ+U5
>>543
eλtはeの λt乗です
0545132人目の素数さん
垢版 |
2020/06/13(土) 18:22:52.30ID:152gx922
kを2以上の整数とするとき,
10進法において,以下の条件を満たす最大の自然数nについて,
nに現れる1の個数はちょうど2^(k-1)であることを証明せよ.

[条件]
nから連続するk桁を選び, k桁の数を得るとする.
すべての選び方を考えたとき,それらは全て異なる.

(例)
k=3 のとき, n=212212 という数を考える
連続3桁の選び方は 全部で 212,122,221,212 の4通りだが
212がダブっているため, すべて異なるの要件を満たさない
0549132人目の素数さん
垢版 |
2020/06/15(月) 11:26:22.21ID:Ep83CBa+
自作問題考えたから誰か解いてみてください笑笑

(1)麻雀の役満に含まれる漢数字を全て足すと幾つ?
(これはひっかけですw)
(2)6面サイコロをn個振ったとき、その出目の積がkの倍数になる確率をp(n,k)とする。
このとき
(2-1)kが7以上の素数の時pを求めよ!
(2-2)k=n^nのとき、pを求めよ
(2-3)k=2^nのときpをもとめよ
(2-4)pをn,kで表せられるか、表せられるなら表せ。
0551132人目の素数さん
垢版 |
2020/06/15(月) 14:22:26.59ID:FRXVIMl9
>>550
546とか元々のスレに答えがあるけど、

x^4 + x^3 - 2x + 1
=(1/4 )x^4 + x^3 - 2x + 1 + (3/4) x^4
=(1/4) (x^2+2x-2)^2 + (3/4) x^4>0
っていうことでしょ。

そして、 √3 -1 は x^2+2x-2=0 の解
0552539
垢版 |
2020/06/15(月) 16:59:52.08ID:m4MzqaBi
>>551
正解です!
0553132人目の素数さん
垢版 |
2020/06/16(火) 11:18:25.94ID:4svmpCM1
全ての成分が1であるn次行列をMとする

(1)
対角成分がλ_1,λ_2,・・,λ_nである対角行列をΛとするとき
det(Λ+λ_0M)がλ_0,λ_1,・・,λ_nの対称多項式であることを示せ

(2)
一般の正方行列をAとしたとき
det(A+λM)はどのように書けるか?
0554132人目の素数さん
垢版 |
2020/06/16(火) 13:30:42.75ID:MA7a0AZ4
(1)
 det(Λ+λ_0 M) = Σ[i=0,n] (Π[j≠i] λ_j)
  = Σ[i=0,n] (λ_i 以外のλの積)
  = f(0) - λ_0・f '(0)
ここに f(x) = det(Λ - x E).
 
(2)
相似変換により対角化しても det は変わらないから
 det(A+λM) = f(0) - λ・f '(0)
ここに f(x) = det(A - x E).

det(A+λM-xE) = f(x) - λ・f '(x)
0555132人目の素数さん
垢版 |
2020/06/16(火) 14:35:09.12ID:4svmpCM1
>>554
(1)
その最初の等式が示したいことなんですが、それはなぜ言えますか?

(2)
Aを相似変換したとき、Mも相似変換されるので(1)をそのままでは使えないように思うのですが、なぜそれで大丈夫なのでしょうか
0556132人目の素数さん
垢版 |
2020/06/17(水) 09:40:59.28ID:6zZiGwb2
>>549
ローカル役満を含むか否か
0557132人目の素数さん
垢版 |
2020/06/17(水) 16:43:02.27ID:WlGjA3y4
文字列 "麻雀の役満" について各桁の各部首に漢数字は含まれないから 0とか?
0558132人目の素数さん
垢版 |
2020/06/17(水) 16:47:09.47ID:WlGjA3y4
少なくとも個々の役満について考える問題ではないだろう
たとえば天和があるから そのときの考えられる形は萬子だけに限っても膨大
ローカル役満もそうだけど それらに言及がないからね
0559132人目の素数さん
垢版 |
2020/06/17(水) 18:02:30.31ID:2VlunGf6
役満「清老頭」の「清」が漢数字ってことに気づくかどうかってだけです
ほんと下らない問題でごめんなさい
0560132人目の素数さん
垢版 |
2020/06/17(水) 18:20:08.41ID:nfWpvKlp
なぜ下らない問題スレに書かなかったかw

それはそうとして、漢数字「清」とはいくつを指す文字なん?
0561132人目の素数さん
垢版 |
2020/06/17(水) 19:32:17.08ID:2VlunGf6
>>560

いや面白いかなと思って笑笑
10^(-23)やで
0562132人目の素数さん
垢版 |
2020/06/17(水) 19:57:08.70ID:WlGjA3y4
清老頭といってもいくつかパターンがあるから全部足すの意味がよくわからん
かりに清の意味だとしても "足す"の意味がよくわからないんだが...答えは何?
0564132人目の素数さん
垢版 |
2020/06/17(水) 20:18:48.08ID:p67rPnvF
ローカル役満というだけあって何を役満とするかは流儀があるし、
役満の呼び方も種類がある
国士無双と呼ぶか十三么九と呼ぶかで解は異なる。

数学向けの問題じゃないんだわ
0565132人目の素数さん
垢版 |
2020/06/17(水) 23:02:15.11ID:WlGjA3y4
萬子で清一色をつくることを考える
このとき考えられるすべての和了形において漢数字の総和を求めよ
ただし門前のツモ和了のみを考え 待ちの区別はしないものとする
(ひっかけでもなんでもなく純粋に組み合わせの問題だが かなり面倒のハズ)
0566132人目の素数さん
垢版 |
2020/06/17(水) 23:23:02.93ID:Iebo131J
>>563
>清浄
清と浄だよ
0567132人目の素数さん
垢版 |
2020/06/18(木) 00:10:23.49ID:nv9HHRvP
カンツで1111持ってて

111122233344455

もあり?
あと
11122233344455
は三コウとも一色三順序ともみれるけど別カウントする?
別カウントするなら
111 234 234 234 55
123 123 123 444 55
の2通りに上がれるけどワンカウント?別カウント?
まぁどのみち数学的にうまい数え上げ考えるよりプログラム組んだ方が良さげだけど。
0568132人目の素数さん
垢版 |
2020/06/18(木) 00:13:40.96ID:nv9HHRvP
あぁ、あたまも考えないとダメか。
11123444666777

11 123 444 666 777

44 111 234 666 777
の2通り上がれるけどコレも別カウントかワンカウントか決めないと。
0569132人目の素数さん
垢版 |
2020/06/18(木) 03:06:13.92ID:ynZ6Bcxv
とりあえず上がり方が複数ある場合全部ワンカウントなら
カンツなし  13259
1カンツ   15643
2カンツ   9435
3カンツ   3360
4カンツ   630
になった。
0570132人目の素数さん
垢版 |
2020/06/18(木) 10:34:12.17ID:yuRO46b1
清浄 = 10^-21 しか知らんかったけど
接頭語は zepto-
zepto〜 は 〜 の10^(-21) を表わす。
0571132人目の素数さん
垢版 |
2020/06/18(木) 10:57:06.01ID:GJYmz6zQ
同じく清浄=10^(-21)しか知らない

もっとも古い文献は読みにくくて、例えば無量大数(10^68)を無量(10^68)と大数(10^72)の2つだと解釈する人がいたりして、かなりカオス
極端に大きな単位と極端に小さな単位は実用的ではないのでは?
0572132人目の素数さん
垢版 |
2020/06/18(木) 11:18:53.52ID:+eD6AEfH
虚空清浄は4つ別々
0573132人目の素数さん
垢版 |
2020/06/18(木) 13:38:17.93ID:yuRO46b1
空気清浄機のこと。
シャープ、クオfuture、トヨトミ ほか
平均で 15000円ぐらいか?
0574132人目の素数さん
垢版 |
2020/06/18(木) 13:49:20.86ID:yuRO46b1
空気星条旗
 T大統領のツィートに初めて「事実確認」の警告ラベルが・・・・(5/26)
0575132人目の素数さん
垢版 |
2020/06/18(木) 15:00:07.65ID:bMfHt7IQ
虚空清浄を4つ分けにするか2つ分けにするかは諸説ある(広辞苑第六版は2文字分け派)ようだけど、ここでは4つ分けなのね
ちなみに阿頼耶以降の小数は出典が不明で使わないほうがいい
0576132人目の素数さん
垢版 |
2020/06/18(木) 21:22:25.73ID:SM7HVIA0
>ここでは4つ分け
いや勝手に決めんなし
2つ分けのほうが合理的
0578132人目の素数さん
垢版 |
2020/06/19(金) 02:52:26.26ID:spqAMhcp
別スレの問題の改題

a1〜a6を最大公約数が1である整数の組みで総和sは0以上とする。
サイコロの各面にa1〜a6の数字を書き込み、最初0にある数直線上の動点をサイコロを振って出た目の数だけ移動させる試行を繰り返す。
何回目かの地点で点nに止まる確率をpnとする。
lim[n→∞]pnを求めよ。
0580132人目の素数さん
垢版 |
2020/06/19(金) 19:15:14.87ID:CTrYxzTF
全てのnについて『点(n)を一回でも通る確率が1である』ことを示すのも
それほど簡単では無さそうだけれども…
0583132人目の素数さん
垢版 |
2020/06/20(土) 07:02:20.19ID:WlbS4rFL
あ、もしかしてa1〜a6は全て正ってこと?

負があるといつまでも試行が終わらないからpnの定義が謎だったんだが
0584132人目の素数さん
垢版 |
2020/06/20(土) 08:42:04.95ID:WlbS4rFL
いや、k回目で初めてnに来る確率をpn(k)として
pn=Σ(k=1〜∞)pn(k)と定義するのか
0585132人目の素数さん
垢版 |
2020/06/20(土) 10:21:39.98ID:H0q0bxsF
>>582
偶数だけなら?
0586132人目の素数さん
垢版 |
2020/06/20(土) 11:26:25.29ID:WlbS4rFL
最大公約数がdの場合は、目をai/d、dの倍数マスのみを数直線とみなした場合の元の問題に帰着できるはず

負のaiがある場合はlim pnは1か1/2になりそうだけどどうなんだろうか
0587132人目の素数さん
垢版 |
2020/06/20(土) 11:36:20.27ID:/qNPwcAR
>>583
s>0ならいつまでも可能性は残るけど、でも確率はさだまるよ。
ピーターウィンクラーのパズル本にa1=3, a2=-2, a3〜a6=0の場合が取り上げられてる。
それは大数の法則使う解法だけど元スレの解法を応用した方が気持ちよく解ける。
0589132人目の素数さん
垢版 |
2020/06/20(土) 11:51:16.39ID:WlbS4rFL
>>587
それは>>584の定義でok?

正の目の合計が多いときは1、正負の目の合計と同じ場合は1/2、負の目の合計が多いときは0となりそうだと思ってるんだけど、もっとaiたちに依存した答えになるの?
0590132人目の素数さん
垢版 |
2020/06/20(土) 11:53:34.24ID:WlbS4rFL
いや、正負の目の合計が同じでも分散が違うと振舞いが違ってくるか
0591132人目の素数さん
垢版 |
2020/06/21(日) 09:48:21.46ID:GMWA6QXT
集合Mに結合的な演算●:M×M→Mがあり
∀m∈Mでm●(-):M→Mが全単射なものを考える

(例1)群演算
群Gに対して、群の乗法は上記を満たす

(例2)右演算
集合Rに演算をa●b=b(∀a,b∈R)と定義すると上記を満たす

実は本質的にこの2例しかない
すなわちMはある群Gと右演算を持つRの直積に同型
(M=G×R)であることを示せ
0592132人目の素数さん
垢版 |
2020/06/21(日) 16:03:57.29ID:RkIiN0Jm
>>578
出題者です。
勘違いでスルッと出ると思ってだけどai<0を許すと中々綺麗にはまとまらないようです。
申し訳ない。
なので改題して

a1=a2=a3=2, a4=a5=a6=-1
のときlim[n→∞]pnを求めよ。

コレならまぁ出せる。
0596132人目の素数さん
垢版 |
2020/06/22(月) 23:30:22.66ID:eqi+y1z3
素数p(≧3)について
p+1から下記の操作を繰り返し行なって、偶数の場合の操作が合計p-1回のときに1となることを証明せよ。
•偶数ならば2で割る
•奇数ならばpを足す
0597132人目の素数さん
垢版 |
2020/06/22(月) 23:59:04.54ID:05JjzqDu
>>591
コレやっぱりダメなんじゃないの?
もし主張が正しいなら条件を満たす演算は
1) 群である。
2) 右単位元を持たない。
のいずれかが成立する。
実際Rのパートが一元からなる自明なものなら全体は群となるし、Rが二x、yをもてば、(a,x)(b,y)=(ab,y)、(a,y)(b,x)=(ab,x)によりGの元bを何に選んでも右単位元となり得ない。
ここでMとして無限次元ベクトル空間の全射線形写像の全体として合成積を●とすれば、∀m∈Mについてm●(-)は全射である。
実際任意のn∈Mに対して方程式m●x=n‥(1)はmが全射準同型だから右逆元lを持つので、x=lnが(1)の解になる。
すなわちm●(-)は全射である。
もちろん合成積だから結合的。
しかしMは群ではなく、恒等写像を右単位元とするので先の主張に反する。
0599132人目の素数さん
垢版 |
2020/06/23(火) 00:24:43.02ID:t68c0n/W
>>596
・2で割る(偶数の時)
・p足して2で割る(奇数のとき)
のいずれかをp-1回行った時1となる事を示せば良い。
最初の1回目でp以下となり、以降常にp以下である。
またp-1回目でxに到達したとすれば
2^(p-1)x≡(p+1) (mod p)
である。
0601132人目の素数さん
垢版 |
2020/06/23(火) 01:02:01.10ID:XVtNFiCj
(´・ω・`)
たまたま目にした企業の入社試験なのですが問2がわかりません。教えてください。

https://dotup.org/uploda/dotup.org2182228.pdf
0602132人目の素数さん
垢版 |
2020/06/23(火) 01:03:42.76ID:uFrMHeah
>>591
M = Q − {0}, a●b = |a|・b
なら群にならないのでは
0603132人目の素数さん
垢版 |
2020/06/23(火) 01:28:24.39ID:VX8FM0/Y
>>599
すごい

>またp-1回目でxに到達したとすれば
>2^(p-1)x≡(p+1) (mod p)
>である。

これは、 k 回目の値を x_k とすると、 x_1 = (p+1)/2 より、
2^k x_k ≡ (p+1) (mod p)
となることが k についての帰納法でわかるということですかね?
0605132人目の素数さん
垢版 |
2020/06/24(水) 00:45:33.81ID:gs0zs19c
>>591
まず任意のmについてme=mを満たすeが一意に定まるのでコレをe(m)と書く。
この時任意のm,nについて
me(m)n = mn
によりe(m)n=n、よって常にe(m)は左単位元となる。
m0を任意にとり
G={g | e(g)= e(m0)}、R={r | e(r) =r}
とおく。
g∈Gにたいしge(m0)=ge(g)=gであるからe(m0)はGの右単位元であり、左単位元でもあったからGは単位元を持つ。
さらにgh=e(m0)であるhが取れるが、ghe(m0)=e(m0)e(m0)=e(m0)=ghによりhe(m0)=h=he(h)となり、e(h)=e(m0)を得てh∈Gである。
さらにghg=e(m0)g=g=ge(m0)からhg=e(m0)となり、Gは両側可逆である。
結合性は明らかだからGは群となる。
任意のrにたいしe(r)は左単位元だから、r,s∈Rに対しrs=e(r)s=sとなりRの積は右演算である。
g,h∈G、r,s∈Rがgr=hsを満たす時、g=ge(m0)=gre(m0)=hse(m0)=he(m0)=hであり、よってr=sである。
またm∈Mを任意にとるときm=me(m)=me(m0)e(m)、me(m0)∈G、e(m)∈RからM=G×Rである。
0606132人目の素数さん
垢版 |
2020/06/24(水) 01:34:49.85ID:QGC2w+2k
>>605
正解です!

ちなみにですが
演算のカリー化 M→ Hom(M,M) は
演算の結合性により準同型、
問題の条件により像が群になっていて
この像がG、核(=idの逆像)がRになっています
0608132人目の素数さん
垢版 |
2020/06/24(水) 22:41:50.91ID:QGC2w+2k
>>607
カリー化という言葉を使ったのは便利だったからで計算論がらみというわけではないです
昔、代数関係で出会った問題でした
0609132人目の素数さん
垢版 |
2020/06/25(木) 09:37:46.39ID:ugKDGz4C
>>596
p+1 ≡ 1 (mod p)
2で割ることは 2^(-1) =a を掛けること。
φ(p)回後には ≡ a^φ(p) ≡ 1 (mod p) ・・・・ オイラーの定理
φ( ) はオイラーのtotient函数。
また、pを足した直後を除いて 1〜p-1 の範囲にあるから = 1

* mod p で考えて 2, 2^(-1) の位数がr なら r回後に初めて1になる。
p=31 なら r=5
0612132人目の素数さん
垢版 |
2020/06/25(木) 23:58:08.52ID:8gb5jAWW
S = {1,2,3,・・・,n}
A1, A2,・・・, An をすべて異なるn個のSの部分集合とする
このとき、A1 - {x}, A2 - {x},・・・, An - {x} がすべて異なるような x∈Sが存在することを示せ
(Aiにxが含まれない場合は、Ai - {x} = Ai)
0616132人目の素数さん
垢版 |
2020/06/26(金) 01:01:40.66ID:/now+Nzk
>>612
Aiの全体を頂点の集合Vとする向き付きのグラフGを
Ai→Aj iff Aj = Ai ∪{x} ∃x
とし、→の上にxを書き込んでおく。
矢印の上に出現した文字の全体をA(G)として、コレを変化させないようにGから辺を取り除いていく。
Gが木になったら終了。
Gが木でないならループを一つ選ぶ。
ループ内の頂点Aiを一つ選んでそこから始めて一周するとき文字を一文字ずつ追加または削除することになるが、一周して元に戻るので同じ文字が書き込まれた矢印が少なくとも1組ある。
そのどちらかの矢印を取り除く。
最終的に辺の上に文字の書き込まれた木を得るが、現れる文字の全体は元のグラフと同一である。
一方で、木であるから辺の数は頂点の数より少ないので現れていない文字が少なくとも一つある。
それをxとして選べば良い。
0617132人目の素数さん
垢版 |
2020/06/26(金) 03:45:04.95ID:uIPUa6II
F_n をフィボナッチ数列
F_0 = 0,
F_1 = 1,
F_(n+2) = F_(n+1) + F_n (n ≧ 0)
とする。

φ := (1+√5)/2 と置く。

複素数 z に対し、関数 E(z) := Σ[n=0,∞] F_n z^n / n! を考える。

(1) E(z) は全ての複素数 z に関して絶対収束することを示せ。
(2) 不等式 E(1) > 2 が成り立つことを示せ。
(3) 不等式 (1/√5) (e^φ - e^(1-φ)) > 2 が成り立つことを示せ。
(4) Σ[n=0,∞] F_(3n+1) i^(3n+1) / (3n+1)! を求めよ。ただし、 i = √(-1) とする。

複素数 z に対し、関数 G(z) := Σ[n=0,∞] F_n z^n を考える。

(5) G(z) は |z| < 1/φ = 0.618… を満たす複素数 z に関して絶対収束することを示せ。
(6) z が有理数かつ |z| < 1/φ のとき、 G(z) も有理数となることを示せ。
(7) G(1/2) を求めよ。
(8) m = 2, 3, 4, … のとき、 m / G(1/m) は常に整数となることを示せ。
(9) G_a,b(z) := Σ[n=0,∞] F_(an+b) z^(an+b) と置くとき、
G_3,0(1/2), G_3,1(1/2), G_3,2(1/2) は全て有理数となる。
このとき、不等式 G_3,0(1/2) < G_3,2(1/2) < G_3,1(1/2) が成り立つことを示せ。
0619132人目の素数さん
垢版 |
2020/06/26(金) 22:52:23.36ID:pGwHB1As
>>612
Sの部分集合全体からなる集合Vを、
対称差△を加法とすることで二元体上n次元ベクトル空間とみなす。
n-1個の元 (A1△A2), … ,(A1△An) が張るVの部分空間Wの次元はn-1以下であるから、
Wは全ての m∈{1,…,n} について{m}を元に持つことはできない。
ゆえに、どの x,y∈{1,…,n} をとっても
Ax△Ay = (A1△Ax)△(A1△Ay) ∈ W
が{m}と等しくならないようなmが存在する。
0621132人目の素数さん
垢版 |
2020/06/27(土) 00:32:54.13ID:rTsmeJAV
>>578
出題者です。
s=0の場合の当初用意した解答の議論が間違ってたんですが、やっと修正できた。
s=0の場合はpnの一般項が全て1になるようです。
すなわちどんなnをとっても確率1で動点はいずれかの時点でnに来るようです。
s>0の場合には元スレに出てた解答を微調整ですみます。
0622132人目の素数さん
垢版 |
2020/06/27(土) 00:51:56.54ID:u6/tJncy
>>619
この解き方もシンプルでいいですな
一応用意した612の答えは、0,1を成分とする行がすべて異なるn×n行列に置き換えて
正則かどうかで場合分けするものでした
0623132人目の素数さん
垢版 |
2020/06/27(土) 07:16:52.39ID:wWzRNyRw
F_nをn番目のフィボナッチ数とする (i.e. F_1=F_2=1,F[n+2]=F[n+1]+F[n])
このとき Σ[n=0,∞]1/(1+F[2n+1]) = √5/2 を証明しなさい

問題の出典はWikipedia(Fibonacci number)ですが証明が書いてないです
誰かこの等式の証明をかいてくれませんか?
0625132人目の素数さん
垢版 |
2020/06/27(土) 17:02:41.29ID:VHLRrvKl
>>624
動画が長いから、掲示板に書くには余白が少なすぎる問題かと思ったけど、簡単に言うと
Σ[n=0→k] 1/(1+F[2k+1]) = 1/2 + F[2n]/(1+F[2n+1]) を示す
lim[n→∞] (1/2 + F[2n]/(1+F[2n+1])) = 1/2 + (-1+√5)/2 = (√5)/2 を導く
ってことなのね
0626132人目の素数さん
垢版 |
2020/06/27(土) 17:05:13.43ID:VHLRrvKl
>Σ[n=0→k] 1/(1+F[2k+1]) = 1/2 + F[2n]/(1+F[2n+1]) を示す
訂正
Σ[k=0→n] 1/(1+F[2k+1]) = 1/2 + F[2n]/(1+F[2n+1]) を示す
0627132人目の素数さん
垢版 |
2020/06/27(土) 17:14:03.47ID:31Ny2A7Z
 1 + F[4n-1] = F[2n-1]・(F[2n+1] + F[2n-1]),
 1 + F[4n+1] = F[2n+1]・(F[2n+1] + F[2n-1]),
2項をまとめて
1/(1+F[4n-1]) + 1/(1+F[4n+1])
 = 1/(F[2n+1]・F[2n-1])
 = (F[2n+1]・F[2n-1] - F[2n]^2) /(F[2n+1]・F[2n-1])   (Lemma 1)
 = (F[2n+1]+2F[2n])/(2F[2n+1]) - (2F[2n]-F[2n-1])/(2F[2n-1]),
 = (F[2n+2]+F[2n])/(2F[2n+1]) - (F[2n]+F[2n-2])/(2F[2n-1]),

Σ[k=0,2n] 1/F[2k+1] = {F[2n+2]+F[2n]}/(2F[2n+1])
  → (φ + 1/φ)/2   (← Lemma 2)
  = (√5)/2,

Lemma 1
 F[m]・F[n+1] - F[m+1]・F[n] = (-1)^n F[m-n] (m≧n)
 F[n-1]・F[n+1] - F[n]^2 = (-1)^n

Lemma 2
lim[n→∞] F[n]/F[n+1] = 1/φ,
0629132人目の素数さん
垢版 |
2020/06/27(土) 17:46:46.86ID:31Ny2A7Z
>>624
stack_exchange さんのコメント
「ビネの式」 F[m] = {φ^m - (-1/φ)^m}/√5 から次のように書ける。
1/(1+F[2n+1]) = (√5)/{(√5) + φ^(2n+1) + (1/φ)^(2n+1)}
 = (√5)φ^(2n+1) /{1 + (√5)φ^(2n+1) + φ^(4n+2)}
 = (√5){φ^(2n+2) - φ^(2n)}/{1 + φ^(2n) + φ^(2n+2) + φ^(4n+2)} (*)
 = (√5){φ^(2n+2) - φ^(2n)}/({1+φ^(2n)}{1+φ^(2n+2)})
 = (√5)(1/{1+φ^(2n)} - 1/{1+φ^(2n+2)})
∴ telescoping するだけで無限和は (√5)/2 になる。

*)  1 = φ - 1/φ, √5 = φ + 1/φ,

arctan版もある。
http://www.youtube.com/watch?v=pn0J0p0R_GM 13:25
0630132人目の素数さん
垢版 |
2020/06/27(土) 20:00:56.78ID:31Ny2A7Z
動画が長いから、掲示板に書くには余白が少なすぎる問題かと思ったけど、簡単に言うと
(F[2n]-1)(1+F[2n-1]) - F[2n-2](1+F[2n+1])
 = (F[2n]-1)(1+F[2n-1]) - (F[2n]-F[2n-1])(1+F[2n+1])
 = F[2n-1]・F[2n+1] - F[2n]^2 -1 + F[2n](F[2n-1]+F[2n]-F[2n+1])
 = F[2n-1]・F[2n+1] - F[2n]^2 -1
 = 0      (← Lemma 1)

∴ 1/(1+F[2n+1]) = F[2n]/(1+F[2n+1]) - F[2n-2]/(1+F[2n-1]),

Σ[k=0,n] 1/(1+F[2k+1]) = 1/2 + F[2n]/(1+F[2n+1]),
以下 >>625-626
ってことなのね
0632132人目の素数さん
垢版 |
2020/06/29(月) 09:10:51.41ID:pLxgzLOg
関数 f(x) は
x が無理数の時 、f(x) = 0
x が有理数の時、 x = n/m とし、 f(x) = 1/n をとる。

区間 0 < x < 200 で、
x が有理数の時、
f(x) の最小値 とその時のxの値を答えよ。
0634132人目の素数さん
垢版 |
2020/06/29(月) 09:43:30.02ID:XuyQ1NeQ
>>632
xが有理数のときの定義が不完全な気がするけど
良心で解釈すると xが正の有理数のときは x=n/mを満たす互いに素な
正の整数n,mの組が取れて そのときは f(x) = 1/n と定めるということ(だとおもう)

それで回答としては 「最小値は存在しない」

一方、最大値のほうをきいてるなら それは 1 となる
関数の定義上, f(x) = 1/n ≦ 1 がいえるから
f(x)=1となるような有理数x=n/mを求めると
それは明らかに x=1/m の形の正の有理数全体となる
しかし 最大値のタイプミスだとすると 200未満の条件を用いていないから...
つまり 問題文(そのまま解釈すると最小値は存在しないになる)はどこかおかしいのだが
どこがおかしいのかよくわからない もとの問題文を推察するのが困難
0635132人目の素数さん
垢版 |
2020/06/29(月) 10:47:11.85ID:tMDtq57r
200とか2020とか
作った年や関係ない数字を問題文に入れるのは
数学の大会問題、懸賞問題ではよくあること
気にしなくていいと思うよ

よく練られてない自作問題を投げる人も多いし
0636132人目の素数さん
垢版 |
2020/06/29(月) 19:23:59.09ID:cwExuDQs
実数 x に対し、 {x} で x の小数部分 ( 0 ≦ {x} < 1 ) を表す。

(1) lim_[n→∞] {n!*e} を求めよ。ここで e はネイピア数 e = 2.718… とする。
(2) lim_[n→∞] {n!*x} = 1 となる実数 x を具体的に1つ求めよ。
( x が有理数のときは必ず lim_[n→∞] {n!*x} = 0 となることに注意)
(3) lim_[n→∞] {n!*(e/2)} は存在しないことを示せ。
(4) 全ての実数 x に対して、 lim_[n→∞] {n!*x} ≠ a となるような実数 a ∊ (0, 1) が存在することを示せ。
(5) (4)の条件を満たす実数 a ∊ (0, 1) を具体的に1つ求めよ。
(6) {n!*√2} や {n!*π} は n → ∞ のときにどのような挙動を示すか?

(5)と(6)は自作問題で、答えはわかりません(もしかしたら未解決かもしれない)
0637132人目の素数さん
垢版 |
2020/06/29(月) 19:42:08.27ID:cwExuDQs
>>636
補足
(4)は、論理記号を使って書くと
∃a ∊ (0, 1) s.t. ∀x ∊ R, lim_[n→∞] {n!*x} ≠ a
を示せという意味です。
ここで (0, 1) は R の開区間とし、 R は実数全体の成す集合とします。
0640132人目の素数さん
垢版 |
2020/06/29(月) 21:50:59.76ID:+3ocje1t
>>638
1


1737662031938094565999824459494356270619397861001172505471732865032623760224580084650943336301208543380031943621630075979872254724835986408433356854417101939662741313385571925863990067892927145547675001947961279645969066059766058736658595806001619985565113685309604009071992534506041686227703502285271246267285386268054188334701076510916419199007254159946899201122191709070235613544840470257137346516087775445798461110010594821321809566894441083157854016421880441787886298535922284673317305198107635595779448820162864939086315031011211661095716822957694703795145311052399652092453140826655185793355112915252303733164866977865323352062741492408134892018287738543530418555987093906754309603810722704323839135427021302024301866373218623310688617767802110828569845060500248953943201394358684846438433680024960899560464199640198775868455302077489943945015055881469790826298713660881217637905553645132439842440041476360402191364434103777980116087227171313236217001593357864456019476016940251078882930170581785626471754610263843434388748614065167671583732790323210962621265516202556666051857894632079443919057568868296675205530147243722453008787860917005634440791070990090033802303564619892603772739860232814440760827834068244717034998446429155877901463847580516635477753360218291710334110437969770421905196578617628042261474807555550852780628662686778424328514217905444070065811486319791485712994
0641132人目の素数さん
垢版 |
2020/06/29(月) 21:51:15.60ID:+3ocje1t
17963950579210719961422405768071335213324842709316205032078384168750091017964584060285240107161561019930505687950233196051962261970932008838279760834318101044311710769457048672103958655016388894770892065267451228938951370237422841366052736174160431593023473217066764172949768821843606479073866252864377064398085101223216558344281956767163876579889759124956035672317578122141070933058555310274598884089982879647974020264495921703064439532898207943134374576254840272047075633856749514044298135927611328433323640657533550512376900773273703275329924651465759145114579174356770593439987135755889403613364529029604049868233807295134382284730745937309910703657676103447124097631074153287120040247837143656624045055614076111832245239612708339272798262887437416818440064925049838443370805645609424314780108030016683461562597569371539974003402697903023830108053034645133078208043917492087248958344081026378788915528519967248989338592027124423914083391771884524464968645052058218151010508471258285907685355807229880747677634789376
0644132人目の素数さん
垢版 |
2020/06/29(月) 22:07:53.20ID:+3ocje1t
>>643
Wolfram先生によると
小数点以下6.002253567994547×10^3638334640023桁
らしいから、掲示板には表示無理だな。
0645132人目の素数さん
垢版 |
2020/06/30(火) 00:03:36.55ID:pO+XZCKK
>>636
>>637
(4)と(5)は誤りでした

すなわち、(4)の否定
(4') ∀a ∊ (0, 1), ∃x ∊ R s.t. lim_[n→∞] {n!*x} = a
が成立します

そのため、(4)と(5)を以下のように変更します
(4) (4')を示せ。
(5) (4')を満たす x の集合を S とするとき、 S のルベーグ測度は 0 であることを示せ。
0646132人目の素数さん
垢版 |
2020/06/30(火) 00:24:18.57ID:+kS8XN12
>>638
3^3^3^3^3 の常用対数を取ると
log_10(3^3^3^3^3) = (3^3^3^3) log_10(3)
だから,log_10(3) を 3 進数で
3^3^3 = 7,625,597,484,987 桁
まで求め,それ以降の数を 10 進数に
復元すれば最上位の桁がわかる.

スパコン持ってる人計算よろ
0647132人目の素数さん
垢版 |
2020/06/30(火) 00:27:48.66ID:+kS8XN12
参考文献
巨大数研究Wiki > テトレーション
https://googology.wikia.org/ja/wiki/%E3%83%86%E3%83%88%E3%83%AC%E3%83%BC%E3%82%B7%E3%83%A7%E3%83%B3

したがって、\(^ba\) の上位の桁は \(10^{\text{frac}(^{b - 1}a \log_{10} a)}\) となり、\(^{b - 1}a \log_{10} a\) の小数部分を計算する問題に帰着する。これは、任意の \(a\) 進数表記の \(^{b - 2}a\) を、\(^{b - 2}a\) 番目の桁から計算することと同じである。
0649132人目の素数さん
垢版 |
2020/06/30(火) 01:29:28.77ID:mvJtwX3u
スパコンがあっても さらに工夫しないと難しい予感
たとえば log_10(3) を高速で計算する方法とか
0651132人目の素数さん
垢版 |
2020/06/30(火) 04:36:50.69ID:ujUIqCoe
ある区間 に素数が1つ以上存在するかどうかの予想。

ルジャンドル予想
n^2 ~ (n+1)^2

インガムの予想
n^3 ~ (n+1)^3

これって数学の専門家たちの予想では、
真or偽 、どちらが多数派なの?
あなた個人では真or偽 どちらだと思う?
素数の存在する区間として証明されているのは
ベルトラン=チェビシェフの n ~ 2n の区間だけだよね?
0652132人目の素数さん
垢版 |
2020/06/30(火) 06:54:44.22ID:mvJtwX3u
>>651
ルジャンドル予想やベルトラン=チェビシェフの一般化にあたる
区間[kn, (k+1)n] に必ず素数が存在するという予想がある
k=1の場合は有名な解決済み問題(ベルトラン=チェビシェフ)で
k=nの場合 [n^2, n(n+1)] となるからルジャンドル予想より強い予想
そしてルジャンドル予想のほうが (n^3,(n+1)^3)の範囲に必ず存在するという予想より強いわけだから
区間[kn, (k+1)n] に必ず素数が存在するという予想がこの中では最強ということになる
0653132人目の素数さん
垢版 |
2020/06/30(火) 07:11:35.00ID:mvJtwX3u
>>651
素数の存在する区間として証明されているのは
ベルトラン=チェビシェフの n ~ 2n の区間だけだよね?

というのは誤っているとおもわれるので改めると
たとえば [kn, (k+1)n] の問題は k=1の他に k=2,3 とかで証明が与えられている
遥かに強力な結果としては [n, n+C*n/{log(n)}^2] に常に素数があるというものがある
Cは定数で実際に計算できる これはDusartが得た結果を変形すればただちにCが具体的に得られる
0654132人目の素数さん
垢版 |
2020/06/30(火) 09:11:42.22ID:ujUIqCoe
>>652
k が 1,2,3... だけでなく、
k = n/3 くらいまで使えたらいいのにな〜。


>>653
[n,n + C* n/(ln(n))^2]
= [64, 64 + C * 3.70]

となったんですが、C って nが64の時はいくらですか?
0655イナ ◆/7jUdUKiSM
垢版 |
2020/06/30(火) 10:27:03.86ID:i3dc601K
>>542
>>638
3を3^7625597484987回掛けると最上位の数がなにかわかることはわかった。
0657132人目の素数さん
垢版 |
2020/06/30(火) 11:28:45.33ID:vzuOiySp
いつものプログラム解

dec2nw <- function(num, N, digit = 4){
r=num%%N
q=num%/%N
while(q > 0 | digit > 1){
r=append(q%%N,r)
q=q%/%N
digit=digit-1
}
return(r)
}

price=c(200,200,300,300,400,400)
A=c(NA,NA, 1, 1, 1, 1)
B=c(NA,NA, 1, 1,NA,NA)
C=c(NA,NA, 1, 1,NA,NA)
D=c( 1, 1, 0, 1,NA,NA)
E=c(NA,NA,NA, 1,NA,NA)
ABCDE=rbind(A,B,C,D,E)
colnames(ABCDE)=c('いか赤','たこ赤','いくら黒','まぐろ黒','うに金','たい金')
ABCDE

i2y<- function(i){ # i -> binary -> matrix 5人 × 6ネタ
x=dec2nw(i,2,digit=17)
y=ABCDE
y['A',c(1,2)]=x[1:2]
y['B',c(1,2,5,6)]=x[3:6]
y['C',c(1,2,5,6)]=x[7:10]
y['D',c(5,6)]=x[11:12]
y['E',c(1,2,3,5,6)]=x[13:17]
y
}

fn <- function(y){
pay=apply(y,1,function(x) sum(x*price))
all(
all(apply(y,1,sum)>=3),# 5人は,それぞれ3皿以上注文した。
sum(pay)==5000,# 5人が注文した金額の合計は5,000円であった
which.max(pay)==1 & sum(y[1,]*price)==1600,# 注文した金額が最も多かったのはAで,1,600円であった。
all(y[,4]==1), # 5人とも,「まぐろ」を注文した。
all(y[1:3,3]==1), # A,B,Cは,「いくら」を注文した。
sum(y[4,1:2])==2 & sum(y[4,3:4])==1 & sum(y[4,5:6])==1, # Dは,赤皿を2皿,黒皿を1皿,金皿を1皿の合計4皿を注文した。
all(y<2) # 同じネタを2皿以上注文した者はいなかった。
)
}

# brute force
ans=NULL
N=2^17-1
dec2nw(N,2)
for(i in 1:N){
y=i2y(i)
if(fn(y)){
ans=append(ans,i)
}
}
length(ans)
0658132人目の素数さん
垢版 |
2020/06/30(火) 11:30:07.50ID:vzuOiySp
Y=NULL
for(j in ans) Y=rbind(Y,as.vector(t(i2y(j))))
all(Y[,2]==1) # Aは「たこ」を注文した。
all(sum(Y[,7:8])==2) # Bは赤皿を2皿注文した。
all(apply(Y[,13:18],1,function(x) sum(x*price))==800) #Cが注文した金額は800円であった。
all(Y[,23]==1) # Dは「うに」を注文した。
all(Y[,29:30]==1) # Eは金皿を1皿注文した。

実行すると

> for(j in ans) Y=rbind(Y,as.vector(t(i2y(j))))
> all(Y[,2]==1) # Aは「たこ」を注文した。
[1] FALSE
> all(sum(Y[,7:8])==2) # Bは赤皿を2皿注文した。
[1] FALSE
> all(apply(Y[,13:18],1,function(x) sum(x*price))==800) #Cが注文した金額は800円であった。
[1] TRUE
> all(Y[,23]==1) # Dは「うに」を注文した。
[1] FALSE
> all(Y[,29:30]==1) # Eは金皿を1皿注文した。
[1] FALSE
>

理詰めの解答は賢者にお願いします。
0659132人目の素数さん
垢版 |
2020/06/30(火) 17:39:52.77ID:pO+XZCKK
実数 x > 0 に対し、 [ ] をガウス記号とし、 {x} = x - [x] を x の小数部分とする。
∀x ∊ (0, 1) に対し、無限級数 Σ[n=0,∞] max(1, [nx])/n! は絶対収束するので、
関数 f : (0, 1) → R を
f(x) := Σ[n=0,∞] max(1, [nx])/n!
によって定めることができる。

(1) f の逆関数 f^(-1) : Im(f) → (0, 1) が存在することを示せ。
(2) f^(-1)(x) = lim_[n→∞] {n!*x} が成り立つことを示せ。
(3) f(1/2) を求めよ。
(4) Im(f) ⊂ (e, 3) - Q が成り立つことを示せ。
(5) |Im(f)| = |R| だが、 Im(f) のルベーグ測度は 0 であることを示せ。
(6) f の連続性を調べよ。
(7) f の微分可能性を調べよ。

(6)と(7)は個人的に未解決です
0660 【大吉】
垢版 |
2020/07/01(水) 19:18:00.37ID:2hRevtG6
>>655
>>656
3.Cはまぐろ300円、いくら300円、赤皿200円1枚を頼んだので800円。
Aはまぐろ300円、いくら300円、金皿400円2枚、赤皿200円1枚を頼んだので、赤皿の選び方が2通りある。
Bはまぐろ300円、いくら300円、赤皿200円1枚を頼んだので、赤皿の選び方が2通りある。
Cはまぐろ300円、いくら300円、赤皿200円1枚を頼んだので、赤皿の選び方が2通りある。
Dはまぐろ300円、いか200円、たこ200円、金皿1枚を頼んだので、金皿の選び方が2通りある。
Eはまぐろ300円、いか200円、たこ200円を選んだ。
2×2×2×2=16(通り)
0662132人目の素数さん
垢版 |
2020/07/02(木) 21:17:00.91ID:PPmvU0cQ
>>659
(6) x が有理数のときは不連続で、 x が無理数のときは連続

したがって、(7)の部分的な解「少なくとも x が有理数のときは微分不可能」が得られる。

( x が有理数のときに不連続であることの証明)
x = a/b ∊ (0, 1) ( a, b は正の整数)とすると、 ∀y ∊ (x/2, x) に対して |f(x) - f(y)| ≧ 1/(2b)! となることを示す。
∀y ∊ (x/2, x) に対し、 2by > bx = a ≧ 1 であるので、 max(1, [2by]) = [2by] である。
また、 2by < 2bx = 2a = [2bx] より [2by] ≦ [2bx] - 1 である。
これより、
|f(x) - f(y)| ≧ |max(1, [2bx]) - max(1, [2by])| / (2b)! = ([2bx] - [2by])/(2b)! ≧ 1/(2b)!
が成り立ち、ゆえに関数 f は x が有理数のときに不連続である。

( x が無理数のときに連続であることの証明)
方針は、 p_n(x) := max(1, [nx]) と置くとき、 x と y が十分近ければ途中の n までは p_n(x) = p_n(y) で
残りの部分は 0 に収束することを利用することである。

∀x ∊ (0, 1) - Q および ∀ε > 0 をとる。
e = Σ[n=0,∞] 1/n! が収束することから、ある整数 N > 1 が存在して、
Σ[n=N+1,∞] 1/(n-1)! < ε/2
が成り立つ。
n = 1, 2, 3, … に対し、 d_n := min(nx - [nx], [nx] + 1 - nx) と置くと、 x が無理数であることから d_n > 0 である。
ここで |x - y| < d_n/n ならば [nx] < ny < [nx] + 1 より [ny] = [nx] が成り立つことに注意して、
上で得た N を使って実数 δ > 0 を
δ := min{d_1/1, d_2/2, … , d_N/N}
と定めると、 |x - y| < δ ⇒ |f(x) - f(y)| < ε が成り立つことを示す。
|x - y| < δ ならば、上の注意より、 n ≦ N のとき p_n(x) = p_n(y) が成り立つ。
したがって、 |p_n(x) - p_n(y)| < n+1 (∀n > 1) が成り立つことから、
|f(x) - f(y)| = Σ[n=N+1,∞] |p_n(x) - p_n(y)| / n!
< Σ[n=N+1,∞] (n+1)/n! = Σ[n=N+1,∞] 1/(n-1)! + Σ[n=N+1,∞] 1/n!
< 2 * Σ[n=N+1,∞] 1/(n-1)!
< ε
が成り立ち、ゆえに関数 f は x が無理数のときに連続である。

(7)について、 x が無理数のときの微分可能性はどうなるんでしょうか?
誰かわかる人いますか?
0663132人目の素数さん
垢版 |
2020/07/03(金) 00:07:20.35ID:wLpvQFy3
(1) lim_[n→∞] {n!f(a)}=a (0<a<1) が成り立つことが言える(直接計算するだけ)。
特に、fは単射である。よって、(1)が成り立つ。

(2) lim_[n→∞] {n!f(a)}=a (0<a<1) において a=f^{−1}(x) (x∈Im(f)) と置けばよい。

(3) 直接計算するだけなのでパス。

(4) 0<x<1とする。max(1, [nx])≧1 (n≧0)であり、あるnに対して max(1, [nx])>1 なので、
Σ[n=0,∞] max(1, [nx])/n! > Σ[n=0,∞] 1/n! = e である。次に、0<x<1なので [nx]≦n−1 であり、

Σ[n=0,∞] max(1, [nx])/n!≦Σ[n=0,∞] max(1, n−1)/n!
=2+Σ[n=2〜∞] (n−1)/n!=2+Σ[n=2〜∞](1/(n−1)!−1/n!)=3

となる。等号が成り立つとすると、任意のn≧0に対して max(1, [nx])= max(1, n−1) でなければならないので、
nが十分大きければ常に [nx]=n−1 であり、両辺を n で割って n→∞ とすれば x=1 となる。これは矛盾。
よって、等号は成り立たない。よって Im(f)⊂(e,3) となる。もし q/p∈Im(f) なる有理数があるとすると、
q/p=f(a) なる 0<a<1 が取れて lim_[n→∞] {n!f(a)}=a すなわち lim_[n→∞] {n!(q/p)}=a となるので、
a=0 となるが、これは矛盾。よって、Im(f)⊂(e,3)−Q である。

(5) f:(0,1) → Im(f) は全単射なので、Im(f)はRと同じ濃度である。
次に、A={a∈R|lim[n→∞] e^{2πia} が存在する } と置くと、Aは(R,+,0)の部分群になることが分かる。
また、Aはボレル可測であることが言える。steinhaus theorem により、Aはゼロ集合である。
また、Im(f) ⊂ A である。よって、Im(f) はルベーグゼロ集合である。

(6) ルベーグの収束定理の数列版により、
0<x<1 が無理数のときは lim[y→x]f(y)=f(x) が成り立ち、
0<x<1 が有理数のときは lim[y↓x]f(y)=f(x) が成り立つ。
また、0<x<1 が有理数のときは、lim[y↑x]f(y) の値が(ルベーグの収束定理の数列版により)直接計算できて、
その形を見る限りは lim[y↑x]f(y) ≠ f(x) が成り立つものと思われる(面倒くさいので検証はパス)。

(7) わからん。
0664132人目の素数さん
垢版 |
2020/07/03(金) 00:13:05.97ID:wLpvQFy3
訂正

× 次に、A={a∈R|lim[n→∞] e^{2πia} が存在する } と置くと、Aは(R,+,0)の部分群になることが分かる。

〇 次に、A={a∈R|lim[n→∞] e^{2πin!a} が存在する } と置くと、Aは(R,+,0)の部分群になることが分かる。


× また、Aはボレル可測であることが言える。steinhaus theorem により、Aはゼロ集合である。

〇 また、Aはボレル可測であることが言える。steinhaus theorem により、Aがゼロ集合でないなら
  Aは開区間を含むことになるが、lim[n→∞] e^{2πin!a} が存在しない a∈R は R 内に稠密に
  存在することが言えるので、矛盾し、Aはゼロ集合になるしかない。
0665132人目の素数さん
垢版 |
2020/07/03(金) 09:43:11.00ID:wLpvQFy3
(7) 0<x≦y<1 のとき max(1, [nx]) ≦ max(1, [ny]) なので、f(x) ≦ f(y) である。
すなわち、f:(0,1)→R は広義単調増加である。
一般に、広義単調増加な g:(0,1)→R は a.e.x∈(0,1) で微分可能であるという定理があるので、
f もまた a.e.x∈(0,1) で微分可能である。

f '(x) の具体的な値は分からん。たぶんこれはカントールの悪魔の階段みたいになってて、
f '(x)=0 a.e.x が成り立つと予想。
0666132人目の素数さん
垢版 |
2020/07/03(金) 11:14:12.92ID:7j7hCdMM
7の倍数を簡単に見分ける方法はなんでしょう
0668イナ ◆/7jUdUKiSM
垢版 |
2020/07/03(金) 11:26:07.08ID:SuxSTpfs
>>660
>>666
7て🤮割ってみな。
割り切れたらいいよ。
そいつは7の倍数に違いない。
0670132人目の素数さん
垢版 |
2020/07/03(金) 13:04:58.11ID:l9EAO2Py
>>665
>一般に、広義単調増加な g:(0,1)→R は a.e.x∈(0,1) で微分可能であるという定理があるので、
>f もまた a.e.x∈(0,1) で微分可能である。

なるほど、ルベーグの定理ですね
さらに
∫[0,1] f'(x) dx ≦ 3-e
もわかるわけですか
その例外集合に無理数が含まれるかどうかは気になるところですね

ところで、 f(0) := e, f(1) := 3 として関数 f の定義域を閉区間 [0, 1] まで拡張すると、
f は単調増加なので [0, 1] 上リーマン積分可能ですが、
S = ∫[0,1] f(x) dx
はいくらになるんでしょうか?明らかに
e < S < 3
であることはわかりますが
例えば、 S > f(1/2) か S < f(1/2) か、それとも S = f(1/2) か?
0671132人目の素数さん
垢版 |
2020/07/03(金) 13:09:07.52ID:+Y/uxVJK
そもそも像の測度が0かつ定数でない時点で連続なわけないのでは?
0672132人目の素数さん
垢版 |
2020/07/03(金) 13:21:53.43ID:l9EAO2Py
>>671
面白いことに、 Im(f) ⊂ (e, 3) - Q なので f(x) は (e, 3) 上の全ての有理数をすり抜けるが、
x が無理数のときは連続になる
0673132人目の素数さん
垢版 |
2020/07/03(金) 13:26:58.75ID:lWgbvUw5
数学掲示板群 ttp://x0000.net/forum.aspx?id=1

学術の巨大掲示板群 - アルファ・ラボ ttp://x0000.net
数学 物理学 化学 生物学 天文学 地理地学
IT 電子 工学 言語学 国語 方言 など

PS 連続と離散を統一した!
ttp://x0000.net/topic.aspx?id=3709-0
微分幾何学入門
ttp://x0000.net/topic.aspx?id=3694-0
0674132人目の素数さん
垢版 |
2020/07/03(金) 13:50:09.40ID:hTY3FFmd
>>666
元の数の1の位を取り去った数(10で割って余りを切り捨てたもの)と、1の位の数の2倍との差が、7の倍数であるか判定する
0675132人目の素数さん
垢版 |
2020/07/03(金) 15:51:14.65ID:jM2SUMu3
>>669
あんまり使えないてゆか1001使った後で7と98使うといい
abc=2a+3b+c
0676イナ ◆/7jUdUKiSM
垢版 |
2020/07/03(金) 15:54:28.83ID:/0Vwgdcs
>>668
ニベヤはある。希乃屋がない。
てかシミはそんなんじゃ消えないんだろ?
0678132人目の素数さん
垢版 |
2020/07/04(土) 08:35:33.57ID:LhRVANFU
神様がおっしゃいました。

これから1週間、社会実験として世界の法則を書き換える。

「実数から全ての無理数を取り除き
有理数だけが存在する世界A」 にしようか。
あるいは、その反対の世界Bにしようか。
選び給え。


あなたはどちらを選ぶか?
その理由を述べて論ぜよ。
0681132人目の素数さん
垢版 |
2020/07/05(日) 08:05:22.74ID:4xu+FlwC
>>678
無理数だけだと無理数どおしの演算も不可能になる
√2 * √2=不能かと思ったがそもそも、2が無理数じゃないから使えないな。
0682132人目の素数さん
垢版 |
2020/07/05(日) 08:21:42.29ID:W9kIejTS
人「そんな世界が作れるならどっちでもいいがあなたに作れるのか?」
神「……」
0683132人目の素数さん
垢版 |
2020/07/05(日) 08:44:15.03ID:3b3HlX6o
>>670
∫[0,1] f'(x) dx がどんな値になるかは不明。f ' がカントールの悪魔の階段みたいになってて
f '(x)=0 a.e.x が成り立つのであれば、∫[0,1] f'(x) dx = 0 になる。
たぶん実際にこうなっていると予想。

これとは別に、S = ∫[0,1] f(x) dx なら自明に計算可能。

f(x)=Σ[n=0,∞] max(1, [nx])/n! (0<x<1)

の右辺は非負値可測関数の無限和なので、
ルベーグ積分とΣの順序交換が無制限に可能で、値が+∞になる可能性を込めて

∫[0,1] f(x) dx = Σ[n=0,∞] ∫[0,1] max(1, [nx])/n! dx

が無条件に成り立つ(厳密な証明は単調収束定理による)。
右辺は簡単に計算可能なので、あとはやるだけ。
0684132人目の素数さん
垢版 |
2020/07/05(日) 20:06:55.33ID:B5RPIpUI
>>683
確かにカントールの悪魔の階段に似ている感じがしますね
微分不可能な例外集合 ( ⊃ Q ) について調べるのは難しいでしょうか?

>ルベーグ積分とΣの順序交換が無制限に可能で、値が+∞になる可能性を込めて
>∫[0,1] f(x) dx = Σ[n=0,∞] ∫[0,1] max(1, [nx])/n! dx
>が無条件に成り立つ(厳密な証明は単調収束定理による)。

なるほど、言われてみれば確かにそうですね

>右辺は簡単に計算可能なので、あとはやるだけ。

やってみました

【結果】
S := ∫[0,1] f(x) dx = 3/2 + Ei(1) - γ = 2.817902…
ここで Ei(x) は指数積分関数であり、 γ はオイラーの定数である。
f(1/2) = 2 + (e + e^(-1))/4 = 2.7715403… であるので、 S > f(1/2) が成り立つ。

(証明的なもの)
I_n := ∫[0,1] max(1, [nx]) dx と置く。 I_0 = 1 は明らか。 n > 0 のとき、積分区間を n 等分することで、
I_n = 1/n + (n-1)/2
となることがわかる。これより、
∫[0,1] f(x) dx = Σ[n=0,∞] I_n/n!
= 3/2 + Σ[n=1,∞] 1/(n(n!))
が成り立つ。あとは右辺の無限和に指数積分関数 Ei(x) の公式
Ei(x) = γ + log(x) + Σ[n=1,∞] x^n/(n(n!))
(x > 0) を適用すれば良い。

【参考】
Ei(1) - γ
https://www.wolframalpha.com/input/?i=Ei%281%29+-+%CE%B3
0685132人目の素数さん
垢版 |
2020/07/05(日) 22:46:48.03ID:zaLNiyGh
f(x) =1/ 2x(x^2 −1) (x < 0)
x(e^x − 3/ 2) (x ≥ 0)のとき


(1) f′(0)を求めよ.
(2) f′(x)を求めよ.
(3) f ∈ C^n(R)としたとき, 最大のn ∈N∪{0}を求めよ
ただし、以上のうちで定まらないものがあればその理由を述べよ
0686132人目の素数さん
垢版 |
2020/07/05(日) 23:17:02.47ID:+qWDHqGw
x(e^x-3/2) =
-x/2 + x^2 + x^3/2 + x^4/6 + x^5/24 + x^6/120 + O(x^7)

1/2x(x^2-1) =
-x/2 + x^3/2

よりC^1級
0687132人目の素数さん
垢版 |
2020/07/05(日) 23:19:39.38ID:kdMLUFmb
広義重積分の問題でおもしろくってためになるのを教えてください
0688132人目の素数さん
垢版 |
2020/07/06(月) 01:15:26.67ID:s8I58AGk
>>679
多い日も 安心!

>>680
ピタゴラスの三平方の定理が
実質的にピタゴラス数の整数倍でしか使えなくなるぞ。

・25^2 = 24^2 + 7^2
・29^2 = 20^2 + 21^2 とか…

>>681
Good piont. そこは重要だぞ。

>>682
神と和解せよ
0689132人目の素数さん
垢版 |
2020/07/06(月) 02:05:40.59ID:PlQuxFm7
>>688
完備化してもう一度作れば結局同じなんで、どっちでも良い。
有理数が先にある方が分かりやすいとは思うが。
0690132人目の素数さん
垢版 |
2020/07/06(月) 02:55:34.04ID:s8I58AGk
無理数が禁止になると、
計算機の浮動小数点が何割か禁止になるな。

計算機の誤差を気にしなくていいから幸せ!
0691132人目の素数さん
垢版 |
2020/07/06(月) 20:06:27.09ID:2dMwYt1i
自然数N={1,2,3,・・・}に対して
通常とは異なる加法演算+':N×N→Nを入れよ

ただし加法は可換かつ結合的で通常の乗法に対し分配的であるものとする
0692132人目の素数さん
垢版 |
2020/07/06(月) 20:21:21.73ID:2dMwYt1i
自然数N={1,2,3,・・・}に対して
通常とは異なる乗法演算×':N×N→Nを入れよ

ただし乗法は可換かつ結合的で通常の加法に対し分配的であるものとする
0693132人目の素数さん
垢版 |
2020/07/06(月) 21:08:29.59ID:d1BDPeOt
>>691
u(2^e3^fm) = 2^f3^em for (2,m)=(3,m)=1と定め、vをその逆写像とする。
x+'y:=v((u(x)+v(y))
で定めれば良い

>>692
m×'n := 2mn
で定めれば良い。
0694132人目の素数さん
垢版 |
2020/07/06(月) 22:03:16.00ID:2dMwYt1i
>>693
正解です!

ちなみにその(+',×)構造は通常の(+,×)構造と同型ですが同型でないものも存在しています

さて整数Zにおいて環構造としての(+,×')は通常のものと同型なものしか存在しないことがわかりますが、(+',×)は通常のものと同型でないものは存在するでしょうか?
0696132人目の素数さん
垢版 |
2020/07/07(火) 07:12:00.28ID:4zsd9ZbG
>>695
正解です!

是非Z版もトライしてみてほしいです
Zに通常と非同型な加法があるか気になってるんですがわかりません
0698132人目の素数さん
垢版 |
2020/07/07(火) 10:08:58.49ID:fH2eRR/k
スレチすみません
昨夜芸スポに貼ってあった数学(?)の問題なのですが、どうしても答えが解りません。
ヒントらしきものは良く観察してと皆が書いてありました。
私は43だと思うのですが合ってますでしょうか?
どうか皆様の明晰な頭脳で正しい答えを教えて下さいm(__)m
因みに沢山の人が色んな答えを出して悩んでました。

//i.imgur.com/9CyCoiC.jpg
0700132人目の素数さん
垢版 |
2020/07/07(火) 11:36:38.71ID:jKy5QVgS
>>697
ヒカカン
0703132人目の素数さん
垢版 |
2020/07/08(水) 09:06:34.03ID:xW7mNK//
あとは、一意分解整域 Z[√-2] の各素元を Z の素元と適当に一対一対応させて、
加法をZ[√-2]からの引き戻しにより定める、とかかな

具体的にはこう
Z[√-2] の素元全体からなる集合をP'、Zの素元全体からなる集合をPとおく。
(ただし同伴なものは含めないとする)
PからP'への全単射をfとおく。
更に、f(-1)=-1 かつ f が完全乗法的になるように、fをZ全体に拡張する。
Z上の加法+'を次のように定めれば良い:
a+b = f^(-1)(f(a)+f(b))
0704132人目の素数さん
垢版 |
2020/07/08(水) 19:03:42.77ID:XIC4TO/7
なるほど
Z[√-1]やZ[√-3]は±1以外の単元が邪魔するけどZ[√-2]とかZ[(1+√-7)/2]の乗法構造は完全にZのものと一致してるのか
0705132人目の素数さん
垢版 |
2020/07/08(水) 20:01:18.04ID:XIC4TO/7
だれか実際にZ[√-2]のノルム順で(同ノルムのは適当に並べて)対応つけてZの非標準的な足し算の九九表作ってほしい

1+1=-4
1+2=5
1+3=16
1+4=-1
1+5=14
0707132人目の素数さん
垢版 |
2020/07/08(水) 23:43:45.65ID:yAchz5v9
>>703
> f が完全乗法的になるように
f(ab)=f(a)f(b)?
f(0)=0, f(1)=1, f(-1)=-1?
k(a+'b)=f^(-1)(f(k))f^(-1)(f(a)+f(b))=f^(-1)(f(k)(f(a)+f(b))=f^(-1)(f(k)f(a)+f(k)f(b))=f^(-1)(f(ka)+f(kb))=ka+'kb
0708132人目の素数さん
垢版 |
2020/07/09(木) 01:07:07.64ID:kTqZ4LdG
そうでしょ
f^(-1)うんぬんで書くと複雑だけど
要はZもZ[√-2]も乗法構造だけ見ればそれぞれ
-1,0,1とp1,p2,p3…(p1=2, p2=3, p3=5…)
-1,0,1とp'1,p'2,p'3…(p1=√-2, p2=1-√-2, p3=1+√-2…)
(素元pi,p'iは自由生成部分)となっていて同型だから
Zでのpiを内実p'iと思って計算すればよくて

例えば
"6"+"8" = "2"×("3"+"2"×"2")
= (√-2)×((1-√-2)+(√-2)×(√-2)) = (√-2)×(-1-√-2)
= "2"×"(-5)" = "-10"
0709132人目の素数さん
垢版 |
2020/07/09(木) 04:03:23.05ID:0Axmwauv
>>706
その等号は記号の濫用ぽくなるが大丈夫なのか
fを使ったほうが紛れない
記号の濫用は深くなるほど論理の誤謬をまねく
今回は足し算の表をつくるだけだから混乱しにくいだろうが
いろんな対象物や構造と組み合わせるとき 混乱しやすくなる
混乱しやすくなるだけならまだしも最悪の場合は矛盾が導かれる
0712132人目の素数さん
垢版 |
2020/07/10(金) 17:15:47.50ID:0+Yso+8+
単元が1と-1のみであるような一意分解整域なら良いって感じなのかね

そしたら標数が0でないものなら F_3[x] とかもいけるか
0713132人目の素数さん
垢版 |
2020/07/10(金) 17:28:47.69ID:sXjd7Mgj
>>712
Zからの全単射が必要では?
0714132人目の素数さん
垢版 |
2020/07/10(金) 21:34:22.88ID:oNEgi4ga
それにしても正標数の場合はn=3のときだけ可能性排除できないのモヤモヤするな
もしかしたら存在するのか
0720132人目の素数さん
垢版 |
2020/07/10(金) 22:07:40.93ID:oNEgi4ga
>>719
Z'をZと同じ乗法と別の加法「+'」(減法も「-'」と書くことにする)を持つ環とする
環準同型Z→Z'(n→(1+'1+'…+'1)(nコ))を考える
Z'が整域なので核は0(単射)か、ある素数がありpZとなる
p≧5の場合、Z'がZ/pZを含み、よって1の(p-1)原始根を含み矛盾
p=2の場合、1+'1=0 よって1=-'1
(1+'(-1))^2=1+'1+'(-1)×(1+'1)=0
よって1+'(-1)=0(冪ゼロ元の唯一性)
よって-'1=-1(加法逆元の一意性)
ところが1=-1ではないので矛盾
0721132人目の素数さん
垢版 |
2020/07/10(金) 22:35:33.88ID:oNEgi4ga
あ、p=2のときは原始2乗根である-1の行き場がなくなるから矛盾、でいいか
0722132人目の素数さん
垢版 |
2020/07/10(金) 22:43:04.12ID:9F9BYvXY
>>720
それにそもそもf:Z^→Fp[x]^ (^は乗法モノイド)は同型である必要はないのでは?
要するに逆向きのgがあって
1) gf = id_Z
2) im f は加法について閉じてる
でいけるのでわ?
0724132人目の素数さん
垢版 |
2020/07/11(土) 09:25:09.36ID:LsbT4Cw+
>>713
>>703 と同じように作ればいいんじゃないかな、Zと同じ乗法構造なんだし

素元同士(あと-1同士、0同士)を適当に一対一で対応づけて
完全乗法的になるように拡張、
その上でZの+'をF_3[x]の+と同じものにすれば良い
0725132人目の素数さん
垢版 |
2020/07/11(土) 14:13:15.11ID:M6x/p+8D
たしかにそれで良いのか
てことはF_3上のn変数多項式環とかもいける感じかな
そしてそれらは全て異なる加法構造を与える?
0729132人目の素数さん
垢版 |
2020/07/12(日) 22:29:23.69ID:xQn5UNgv
>>727
>>728
ありがとうございます、正しくは6+'5=13ですね

これプログラミング強い人であれば自動化して非標準電卓みたいなの作っちゃえるんでしょうね

ところでZ[√-2]やF_3[x]といったユークリッド整域はユークリッド次数が正の元pを使って割り算も形式的にp進少数展開で表現できるんでしょうか?
つまりa÷b=q+Σr_i×p^(-i)の形です
互除法
a=qb+r
deg(r)<deg(b)としてから
rにpを何回か掛けてdeg(rp^k)>deg(b)として再び互除法
rp^k=(r_k)b+r'
のように繰り返すと形式的に上の形に表現できそうに思うのですが
0731132人目の素数さん
垢版 |
2020/07/14(火) 11:00:36.38ID:AhpQwq7+
2020^2020の各桁の和の各桁の和の各桁の和を求めよ.
0732132人目の素数さん
垢版 |
2020/07/14(火) 11:06:27.81ID:/5FQKZKF
9で割れ
0733132人目の素数さん
垢版 |
2020/07/14(火) 12:28:48.94ID:NURZti0V
>>731
4
0734132人目の素数さん
垢版 |
2020/07/14(火) 12:50:40.46ID:AhpQwq7+
>>732
>>733
早い
正解
0735132人目の素数さん
垢版 |
2020/07/14(火) 12:59:09.20ID:NURZti0V
>>734
よく言われます ( ^〜^)

>>731
各位の和、数字和、digit sum って奴だろ。

そういえば、こういうのって
中学・高校の数学では余りやらなかったな。
分野としては
数学 I,II, III, A,B,C のどこに入るんだろうか?
旧帝大の二次試験で出そうな問題だよな。
0736132人目の素数さん
垢版 |
2020/07/14(火) 13:34:34.61ID:AhpQwq7+
各桁の和の各桁の和、なら
「4,13,22,31,40」の5つのどれかに絞ることは出来るけどそこから特定する方法ってないかな?
0737132人目の素数さん
垢版 |
2020/07/14(火) 13:51:40.41ID:/5FQKZKF
>>735
>中学・高校の数学では余りやらなかったな。
放課後に女子が盛んにやってなかったか?
0738733
垢版 |
2020/07/14(火) 15:16:55.86ID:NURZti0V
>>734 >>736-737
「 え、もう解いたの?
社長さん、頭良いですね!!」

って俺に言って ( ^〜^)
0739132人目の素数さん
垢版 |
2020/07/14(火) 18:21:45.26ID:5+mhCYXX
下の1と2の会話読み解ける方いらっしゃいませんか?

1→30A6 30A2 30BB 30C1 30B2 30EA 30E1 306F 305B 3048 305A 306C 3046 3080 3072 3044 308C FF1F 000A

2→0056 006F 0062 006F 0074 0078 0066 0073 0062 0063 006D 0066 0020 0067 0070 0073 0020 006F 0070 0078
0740中学生
垢版 |
2020/07/14(火) 21:38:20.57ID:Gs3vD2vL
袋の中に1.2.3.4.5の数字の付いた玉が五個持っていた。
この袋の中から同時に3個の球を取り出すとき何通りあるか?
0742132人目の素数さん
垢版 |
2020/07/14(火) 22:25:58.07ID:83qKvA7Q
【大セレブ】秒速で一億円稼ぐ戸田英志が価値観について語った結果...
https://www.youtube.com/watch?v=2rjYCJe_2mg
【人生勝ち組】何もしなくてもお金が稼げる簡単な仕組み大公開!!
https://www.youtube.com/watch?v=mVK89gFM1_M
【ご報告】福沢諭吉に愛されすぎて困ってます
https://www.youtube.com/watch?v=db9o-iM8nk4
【行動者こそ成功者】お前らアンチなんかしてないで行動しろ
https://www.youtube.com/watch?v=IjX2r-uv3-8
【秒速で1億円】誰でも簡単に1億円稼ぐ神マインド大公開!
https://www.youtube.com/watch?v=tL4C_OtfrUw
【超簡単】SNSを使いこなせばいくらでも稼げるって知ってたか?
https://www.youtube.com/watch?v=F1pDLMTXikY
【失敗者の共通点】貴方も当てはまる?意外としているあれをやめないと一生成功出来ない事実とは...
https://www.youtube.com/watch?v=DA8TZrRsuho
【成功者の共通点】実際に一円も稼げていなかった僕が真似して稼げるようになった成功者の共通点大公開!!!
https://www.youtube.com/watch?v=XRwZB348Hdw
0744イナ ◆/7jUdUKiSM
垢版 |
2020/07/14(火) 23:08:27.72ID:snn++hGJ
>>743訂正。
>>740
5C3=5!/(5-3)!3!=5×4/2=10(通り)
確認する。
(1,2,3),(1,2,4),(1,2,5),(1,3,4),(1,3,5),(1,4,5),(2,3,4),(2,3,5),(2,4,5),(3,4,5)
∴示された。
0745中学生
垢版 |
2020/07/14(火) 23:15:50.90ID:Gs3vD2vL
>>744
ありがとうございます
0747132人目の素数さん
垢版 |
2020/07/15(水) 04:56:08.81ID:+VNqdAmi
>>746
下2020桁の「0」を省くため、N = 202^2020 で考えると
4657桁になる。

下1〜1000  4423
1001〜2000  4491
2001〜3000  4439
3001〜4000  4551
4001〜4657  2872
------------------
  計   20776
0748132人目の素数さん
垢版 |
2020/07/15(水) 05:05:46.34ID:pnrlHIku
>>746
そうなった
figsum x = sum $ map (read.(:"")) $ show x
main = do
print $ figsum $ 2020^2020
print $ figsum $ figsum $ 2020^2020
print $ figsum $ figsum $ figsum $ 2020^2020

20776
22
4
0749132人目の素数さん
垢版 |
2020/07/17(金) 05:57:54.49ID:l8XG6+rW
100個まで玉が入る袋に通し番号の書いてある玉が入っている。
袋を持ち上げたら袋が破れて玉が転んで全部池に沈んでしまった。
その過程でみえた最大の番号は60であった。
何個観察したかは不明である。
袋に入っていた玉の数の期待値はいくらか?
0750132人目の素数さん
垢版 |
2020/07/17(金) 06:02:39.82ID:l8XG6+rW
100個まで玉が入る袋に通し番号の書いてある玉が入っている。
手を入れて5個取り出したら、番号は11,36,45,49,60であった。
袋に入っていたいる玉の数の期待値とその95%信頼区間を求めよ。
0751132人目の素数さん
垢版 |
2020/07/17(金) 09:12:56.36ID:7KjVzawt
一度に転がって行ったとしても、
5個ぐらいは番号を観察できただろうなぁ。
0752132人目の素数さん
垢版 |
2020/07/17(金) 09:49:55.03ID:7KjVzawt
>>747
数字の分布は
桁, 和, 0, 1, 2, 3, 4, 5, 6, 7, 8, 9, 計,
-------------------------------------------------------------------------------------
下1〜1000, 4423, 106, 111, 90, 111, 94, 83, 117, 99, 88, 101, 1000,
1001〜2000, 4491, 103, 107, 103, 82, 91, 104, 108, 109, 100, 93, 1000,
2001〜3000, 4439, 108, 83, 110, 109, 97, 100, 116, 83, 102, 92, 1000,
3001〜4000, 4551, 103, 91, 81, 115, 107, 105, 99, 91, 103, 105, 1000,
4001〜4657, 2872, 75, 72, 64, 72, 58, 54, 71, 67, 57, 67, 657,
--------------------------------------------------------------------------------------
合計, A=20776, 495, 464, 448, 489, 447, 446, 511, 449, 450, 458, 4657,
0754イナ ◆/7jUdUKiSM
垢版 |
2020/07/17(金) 18:19:18.28ID:++P9BJNj
>>744
>>750
11の前に10個の玉がある。
11から36までのあいだに35-11=24(個)の玉がある。
36から45までのあいだに44-36=8(個)の玉がある。
45から49までのあいだに48-45=3(個)の玉がある。
49から60までのあいだに59-49=10(個)の玉がある。
連続する球の数の平均は、
(10+24+8+3+10)/5=55/5=11(個)
球の数の期待値は60+11=71(個)
これでいいのかな?
0755132人目の素数さん
垢版 |
2020/07/17(金) 18:39:48.31ID:8Lei1qDK
>>754
高校数学の範囲ならそれで正解

他スレで暴れてる出題者なので
極力スルーでお願いします
0757132人目の素数さん
垢版 |
2020/07/17(金) 20:13:19.69ID:l8XG6+rW
>>756
嘘書いた。(0+0+0+0+59-4)/5=11だな。
つまり、最大番号60と5個取り出したという情報だけで計算できるな。
0758イナ ◆/7jUdUKiSM
垢版 |
2020/07/17(金) 21:06:28.47ID:++P9BJNj
>>754
>>749
60個から100個まで球の数に可能性があるとすると、
100個に近ければ近いほど袋は破れやすく、
60個に近ければ近いほど60と書かれた球を見やすい。
逆に100個に比べ60個だと破裂点にかかる重さは6割。
60個で60と書かれた球を最大と思って見る可能性に比べ100個で60と書かれた球を最大と思って見る可能性は6割。
相反する事象だからあいだをとって期待値は80個。
0759132人目の素数さん
垢版 |
2020/07/17(金) 21:15:14.19ID:b/ys68m4
>>754
> 11の前に10個の玉がある。
これがもう違うんじゃないか?
あの問題文だと番号は1からだとは限らないんじゃ?
0760イナ ◆/7jUdUKiSM
垢版 |
2020/07/17(金) 21:33:32.02ID:++P9BJNj
>>758
>>759なるほどね。近くにある本の開始ページは、
8,7,5,7,5……じつに様々だ。奇数が多い。球はページとはちがうからなぁ。
1で始まってもいいんじゃないかな、わからんけど。
2である必要も3がいい理由もなく、出題者の作為がないなら、1始まりでいいと思う。
0762132人目の素数さん
垢版 |
2020/07/17(金) 22:38:50.84ID:l8XG6+rW
>>758
>750の問題で10個取り出した時に最大番号が60だったときは期待値は?と考えると観察される玉の数によって期待値が変わることがわかる。
0764132人目の素数さん
垢版 |
2020/07/20(月) 21:25:27.93ID:5DiCRFL4
Σ[i=0〜n](sin((2i-1)π/n))^2=n/2を使えば示せるけど
何かエレガントな方法があるのか
0767132人目の素数さん
垢版 |
2020/07/20(月) 23:09:49.12ID:LoyrGSpM
>>763
一定?
0770132人目の素数さん
垢版 |
2020/07/21(火) 10:40:22.79ID:BXNxkEu6
>>763
正七角形 ?
どのあたりが正なの?
0771イナ ◆/7jUdUKiSM
垢版 |
2020/07/21(火) 14:38:00.89ID:kN76GBZR
>>760
>>763
一辺4の正n角形を1つの頂点を通る直線で二等分したとき、
二等分線上に正方形を菱形状に並べると、
正方形の面積の総和は2nになるとすると、
n=4,5のとき、面積の総和は図の通り10,12であり、
正n+1角形のとき面積の総和が2(n+1)になることが示せれば数学的帰納法により、
正方形以上のすべての正多角形でそれが言える。
0772132人目の素数さん
垢版 |
2020/07/22(水) 15:14:29.48ID:U4xy9LSi
外接円の半径をRとすれば 図より
 2R sin(π/n) = 4,
和積公式より
 R・cos(2(i-1)π/n) - R・cos(2iπ/n) = 2R sin(π/n) sin((2i-1)π/n)
 = 4 sin((2i-1)π/n),
よって 求めるものは
 (1/4) Σ[i=1,n] {R・cos(2(i-1)π/n) - R・cos(2iπ/n)}^2
 = 4 Σ[i=1,n] {sin((2i-1)π/n)}^2
 = 2 Σ[i=1,n] {1 - cos(2(2i-1)π/n)}
 = 2 Σ[i=1,n] {1 - [sin(4iπ/n) - sin(4(i-1)π/n)]/2sin(2π/n)},
 = 2n,    (←周期性)
0773132人目の素数さん
垢版 |
2020/07/22(水) 16:54:24.19ID:oygEfVDW
円周率100万桁までに現れる数字の頻度

> table(pai)
pai
0 1 2 3 4 5 6 7 8 9
99959 99758 100026 100230 100230 100359 99548 99800 99985 100106
0774132人目の素数さん
垢版 |
2020/07/22(水) 19:31:59.87ID:F70Y3Sfc
表がでる確率が1/2のコインを投げて表が連続してでた回数の最大値をHとする。
Hを当てる賭けをする。(Hは表Headの頭文字w)

例;表表表裏裏表表裏裏裏裏表ならH=3

問: コインを1000回投げるときHをいくつにかけるのが最も有利か?
0775132人目の素数さん
垢版 |
2020/07/22(水) 21:37:37.76ID:M5EzV2Sa
>>774
7回
0776132人目の素数さん
垢版 |
2020/07/22(水) 21:52:32.47ID:F70Y3Sfc
>>775

100万回シミュレーションしてみた。

> table(hmax)
hmax
5 6 7 8 9 10 11 12 13 14 15 16 17 18 19 20
287 18099 121112 236483 238721 169833 101347 55523 28902 14755 7481 3659 1951 944 437 240
21 22 23 24 25 26 27 29 30
108 64 20 16 11 2 3 1 1

9回連続が最頻という結果
0777132人目の素数さん
垢版 |
2020/07/22(水) 22:22:04.84ID:M5EzV2Sa
思いつきで
ln (1000) ≒ 7 くらいかな〜
と思ったけど…外したか。
0779132人目の素数さん
垢版 |
2020/07/23(木) 12:53:08.81ID:8crzF/FX
>>774
漸化式で算出した値

> data.frame(試行N=N,連続H=unlist(y[1,]),確率P=unlist(y[2,]))
試行N 連続H 確率P
1 20 3 0.3090096
2 30 4 0.2708245
3 40 4 0.2824943
4 50 4 0.2755387
5 60 4 0.2580746
6 70 5 0.2633771
7 80 5 0.2675453
8 90 5 0.2674380
9 100 5 0.2640160
10 200 6 0.2572243
11 300 7 0.2526756
12 400 7 0.2533018
13 500 8 0.2402375
14 600 8 0.2500314
15 700 8 0.2530789
16 800 8 0.2510628
17 900 8 0.2453193
18 1000 9 0.2387912
0780132人目の素数さん
垢版 |
2020/07/23(木) 13:50:31.79ID:d8ET9cVr
>>779
どういう方程式で9が出てくるんや。
0781132人目の素数さん
垢版 |
2020/07/23(木) 15:39:22.95ID:hi0FFTaD
「表が H 回以上連続して出る」 ということが起こる確率を 1/2^(H+1) として、
これに、ポアッソン分布の 期待値でλ回 起こる様なことが、0回起こる確率は、Exp(-λ) という
結果を組み合わせると、試行 n 回で、表が最長連続 H 回出る確率は、
Exp[-n/2^(H+2)]-Exp[-n/2^(H+1)]
と出せます。これを、n=100から1000まで100単位で、Hを3から12まで変化させ、表を作ってみました。

{{0.0420065, 0.165674, [0.248222], 0.2188, 0.145944, 0.0843831, 0.0453842, 0.0235368, 0.0119856, 0.0060479},
{0.00192673, 0.0420065, 0.165674, [0.248222], 0.2188, 0.145944, 0.0843831, 0.0453842, 0.0235368, 0.0119856},
{0.000084811, 0.00912486, 0.0867574, 0.213818, [0.246798], 0.189462, 0.117694, 0.0656365, 0.0346656, 0.0178147},
{3.72664 10^-6 , 0.00192673, 0.0420065, 0.165674, [0.248222], 0.2188, 0.145944, 0.0843831, 0.0453842, 0.0235368},
{1.63738 10^-7 , 0.000404481, 0.0197111, 0.121714, 0.234773, [0.237077], 0.169697, 0.101709, 0.055704, 0.0291532},
{7.19413 10^-9 , 0.000084811, 0.00912486, 0.0867574, 0.213818, [0.246798], 0.189462, 0.117694, 0.0656365, 0.0346656},
{3.16088 10^-10 , 0.0000177786, 0.00419872, 0.0607181, 0.189888, [0.249977], 0.205693, 0.132415, 0.0751926, 0.0400755},
{1.38879 10^-11 , 3.72664 10^-6 , 0.00192673, 0.0420065, 0.165674, [0.248222], 0.2188, 0.145944, 0.0843831, 0.0453842},
{6.10194 10^-13 , 7.81148 10^-7 , 0.000883045, 0.0288454, 0.142692, [0.242815], 0.229152, 0.15835, 0.0932184, 0.0505932},
{2.681 10^-14 , 1.63738 10^-7 , 0.000404481, 0.0197111, 0.121714, 0.234773, [0.237077], 0.169697, 0.101709, 0.055704}}

まぁ、それなりの結果の様です。
0786132人目の素数さん
垢版 |
2020/07/23(木) 20:21:52.19ID:hi0FFTaD
>>785
 781 の n=1000でのH=3から12の値と、別で計算のところで計算したそれに対応するエグザクトな値を並べると、

2.681 10^-14  , 1.63738 10^-7 , 0.000404481 , 0.0197111 , 0.121714 , 0.234773 , 0.237077 , 0.169697 , 0.101709 , 0.055704
1.03858 10^-16 , 3.68449 10^-8 , 0.000272408 , 0.0179442 , 0.120638 , 0.236904 , 0.238791 , 0.170018 , 0.101436 , 0.0553710

H<6では、相対誤差は大きいけど、絶対誤差は小さい。
H=6では、相対誤差は10%ほどあるけど、絶対誤差は 0.0018未満
H>6では、相対誤差1%未満

使った式は、
 Exp[-n/2^(H+2)]-Exp[-n/2^(H+1)]
だけです。コスパ的には十分だと感じました。
0787132人目の素数さん
垢版 |
2020/07/23(木) 20:53:20.96ID:CuEzKWS6
>>786
だから厳密解と近似解の誤差評価が先やろ?
後で自分の近似解を厳密解出してくらべるんなら近似解出す意味ないやろwww
0788132人目の素数さん
垢版 |
2020/07/23(木) 20:57:21.93ID:d8ET9cVr
多項式うぜぇええええええええ!!!
0789132人目の素数さん
垢版 |
2020/07/23(木) 21:50:30.92ID:cdENLWJx
>>773
p = 0.1
m = 1000001,
 出現回数X 〜 2項分布 ≒ N(μ,σ^2)
 μ = (m+1)p -1/2 = 99999.7
 σ^2 = (m+1)p(1-p) = 90000.18
 σ = 300.0003

  99959 = μ - 0.1356665σ
  99758 = μ - 0.8056659σ
 100026 = μ + 0.0876666σ
 100230 = μ + 0.7676659σ
 100359 = μ + 1.1976655σ
  99548 = μ - 1.5056652σ
  99800 = μ - 0.6656660σ
  99985 = μ - 0.0490000σ
 100106 = μ + 0.3543330σ

計 m = 1000001
0790132人目の素数さん
垢版 |
2020/07/23(木) 22:19:01.00ID:d8ET9cVr
>>782
多項式うぜぇ!
ってワイが怒ってたって
回答者に伝えといて、ぷんすか! ( ^〜^)
0791132人目の素数さん
垢版 |
2020/07/23(木) 22:19:44.45ID:d8ET9cVr
>>781
自閉症っぽい書き込み止めろ(ワイを除く)
0793132人目の素数さん
垢版 |
2020/07/24(金) 02:45:24.33ID:gGru2ral
>>787

9回連続が最も有利で厳密解として分数表示すると
1279334345138054116703387805816574492475733319271556635225122353426525246719007709820160126958797561571107282045989946953175158323114922911077578538088124336136684673995419399768527438369423015051518883496014425392294201096683634357280521115135900842944232544396696264692655374681609184183329560302491
/5357543035931336604742125245300009052807024058527668037218751941851755255624680612465991894078479290637973364587765734125935726428461570217992288787349287401967283887412115492710537302531185570938977091076523237491790970633699383779582771973038531457285598238843271083830214915826312193418602834034688
0794132人目の素数さん
垢版 |
2020/07/24(金) 03:19:17.91ID:leZfEHQl
>>793
で?
何がおもろい?
連続二回の確率とかなら受験数学レベル、
連続三回ならそれがめんどくさくなるだけ、
連続四回以上になると手計算ではしんどいから計算機使う、
で?
何がおもろいん?
0795132人目の素数さん
垢版 |
2020/07/24(金) 09:36:13.67ID:AqvcbG6k
>>786
Excel 表計算(n=1000)の結果は以下のとおり。

H, 試行n回で 表が最長連続H回出る確率
0, 1/(2^n),
1, (F_{n+2} -1)/(2^n),
2, (T_{n+2} - F_{n+2})/(2^n)
3, 1.03858E-16,
4, 3.6844929559E-8,
5, 0.000272408353627637
6, 0.0179442231630709
7, 0.120638522508376
8, 0.236903816673717
9, 0.238791240043798
10, 0.170018079219428
11, 0.101436144036570
12, 0.0553709746874828
13, 0.0289103452540215

ただし、F_n はフィボナッチ数、T_n はトリボナッチ数
0796132人目の素数さん
垢版 |
2020/07/24(金) 10:36:23.74ID:AqvcbG6k
>>793

(1.27933434513805411670338780581657449247573331927155*10^300)
/
(5.35754303593133660474212524530000905280702405852766*10^300)

= 0.2387912400437972578896579687102992815604513373849916642

(問題)
 301桁の中に 0 が最長4個連続する確率は?
0797132人目の素数さん
垢版 |
2020/07/24(金) 10:48:15.62ID:GAZu2DVL
n回で表がk回未満の並べ方の総数をH(n,k)とすると
最後からk回の中に必ず裏が出てるはずなので
最後の裏の場所で場合分けすると
n回目が裏H(n-1,k)通り
n-1回目が裏H(n-2,k)通り
・・・
n-k+1回目が裏H(n-k,k)通り
これらは排反だから
H(n,k)=H(n-1,k)+H(n-2,k)+・・・+H(n-k,k)
0≦i<k, H(i,k)=2^i
t^n=t^(n-1)+t^(n-2)+・・・+t^(n-k)
t^k=t^(k-1)+t^(k-2)+・・・+1
t^k=(t^k-1)/(t-1)
t^(k+1)-2t^k+1=0, t≠1
(k+1)t^k-2kt^(k-1)=0
t=2k/(k+1)
(2k/(k+1))^(k+1)-2(2k/(k+1))^k+1=0
(2k)^(k+1)-2(2k)^k(k+1)+(k+1)^(k+1)=0
-2(2k)^k+(k+1)^(k+1)=0
((k+1)/2)^(k+1)=k^k=u^((k+1)k), u>1
(k+1)/2=u^k, k=u^(k+1)=u(k+1)/2
2k=u(k+1)≧2(k+1) NG
t^k=t^(k-1)+t^(k-2)+・・・+1は重根無し
t=t1,・・・,tk
H(n,k)=a1t1^n+・・・+aktk^n
0≦i<k, 2^i=a1t1^i+・・・+aktk^i
ai=Δ[ti^j; j≠i, 2^i; j=i]/Δ[ti^j]
0798132人目の素数さん
垢版 |
2020/07/24(金) 11:34:17.90ID:cJjLl+Ec
>>779 >>782
これって試行回数n の時、
log_2 (n) - 1
に収束するんかな?

n=
30 → 3.91
100 → 5.64
1000 → 8.97
100万 → 18.93

100万の時は 「19連続」 に賭ければええんか?
0799132人目の素数さん
垢版 |
2020/07/24(金) 13:38:23.11ID:iAiUGZLY
>>798
乱数発生させて1万回の試行からもとめたみた。

> fn(100)
H p
1 5 0.2599
> fn(1000)
H p
1 8 0.2411
> fn(10000)
H p
1 12 0.2485
> fn(100000)
H p
1 15 0.2434

百万回は計算が終わったら書くけど、PCがフリーズするかもしれんw
0800132人目の素数さん
垢版 |
2020/07/24(金) 14:12:20.06ID:iAiUGZLY
>>798
シミュレーションでは百万回のときは19回連続が最も起こりやすいようです。

> fn(1000000)
H p
1 19 0.2351
0801132人目の素数さん
垢版 |
2020/07/24(金) 15:12:56.68ID:AqvcbG6k
「百万遍生きたねこ」
吾輩は猫である。名前はマダナイ。
K大の中で、百万遍の交差点の近傍に居候している。
いまの総長は霊長類ばかり関心があるようで、けしからぬ。
後ry)
0803132人目の素数さん
垢版 |
2020/07/24(金) 15:43:15.04ID:cJjLl+Ec
>>800
ありがとう。
実際に収束する値がどのような式になるかは知らんが
>>798 は近似としては良い線いってるな、さすがワイ ( ^〜^)

もう計算しなくていいや、電気代もったいないよ。
0805132人目の素数さん
垢版 |
2020/07/24(金) 16:15:50.09ID:cJjLl+Ec
てすと。

  0   1   2     3    4    5    6   7   8    9
99959 99758 100026 100230 100230 100359 99548 99800 99985 100106
0806132人目の素数さん
垢版 |
2020/07/24(金) 16:22:42.51ID:iAiUGZLY
>>773
有意差なし

> chisq.test(x)

Chi-squared test for given probabilities

data: x
X-squared = 5.5137, df = 9, p-value = 0.7874

> pairwise.prop.test(x,rep(1e6,10),p.adjust.method = 'none')

Pairwise comparisons using Pairwise comparison of proportions

data: x out of rep(1e+06, 10)

1 2 3 4 5 6 7 8 9
2 0.637 - - - - - - - -
3 0.876 0.529 - - - - - - -
4 0.525 0.267 0.633 - - - - - -
5 0.525 0.267 0.633 1.000 - - - - -
6 0.347 0.157 0.434 0.763 0.763 - - - -
7 0.333 0.622 0.260 0.108 0.108 0.056 - - -
8 0.709 0.923 0.596 0.312 0.312 0.189 0.554 - -
9 0.953 0.594 0.925 0.565 0.565 0.380 0.304 0.664 -
10 0.731 0.413 0.852 0.772 0.772 0.553 0.189 0.472 0.777

P value adjustment method: none
0807132人目の素数さん
垢版 |
2020/07/24(金) 17:17:49.26ID:UD0c40NE
>>798

EXは、エグザクトな値(分母分子ともに30万桁)を小数表示したものです。
PKは、Exp[-n/2^(H+2)]-Exp[-n/2^(H+1)] を使った計算です。

H17EX 0.126428026601212362722548404568479006590
H17PK 0.1264291360

H18EX 0.236853974020507393964934630163518219675
H18PK 0.2368490990

H19EX 0.2354240033524223124640651569499966602203
H19PK 0.2354209821

H20EX 0.1671294025153194307125653945982248983094
H20PK 0.1671292309

18連が最頻
0809132人目の素数さん
垢版 |
2020/07/24(金) 18:05:51.25ID:UD0c40NE
>>808
https://rio2016.5ch.net/test/read.cgi/math/1592497360/833
の前半で書いた方法に準じています。
20連を計算するときには、21×21 と 20×20 という大きな行列を用意する必要がありますが、
100万乗の計算も、行列の数十回の掛け合わせにすることが可能なので、
メモリ的にも時間的にも、漸化式を用いるときより、効率的になります。
0810132人目の素数さん
垢版 |
2020/07/24(金) 20:40:32.09ID:iAiUGZLY
>>809
レスありがとうございます。

1-(MatrixPower[M,1001].v).v とは
1- (行列Mの1001乗 内積 v) 内積v
という意味と理解していいのでしょうか?

その理解で実行したら、
> # 1-(MatrixPower[M,1001].v).v

> M=matrix(c(
+ 1,1,1,1,1,1,1,1,1,1,0,
+ 1,0,0,0,0,0,0,0,0,0,0,
+ 0,1,0,0,0,0,0,0,0,0,0,
+ 0,0,1,0,0,0,0,0,0,0,0,
+ 0,0,0,1,0,0,0,0,0,0,0,
+ 0,0,0,0,1,0,0,0,0,0,0,
+ 0,0,0,0,0,1,0,0,0,0,0,
+ 0,0,0,0,0,0,1,0,0,0,0,
+ 0,0,0,0,0,0,0,1,0,0,0,
+ 0,0,0,0,0,0,0,0,1,0,0,
+ 0,0,0,0,0,0,0,0,0,1,2),11,11,byrow=TRUE)
> v=c(1,0,0,0,0,0,0,0,0,0,0)
> '%.%' <- function(x,y) sum(x*y)
> 1- (M%^%1001%.%v)%.%v
[1] -5.9323e+301
0811132人目の素数さん
垢版 |
2020/07/24(金) 21:24:00.40ID:UD0c40NE
MatrixPower[M,1000] で、行列 M の1000乗です。
これに、ベクトルvを作用させると、長さ1000の文字列で、「表」の連続について、
{最後が裏かつ表10連未達成,1連中かつ表10連未達成,2連中かつ表10連未達成,...,表9連中かつ表10連未達成,表10連以上を含む}
を満たす文字列の数を示すベクトルが得られます。

あの問題では、10連以上を含むもの が問われていたので、このベクトルの11番目の要素を答えればいいのですが、
もう一度行列Mを作用させると、11番目の値が、ベクトルの一番目に来るため、1000乗では無く、1001乗にして、
vとの【内積】を取ることで、「Mの1000乗 に v を作用させたときの、第11成分」を取り出しています。

>> (行列Mの1001乗 内積 v) 内積v
最初の内積はおそらく正しく無く、【行列とベクトルの積】で、後ろの【内積】は、正しいです。

> # 1-(MatrixPower[M,1001].v).v
申し訳ありません。 /2^1000 付け加え忘れていました。
1-((MatrixPower[M,1001].v).v)/2^1000
が正しい式です。
0812132人目の素数さん
垢版 |
2020/07/24(金) 21:46:40.81ID:iAiUGZLY
>>811
ありがとうございます。
漸化式と違って爆速で計算されました。

> # 1-(MatrixPower[M,1001].v).v/2^1000
> library(expm)
> M=matrix(c(
+ 1,1,1,1,1,1,1,1,1,1,0,
+ 1,0,0,0,0,0,0,0,0,0,0,
+ 0,1,0,0,0,0,0,0,0,0,0,
+ 0,0,1,0,0,0,0,0,0,0,0,
+ 0,0,0,1,0,0,0,0,0,0,0,
+ 0,0,0,0,1,0,0,0,0,0,0,
+ 0,0,0,0,0,1,0,0,0,0,0,
+ 0,0,0,0,0,0,1,0,0,0,0,
+ 0,0,0,0,0,0,0,1,0,0,0,
+ 0,0,0,0,0,0,0,0,1,0,0,
+ 0,0,0,0,0,0,0,0,0,1,2),11,11,byrow=TRUE)
> v=c(1,0,0,0,0,0,0,0,0,0,0)
> '%.%' <- function(x,y) sum(x*y)
> 1- ( (M%^%1001) %*% v ) %.% v/ (2^1000)
[1] 0.3854497524124814
0813132人目の素数さん
垢版 |
2020/07/24(金) 23:00:49.43ID:gGru2ral
>>811
私の使っているRだと
1-((MatrixPower[M,1001].v).v)/2^1000
2^1000がオーバフローしてしまうので
1-(2*(MatrixPower[M/2,1001].v).v)とすることでオーバーフローを回避できました。 

1万回だと12回連続が最多
> sapply(11:13, function(x) flip.max(10000,x)$p)
[1] 0.2082006 0.2484186 0.1939128
10万回だと15回が最多
> sapply(14:16, function(x) flip.max(100000,x)$p)
[1] 0.1701653 0.2489043 0.2165800
100万回だと18回が最多
> sapply(17:19, function(x) flip.max(1e6,x)$p)
[1] 0.126428 0.236854 0.235424
1000万回だと22回が最多
> sapply(21:23, function(x) flip.max(1e7,x)$p)
[1] 0.2114217 0.2474009 0.1912985
1億回だと25回が最多
> sapply(24:26, function(x) flip.max(1e8,x)$p)
[1] 0.1745655 0.2493603 0.2142829
という結果が瞬時に出せました。

解説どうもありがとうございました。
0814132人目の素数さん
垢版 |
2020/07/24(金) 23:14:37.26ID:UD0c40NE
nが大きくなるほど、ポアッソン分布を使った近似式 Exp[-n/2^(H+2)]-Exp[-n/2^(H+1) の
精度が良くなっているのが、個人的にはうれしい

In[37]:= N[Table[Exp[-n/2^(H+2)]-Exp[-n/2^(H+1)],{n,10000,10000},{H,11,13}],10]
Out[37]= {{0.2079842885, 0.2481372247, 0.1938339325}}
In[38]:= N[Table[Exp[-n/2^(H+2)]-Exp[-n/2^(H+1)],{n,100000,100000},{H,14,16}],10]
Out[38]= {{0.1701541230, 0.2488638895, 0.2165632963}}
In[39]:= N[Table[Exp[-n/2^(H+2)]-Exp[-n/2^(H+1)],{n,1000000,1000000},{H,17,19}],10]
Out[39]= {{0.1264291360, 0.2368490990, 0.2354209821}}
In[40]:= N[Table[Exp[-n/2^(H+2)]-Exp[-n/2^(H+1)],{n,10000000,10000000},{H,21,23}],10]
Out[40]= {{0.2114212368, 0.2474004598, 0.1912984145}}
In[41]:= N[Table[Exp[-n/2^(H+2)]-Exp[-n/2^(H+1)],{n,100000000,100000000},{H,24,26}],10]
Out[41]= {{0.1745654478, 0.2493602509, 0.2142829122}}
0815132人目の素数さん
垢版 |
2020/07/25(土) 01:37:01.16ID:MJwYl0BZ
1 歩で 1 段または 2 段のいずれかで階段を昇るとき、 1 歩で 1 段昇ることは連続しないものとする。
n 段の階段を昇る昇り方を s[n] 通りとする。
このとき、
lim_[n→∞] s[n+1] / s[n] = p が存在することを示し、
p^k ( k = 1, 2, 3, … ) は限りなく整数に近づくことを示せ。
ここで p^k が「限りなく整数に近づく」とは、
p^k に最も近い整数と p^k との距離が k → ∞ で 0 に収束することを意味する。
0818132人目の素数さん
垢版 |
2020/07/25(土) 13:15:25.84ID:1FL39oBq
手持ちの金0のギャンブラーがいるとする。
朝起きて1回コインを投げて表が出たら1万円貰える。
裏が出れば1万円を払う。手元に金がないときは借金として記録される。金利は0
1年(365日とする)間これを行って黒字であった日数を記録して精算して持ち金を0にリセットする。
これを毎年繰り返す。1年に300日以上黒字である確率は?
0820132人目の素数さん
垢版 |
2020/07/25(土) 13:34:40.33ID:JNwKp/Jv
>>818
それまた計算機使うしかないやつにしか見えへんけどホンマに計算機使わないでエレガントに解く方法持ってんの?
0823132人目の素数さん
垢版 |
2020/07/25(土) 15:00:58.82ID:teQTbjXI
f(n)=1^m+2^m+3^m+…+n^mとする
mが正の偶数であるときf(n)はnの多項式としてn(n+1)(2n+1)で割り切れることを示せ
0827132人目の素数さん
垢版 |
2020/07/25(土) 15:43:25.97ID:zrd1o9Ws
>>823
f(n)-f(n-1)=n^m
が恒等式よりmが偶数のとき
f(n)-f(n-1)=f(-n)-f(-n+1)
よって帰納的に
fn)=-f(1-n)
よってn(n+1)で割った商rはr(n)=r(1-n)である一次式
0828132人目の素数さん
垢版 |
2020/07/25(土) 15:43:31.93ID:1FL39oBq
>>820
こういう厳密解が出せるようです。すでにネタバレしてしまったがw

1-2/pi*asin(sqrt(300/365))
0830132人目の素数さん
垢版 |
2020/07/25(土) 15:48:55.91ID:g3fpMEvS
>>786 >>795 でp値(エグザクト)を見ると
H=8 と H=9 の値は近く、H=8.5 の辺りで極大らしい。

>>807 のp値(エグザクト)を見ると
H=18 と H=19 の値は近く、H=18.5 の辺りで極大らしい。


>>786 の近似は、極大の周辺では相対誤差が小さいようで
exp[-n/2^(H+2)] - exp[-n/2^(H+1)] = t(1-t) ≦ 1/4.
ここに t = exp[-n/2^(H+2)],
極大は t=1/2 のとき。
 2^(-1) = exp[-n/2^(H+2)],
2を底とする対数を2回とると
 H = log_2(n) + log_2{log_2(e)/4}
  = log_2(n) - 1.47123363

n=1000 → 8.4945
n=10^6 → 18.460
でつね >>798
0834132人目の素数さん
垢版 |
2020/07/25(土) 16:05:15.79ID:zrd1o9Ws
なんかメチャメチャやろ?
「この確率の近似解を精度は問わないから好きな事かけ」なんか?
今上がってる答えが答えなら問題になってない
0836132人目の素数さん
垢版 |
2020/07/25(土) 16:12:28.42ID:1FL39oBq
>>832
ベータ分布でも出せるよ。無理数になるけど。
シミュレーション結果と一致したし。

> pbeta(300/365,0.5,0.5,lower.tail=FALSE)
[1] 0.2773420554
0837132人目の素数さん
垢版 |
2020/07/25(土) 16:14:31.14ID:zrd1o9Ws
>>836
だからそれ近似解やろ?
>>818の問題で勝手に「近似解を好きな精度で求めよ」なんて読めるわけないやん?
0840132人目の素数さん
垢版 |
2020/07/25(土) 16:19:16.32ID:zrd1o9Ws
>>839
コイントスを365回やるのが全事象で確率がなんで無理数になんの?
どう考えても
P(300日黒字)
=N(365日黒字)/2^365
=整数/整数
にしかならんやろ?
0843132人目の素数さん
垢版 |
2020/07/25(土) 16:26:20.83ID:1FL39oBq
>>840
(1+1/n)^nはnを増やすと無理数になるだろ。
わかるひとには速攻でラムダムウォークの逆正弦法則の問題と見抜かれてしまった。
0844132人目の素数さん
垢版 |
2020/07/25(土) 16:30:36.04ID:zrd1o9Ws
>>843
ハァ?
>>818の問題分で漸近展開の主要項なんて読めるはずないやろ
アホか?
そもそもホンマに>>841の分子読めてんのか?
それ365回の実験を何回もしたらその値に収束するんじゃないぞ?
意味わかってないやろ?
アホか?
0845132人目の素数さん
垢版 |
2020/07/25(土) 16:33:12.77ID:8ct1Izaa
1年目での確率

永遠に継続したあとでランダムに選んだ1年での確率

で話がズレてる感じか
てか逆正弦のセッティングは最終的に0に返ってくるとしてるけど大丈夫なんかな?
0847132人目の素数さん
垢版 |
2020/07/25(土) 16:37:08.21ID:zrd1o9Ws
>>845
いや、
一年での確率も
何年か繰り返してその中からランダムにサンプルとっても確率なんぞ変わるわけない
>>841の確率は365=nを無限に飛ばす
その中で黒字の日数も日が300/365のまま飛ばしたときの極限
確率のイロハがわかっとらん
0848132人目の素数さん
垢版 |
2020/07/25(土) 16:38:50.68ID:zrd1o9Ws
>>846
それを計算機使わんと鮮やかに解く「面白い」方法があるん?
もうこの数レスでお前のレベルはわかった
お前ここに問題出せるレベルじゃないよ
0850132人目の素数さん
垢版 |
2020/07/25(土) 16:51:28.51ID:zrd1o9Ws
>>849
もうやめとけ
多分お前これだけハッキリお前が>>841の文章を誤読してるか指摘してやってるのにまだわかってないやろ?
365回やる試行を何回も繰り返して300回黒字の確率なんぞ何万回やっても有理数のままや
1/1に所持金がリセットされないと読んでも黒字やった日数はリセットせんとトータルで300日の意味になってそれなら確率0になる
黒字の日数をリセットすると解釈したら今度はある年は黒字日数312日、次の年は214日、‥となって結局問題文のイベントが何聞いてるかわからんようになる
結局確率の基本ができてないからお前には>>841レベルの話が理解できてない
もうやめといてくれ
出てってくれ
0851132人目の素数さん
垢版 |
2020/07/25(土) 17:17:03.58ID:1FL39oBq
>問題文のイベントが何聞いてるかわからんようになる
いや、わかっている人もいるようだよ。
0852132人目の素数さん
垢版 |
2020/07/25(土) 17:24:23.67ID:f5HIC6wu
>>851
じゃあちゃんと意味が通るように書き直してみろ
ちなみに>>841の文章は

n自然数、α∈[0,1]、
p[n]=P(黒字がpα日以上)
とおくときの
lim p[n] = ‥‥

や。
どう読んでもお前の書いた文章の意味にはならん
お前は文章がわかってないんじゃなくて自分が文章の意味がわかってないのが自分で気づけてない
確率論の教科書なんか一冊も読んだ事ないのにネットだけで理解したつもりなだけや
もう出てってくれ
お前には数学は無理や
0853132人目の素数さん
垢版 |
2020/07/25(土) 18:08:14.52ID:teQTbjXI
>>827
> よってn(n+1)で割った商rはr(n)=r(1-n)である一次式

ここの部分も変なので説明よろ
一次式にはならないので
0859132人目の素数さん
垢版 |
2020/07/25(土) 20:02:13.87ID:f5HIC6wu
>>858
n(n+1)で繰り返し破らないとダメだ
繰り返し割って
f(n) = Q(n(n+1))(pn+ q) + rn + s
f(n)=f(-n-1)により
p=2q、r=2s
まぁ
f(n)=-f(-n-1)が恒等式
⇔f(n-1/2)が奇関数
⇔f(n-1/2)=Σ[i:odd]c[i]n^i
の方がいいかな?
0860イナ ◆/7jUdUKiSM
垢版 |
2020/07/26(日) 02:45:51.91ID:IGjJdTLK
>>778
>>818コインの裏表だろ。
(1/2)×100=50(%)と言いたいところだが、
勝たねえと黒字にはならねえ。
つまりもっとも起こりがちな勝ったり負けたりは負けだよ。
{(1+2+……+150)×100}/{(1+2+……+150)2+151={151(150/2)}/(151×150+151)
=75×100/151
=7500/151
=49.66887……
∴5割弱
有り金なしでギャンブルすんじゃいいとこ5割なんじゃない?
0861イナ ◆/7jUdUKiSM
垢版 |
2020/07/26(日) 03:04:55.12ID:IGjJdTLK
>>860訂正。
>>818
{(1+2+……+66)+100}/{(1+2+……+183)+(1+2+……+182)}
={67×(66/2)×100}/{184×(184/2)+183×(183/2)}
=442200/(184^2+183^2)
=6.56618902665(%)
0862132人目の素数さん
垢版 |
2020/07/26(日) 10:15:27.16ID:FqjSGBlb
>>860
手持ちの金の期待値でなくて
1年間の黒字の日が1年で何日あったかという問題。
0864132人目の素数さん
垢版 |
2020/07/29(水) 09:28:19.47ID:JcI53Ddd
m日目に黒字(勝ち越し)の確率は
 p_m = 1/2   (mが奇数)
 p_m = 1/2 - C(2m,m)/2^{m+1}   (mが偶数)
だけど、mが偶数のときは相関があるから面倒だ
0865132人目の素数さん
垢版 |
2020/07/29(水) 09:38:24.33ID:JcI53Ddd
>>854
Faulhaber の定理(?)
 nの多項式だから n ↔ -n-1 の対称性から出す人が多いが
mについての帰納法でもできるんぢゃ?
0866132人目の素数さん
垢版 |
2020/07/29(水) 09:51:24.44ID:JcI53Ddd
>>823
mについての帰納法でやるならこうか。

m=1, m=2 のときは
 f_1(n) = 1 + 2 + ・・・・ + n = n(n+1)/2,
 f_2(n) = 1^2 + 2^2 + ・・・ + n^2 = n(n+1)(2n+1)/6,
により成立する。
ここで f_m(n) = 1^m + 2^m + ・・・・ + n^m.

m-2 以下の自然数に対して成立つとする。
二項公式で
 (k+1)^{m+2} - k^{m+2} - 1 = C(m+2,1)k^{m+1} + C(m+2,2)k^m + ・・・・ ,
 (k-1)^{m+2} - k^{m+2} - (-1)^{m+2} = -C(m+2,1)k^{m+1} + C(m+2,2)k^m - ・・・・ ,
辺々たす。
 (k+1)^{m+2} - 2k^{m+2} + (k-1)^{m+2} -1 -(-1)^m = 2C(m+2,2)k^m + 2C(m+2,4)k^{m-2} + ・・・・,
k=1 から k=n までたすと
 (n+1)^{m+2} - n^{m+2} -1 - [1+(-1)^m]n = 2C(m+2,2)f_m(n) + 2C(m+2,4)f_{m-2}(n) + ・・・・,

ところで
 x^{m+2} - y^{m+2} = (x-y) (x^{m+1} + x^m・y + ・・・・ + x・y^m + y^{m+1})
は x-y を因子にもつ。
 (左辺) = [(n+1)^{m+2} -1] - n^{m+2} - [1+(-1)^m]n
     = (n+1)^{m+2} - [n^{m+2} - (-1)^{m+2}] - [1+(-1)^m](n+1)
は n, n+1 を因数にもつ。
また mが偶数のときは
 (左辺) = [(n+1)^{m+2} - (-n)^{m+2}] - (1+2n)
より 2n+1 も因数にもつ。         (← 対称性)

帰納法の仮定より、右辺の m-2, m-4, ・・・・ に対して成立つ。
∴ m に対しても成り立つ。    (終)
0867132人目の素数さん
垢版 |
2020/07/29(水) 10:38:49.15ID:JcI53Ddd
O(0,0,0) A(a,0,0) B(0,b,0) C(0,0,c)
とおく。

〔Pythagorasの定理〕
直方体OABCDEFGの
辺の長さを OA=a, OB=b, OC=c,
体対角線の長さをdとおくと
 d = √(aa + bb + cc),


〔Faulhaberの定理〕
直交四面体OABCの各面の面積を
 傳OC = S_a, 僂OA = S_b, 僊OB = S_c,
 僊BC = S_d,
とおくと
 S_d = √{(S_a)^2 + (S_b)^2 + (S_c)^2},
0868132人目の素数さん
垢版 |
2020/07/29(水) 13:06:58.51ID:JcI53Ddd
(1)
 ↑OA =↑a, ↑OB =↑b, ↑OC =↑c,
 ↑OD =↑d (体対角線)
とおくと
 ↑d = ↑a + ↑b + ↑c,
(2)
 ↑S_a = (1/2)b×c,
 ↑S_b = (1/2)c×a,
 ↑S_c = (1/2)a×b,
 ↑S_g = -(1/2)(b×c + c×a + a×b),
より
 ↑S_a + ↑S_b + ↑S_c + ↑S_g = ↑o,
0869132人目の素数さん
垢版 |
2020/07/29(水) 17:47:35.57ID:vPI3GJy4
>>866
なるほど

>>823
ちなみに明示的には
母関数S(n,z)=Σ[m=1〜∞]f_(2m)(n)z^m/(2m)!を計算すると
S(n,z)=(2n+1)(Π[r=1〜∞](1-4n(n+1)a_r(z))-1)/2
(ただしa_r(z)=Σ[k=1〜∞](-z)^k/(2πr)^(2k)))
となるのでf_(2m)(n)がn(n+1)(2n+1)で割れることがわかる
0870132人目の素数さん
垢版 |
2020/07/29(水) 20:47:01.93ID:JcI53Ddd
>>866
二項公式で
 (k+1)^{m+1} -1 = k^{m+1} + C(m+1,1)k^m + C(m+1,2)k^{m-1} + ・・・・ ,
 (k-1)^{m+1} - (-1)^{m+1} = k^{m+1} - C(m+1,1)k^m + C(m+1,2)k^{m-1} - ・・・・ ,
辺々引いて
 (k+1)^{m+1} - (k-1)^{m+1} -1 + (-1)^{m+1}
  = 2C(m+1,1)k^m + 2C(m+1,3)k^{m-2} + 2C(m+1,5)k^{m-4} + ・・・・,

としても同じことだが・・・・
0871132人目の素数さん
垢版 |
2020/07/29(水) 23:06:38.56ID:ck1YOXiH
>>864
20までの偶数で計算してみた。

> data.frame(n,unlist(a),unlist(b))
n unlist.a. unlist.b.
1 2 1/4 0.25000
2 4 5/16 0.31250
3 6 11/32 0.34375
4 8 93/256 0.36328
5 10 193/512 0.37695
6 12 793/2048 0.38721
7 14 1619/4096 0.39526
8 16 26333/65536 0.40181
9 18 53381/131072 0.40726
10 20 215955/524288 0.41190

確かに面倒。
0872132人目の素数さん
垢版 |
2020/07/29(水) 23:48:47.61ID:0b6MtoQy
コラッツ操作について
奇数を偶数化するのに、3n+1 のかわりに
7n+1 や 11n+1 を使ったら…
同様に成立するんやろうか?
0874132人目の素数さん
垢版 |
2020/07/30(木) 08:30:11.40ID:6oIMYSQz
3より大きい奇数かけて足す1にするとヒューリスティックには確率的に増大する傾向になるよね
3のときは大丈夫だけど
0876132人目の素数さん
垢版 |
2020/07/30(木) 11:56:52.51ID:WYLd0gfx
スマホの規制が続いてるのでPCから

>>774のコイン連続投げ問題
>>830でほぼ解決してるけど、さらに精度を上げた近似式が作れる

フィボナッチ数列を k 項和に拡張した数列
F_k(n)=F_k(n-1)+F_k(n-2)+...+F_k(n-k)
F_k(0)=...=F_k(k-2)=0, F_k(k-1)=1
の一般項は、k と n が十分大きいとき
F_k(n)≒(2(1/(2^(k+1)-k)))^(n-k)
※ 2^(k+1)-k は厳密には x^(k+1)-2^(k+1)(x-1)^n=0 の最大の実数解

これを用いて、試行 n 回, 最高連続回数 H 回の確率
=(F_(H+1)(n+(H+1))-F_H(n+H))/(2^n) >>795
≒( (2(1/(2^(H+2)-(H+1))))^n - (2(1/(2^(H+1)-H)))^n )/(2^n)
=(1/(2^(H+2)-(H+1)))^n - (1/(2^(H+1)-H))^n
≒Exp[-n/{2^(H+2)-(H+1)}]-Exp[-n/2^{(H+1)-H}] >>781

nからHを概算で求めるなら>>830の H≒log_2(n)-1.47 で十分
0878132人目の素数さん
垢版 |
2020/07/31(金) 03:35:41.18ID:lUZmSg6J
Σ[m=1〜∞] (kkz)^m /(2m)!
 = Σ[m=1〜∞] (k√z)^{2m} /(2m)!
 = cosh(k√z) - 1
 = {sinh((k+1/2)√z) - sinh((k-1/2)√z)}/2sinh((√z)/2) - 1,

母関数は

S(n,z) = Σ[m=1〜∞] f_{2m}(n) z^m /(2m)!
 = Σ[m=1〜∞] (Σ[k=1〜n] k^{2m})z^m /(2m)!
 = Σ[k=1〜n] (Σ[m=1〜∞] (kkz)^m /(2m)!)
 = Σ[k=1〜n] {cosh(k√z) - 1}
 = sinh((n+1/2)√z) - sinh((√z)/2)}/{2sinh((√z)/2) - n
 = cosh(((n+1)/2・√z)sin((n/2)√z)/sinh((√z)/2) - n,

ありゃりゃ・・・・
0879132人目の素数さん
垢版 |
2020/07/31(金) 09:43:08.43ID:lUZmSg6J
>>795では
 F_0 = 0, F_1 = F_2 = 1, F_3 = 2, ・・・・
 T_0 = T_{-1} = 0, T_1 = T_2 = 1, T_3 = 2, ・・・
とおきますた。
0880132人目の素数さん
垢版 |
2020/07/31(金) 13:02:20.05ID:C5LrUt3P
ちょっと気になったので投稿。答えはわかりません

数列 {a_n}_(n=0,1,2,…) を次のように定める。
・ a_0=1
・ n≧1の時、2/3の確率で a_n=a_(n-1) となり、1/3の確率で a_n=-a_(n-1) となる

この時、確率1で
| Σ_(k=1,n) a_k | = O(n^α) (as n→∞)
が成り立つような実数αを全て求めよ
0882132人目の素数さん
垢版 |
2020/07/31(金) 14:46:20.66ID:yG2F0MGz
↑ 2chのラマヌジャン
0884132人目の素数さん
垢版 |
2020/08/02(日) 18:10:05.22ID:Vu/4mdzK
>>883
O … オーダー

計算量の変化の仕方を表す、一般に最も高次の変数。

オーダー n^α ということは、
n^α で計算量が変化して(増えて)いくという意味。
0885132人目の素数さん
垢版 |
2020/08/02(日) 18:44:47.37ID:YI8iBIPx
ランダウの記号(ビッグオー)のことでしょ?
f(n) を自然数 N 上で定義された実数値関数とするとき、
f(n) = O(n^α) (as n → ∞) の定義は
∃M ∊ N, ∃ C > 0 s.t. ∀n ∊ N, n > M ⇒ |f(n)| < C|n^α|
でしょ?
0886132人目の素数さん
垢版 |
2020/08/02(日) 22:42:44.08ID:55ujZ5fc
てす
他スレのコンプガチャの問題
ググったら答えが見つかったけどどうするかな
0887132人目の素数さん
垢版 |
2020/08/03(月) 08:02:07.23ID:pGHv6+Gw
有名な問題だからね
ただ具体的な数字で計算はだるいって話でしょ
0889132人目の素数さん
垢版 |
2020/08/04(火) 09:27:31.15ID:+Srm/snw
>>888
まずは具体的な数字が与えられていてその期待値を計算せよ、と書いてあったでしょ
そして追記の計算式の方もa_iとかならともかくa,b,c…となっていたから書き下すのは大変
0892132人目の素数さん
垢版 |
2020/08/06(木) 06:16:24.50ID:meNNWVIo
>>889
a[i]で書くほうが手間だぞ。

1/a[1]+1/a[2]+1/a[3]+1/a[4]+1/a[5]+1/a[6]+1/a[7]+1/a[8]+1/a[9]+1/a[10] - (1/(a[1]+a[2])+1/(a[1]+a[3])+1/(a[1]+a[4])+1/(a[1]+a[5])+1/(a[1]+a[6])+1/(a[1]+a[7])+1/(a[1]+a[8])+1/(a[1]+a[9])+1/(a[1]+a[10])+1/(a[2]+a[3])+1/(a[2]+a[4])+1/(a[2]+a[5])+1/(a[2]+a[6])+1/(a[2]+a[7])+1/(a[2]+a[8])+1/(a[2]+a[9])+1/(a[2]+a[10])+1/(a[3]+a[4])+1/(a[3]+a[5])+1/(a[3]+a[6])+1/(a[3]+a[7])+1/
....
以下省略
0893132人目の素数さん
垢版 |
2020/08/06(木) 06:19:56.72ID:meNNWVIo
>>892
a_iなら少し手間が省けるかな。
計算式の最後の方を書き出すと
+1/(a_1+a_2+a_3+a_4+a_5+a_6+a_8+a_9+a_10)+1/(a_1+a_2+a_3+a_4+a_5+a_7+a_8+a_9+a_10)+1/(a_1+a_2+a_3+a_4+a_6+a_7+a_8+a_9+a_10)+1/(a_1+a_2+a_3+a_5+a_6+a_7+a_8+a_9+a_10)+1/(a_1+a_2+a_4+a_5+a_6+a_7+a_8+a_9+a_10)+1/(a_1+a_3+a_4+a_5+a_6+a_7+a_8+a_9+a_10)+1/(a_2+a_3+a_4+a_5+a_6+a_7+a_8+a_9+a_10)) - 1/(a_1+a_2+a_3+a_4+a_5+a_6+a_7+a_8+a_9+a_10)
0894132人目の素数さん
垢版 |
2020/08/06(木) 06:24:25.19ID:meNNWVIo
>>889
a,b,c,,,,,jだと計算式の文字数は14345
a_1,a_2,....a_10だと文字数は25097
になったが、数え落としがあるかわからんから、手書きして数えてくれw
0901132人目の素数さん
垢版 |
2020/08/10(月) 10:18:04.96ID:bhsRQgCg
>>898
こんな都合良く敷き詰められる者なの?
0902132人目の素数さん
垢版 |
2020/08/10(月) 10:25:26.62ID:rMjX33R2
学術の巨大掲示板群 - アルファ・ラボ ttp://x0000.net
数学 物理学 化学 生物学 天文学 地理地学
IT 電子 工学 言語学 国語 方言 など
0903132人目の素数さん
垢版 |
2020/08/10(月) 10:59:46.31ID:Y0L2GZC5
>>898
非負整数mと正整数nとでmm+mn+nnで表される整数が面積比として現れる

1 3 4 7 9 12 13 16 19…となるが、各々の後ろに2をつけて
12 32 42 72 92 122 132 162 192…とすると、対応するゴールドバーグ多面体が現れるところが面白い
0904132人目の素数さん
垢版 |
2020/08/10(月) 11:16:35.99ID:LgtUhmnG
>>898
おまえの連休、それでええんか?
0905132人目の素数さん
垢版 |
2020/08/10(月) 12:28:50.84ID:OMgDriQH
自然数 n に対し、以下を示せ。
(1) lim[n→∞] ∫[0,1] ((x^n)/(x+1)) dx = 0
(2) lim[n→∞] Σ[k=1,n] ((-1)^(k+1))/k = log(2)
0906132人目の素数さん
垢版 |
2020/08/10(月) 19:50:51.02ID:OMgDriQH
>>905
【追加】
(1)と(2)は同値であることを示せ。
0907132人目の素数さん
垢版 |
2020/08/11(火) 01:19:08.10ID:dwVOjOlW
立方体にくり抜いた穴にその立方体を通すことができるか?という有名な問題。
こういうCGはわかりやすいけど、グラフィックスで騙されていないかなとつい思ってしまうのだが、
https://www.youtube.com/watch?v=ua4LadxA6K8

実物作って実験した動画があった。
Prince Rupert’s cube live demonstration
https://www.youtube.com/watch?v=nWPdpqUEfE0

まだ、10人しか見ていないw
0908132人目の素数さん
垢版 |
2020/08/11(火) 01:26:59.51ID:KLYyGItm
>>907
単に最大サイズっぽい6角形に正方形が含まれることを示したら良いのでは
0909132人目の素数さん
垢版 |
2020/08/11(火) 01:38:53.74ID:KLYyGItm
1辺(√6)/3の正六角形だから中に1辺1の正方形はほんの少し余裕持って収まるよ
0911132人目の素数さん
垢版 |
2020/08/11(火) 11:10:59.31ID:KLYyGItm
>>910
対角線に沿って射影したのが最大じゃ無いの?
まあ立方体を通すには対角線に沿ってでいいけど
0912132人目の素数さん
垢版 |
2020/08/11(火) 11:36:18.24ID:dlrqXygC
対頂点を1組選び、各々がら出ている三本の辺計6本を除いた6本の辺の中点を結んだ正6角形
立方体の一辺が1なら正六角形の一辺は1/√2
0913132人目の素数さん
垢版 |
2020/08/11(火) 12:14:13.14ID:KLYyGItm
>>912
>6本の辺の中点を結んだ正6角形
何でそれ考えるの?
平面への射影の中に1辺1の正方形が収まれば良いんでしょうに?
0916132人目の素数さん
垢版 |
2020/08/11(火) 13:09:26.21ID:KLYyGItm
>>914
面白い!
しかし正方形だと単純なのが立方体だと相当複雑
この先次元上げて同じ問題考えたときはどうなるンかいヤ
0917132人目の素数さん
垢版 |
2020/08/11(火) 15:41:31.17ID:cs2e13nz
モンティホール問題はモンティが意図的にドアを開けるから
プレイヤーにとって最初に選んだ当たりのドアの確率は
1/3のまま不変

トランプ問題はシャッフルして無作為に選択するから
10/49に下がる
0918132人目の素数さん
垢版 |
2020/08/11(火) 16:44:23.90ID:vLWirhb5
a(n)=Σ[i=1,n]gcd(i,n)とする
nの素因数分解をn=Π(p_i)^(e_i)としたとき
a(n)=n×Π((e_i)(1-1/(p_i)+1)となることを示せ

例えばn=720=2×2×2×2×3×3×5のとき
a(n)=720×(4×(1-1/2)+1)(2×(1-1/3)+1)(1×(1-1/5)+1)=9072
となる
0919132人目の素数さん
垢版 |
2020/08/11(火) 16:45:55.77ID:vLWirhb5
>>918
カッコミス

誤 a(n)=n×Π((e_i)(1-1/(p_i)+1)となることを示せ
正 a(n)=n×Π((e_i)(1-1/(p_i))+1)となることを示せ
0920132人目の素数さん
垢版 |
2020/08/11(火) 20:23:50.39ID:vj5zwGo/
>>918
a(n) はオイラーの φ 関数を使うと
a(n) = n Σ[d|n] φ(d)/d
と書けることから、 a(n) は(互いに素な数の積に関して)乗法的であることがわかる。
( a(n) はgcd-sum functionまたはPillai's arithmetical functionと呼ばれているらしい)
また、上の式と φ 関数の性質から、素数 p のべき乗 p^k について
a(p^k) = (p^k)(1 + k(1 - 1/p))
が成り立つので、
a(n) = Π a((p_i)^(e_i)) = Π ((p_i)^(e_i))(1 + (e_i)(1 - 1/p_i))
= n Π (1 + (e_i)(1 - 1/p_i))
0922132人目の素数さん
垢版 |
2020/08/11(火) 23:14:46.70ID:dwVOjOlW
ジュースディスペンサー
例 https://event21.co.jp/pic/mat_154.png
に100%天然果汁が10L入っていて常に撹拌されている。
すなわち、濃度はいつも均一とする設定。

コックにトラブルがあって1分間に10mLずつ漏れている。
それを補うために1分間に10mLの天然水を補っているとする。
50%天然果汁のジュースになるのは何分後か?
0923132人目の素数さん
垢版 |
2020/08/12(水) 01:00:27.00ID:oejKmXQz
>>918
部分和の謎公式みつけたかも。q(x)=1-2(x-[x])とおく
Σ[i=1〜k]gcd(i,n)=Σ[i=1〜k, j=1〜n]q(ij/n)
0924132人目の素数さん
垢版 |
2020/08/12(水) 01:05:17.54ID:vgqXhROq
充填した時刻をt=0、濃度をC(0) とする。
時刻tでの濃度をC(t)とすると、題意より
 dC(t)/dt = - 0.001 C(t),
これを解いて
 C(t) = C(0)exp(-0.001t),

半減期をT(分)とする。
 C(T) = C(0)/2,
 log(2) = 0.001T
 T = 1000 log(2) = 693.15(分)
0925132人目の素数さん
垢版 |
2020/08/12(水) 01:23:25.46ID:JIB7mcFI
>>923
{x} = x - [x] ( x の小数部分)を使えばさらに不思議な式に
Σ[i=1,k] gcd(i,n) = Σ[i=1,k] Σ[j=1,n] (1 - 2{ij/n})
0926132人目の素数さん
垢版 |
2020/08/12(水) 01:36:04.66ID:oejKmXQz
>>925
よく考えたらもっと直接
gcd(i,n)=Σ[j=1,n] (1 - 2{ij/n})
と計算できることがわかりました!
0930132人目の素数さん
垢版 |
2020/08/12(水) 06:28:25.58ID:KrQ981jo
0.1分後の濃度は1-1/10000になるから
50%になるのは
(log0.5)/log(0.9999)*0.1=693.112522623342になる
0.01分後だと
(log0.5)/log(0.99999)*0.01
=693.143714821421
になって
>924の値に近づいた。
微分方程式なしで子供に説明できそう。
0931132人目の素数さん
垢版 |
2020/08/12(水) 06:30:52.24ID:KrQ981jo
>>922
>それを補うために1分間に10mLの天然水を補っているとする。
これを5mLに変えたらどうやって計算すればいいんだろう?
0932イナ ◆/7jUdUKiSM
垢版 |
2020/08/12(水) 15:08:38.79ID:VaAaef6o
>>862
>>922
1分後 果汁9990ml+天然水10ml
2分後 果汁9980.01ml+天然水19.99ml
3分後 果汁9970.02999ml+天然水29.97001ml
4分後 果汁9960.05996001ml+天然水39.94003999ml
5分後 果汁9950.09990004999ml+天然水49.90009995001ml
n分後 果汁A_n=0.999A_n-1=10000(0.999)^n=5000
天然水10000-A_n=10000-0.999A_n-1
(0.999)^n=1/2
0933イナ ◆/7jUdUKiSM
垢版 |
2020/08/12(水) 15:25:28.84ID:VaAaef6o
>>932
>>922
(0.999)^692=0.50040063528......
(0.999)^693=0.49990023464......
∴693分で希釈できる。
0934132人目の素数さん
垢版 |
2020/08/12(水) 16:04:20.55ID:/JkmxmVW
>>931
1分毎に間欠的に5mLの水を補充するモデルで考えた。

V(t):t分後のジュースの総量
C(t):をt分後の濃度

V(0)=10000
V(1)=10000-10+5
V(t)=10000-5*t

C(0)=1
C(1)=9990/9995=C(0)*(V(0)-10)/V(1)
C(2)=C(1)*(V(1)-10)/V(2)

C(t+1)=C(t)*(V(t)-10)/V(t+1) = C(t)*(10000-5*t-10)/(10000-5*(t+1))

> C[999]
[1] 0.50025012506253153
> C[1000]
[1] 0.49974987493746903

になったので1000分後でいいかな?

連続注入のときに>924のような立式はどうするのかわからないので得意な方にお願いします。
0935132人目の素数さん
垢版 |
2020/08/12(水) 16:13:37.59ID:/JkmxmVW
>>924
放射性物質の崩壊仮定とそっくりの式になるんですね。
レスありがとうございました。
0937132人目の素数さん
垢版 |
2020/08/14(金) 07:57:53.41ID:GJ+vKVSe
セシウム137の半減期を30年、セシウム134の半減期を2年としたときに総セシウムの数の半減期と総放射能の半減期は何年か?
って厳密解が計算できる?

どうやって計算するの分からなかったからシミュレーション解しか出せなかった。
0939132人目の素数さん
垢版 |
2020/08/14(金) 08:43:46.83ID:pzSK4b+7
>>937
式を簡単にするために、最初の量を2とする
1/(2^(t/30))+1/(2^(t/2))=1
wolfram先生によると t=5.93045 らしい
0940132人目の素数さん
垢版 |
2020/08/14(金) 11:29:31.46ID:cIdouH6q
数値解ならだせるんだが、

C137 <- function(t) 0.5*(1/2)^(t/30)
C134 <- function(t) 0.5*(1/2)^(t/2)
C_total <- function(t) C137(t)+C134(t)
curve(C_total(x),0,30)
(t0.5=uniroot(function(t,u0=1/2) C_total(t)-u0, c(0,30))$root)

> (t0.5=uniroot(function(t,u0=1/2) C_total(t)-u0, c(0,30),tol = 1e-12)$root)
[1] 5.930454

https://i.imgur.com/DIyJXk0.png
0941132人目の素数さん
垢版 |
2020/08/14(金) 12:11:41.17ID:cIdouH6q
総放射能の半減期は

C137 <- function(t) 0.5*(1/2)^(t/30)
C134 <- function(t) 0.5*(1/2)^(t/2)

CR_total <- function(t) (log(2)/30)*C137(t) + (log(2)/2)*C134(t) # log(2)/半減期=崩壊定数
curve(CR_total(x),0,30)
CR_total(0)
(tr0.5=uniroot(function(t,u0=CR_total(0)/2) CR_total(t)-u0, c(0,30))$root)

> (tr0.5=uniroot(function(t,u0=CR_total(0)/2) CR_total(t)-u0, c(0,30))$root)
[1] 2.178902
0942132人目の素数さん
垢版 |
2020/08/17(月) 02:25:40.50ID:U7f6nYy/
時刻tで未崩壊の総セシウム数は
 f(t) = 30・(1/2)^(t/30) + 2・(1/2)^(t/2)
に比例する。
これが現在の半分になる時が t=T とすると
 f(T) = f(0)/2,
 T = 27.2071525 (年)
 セシウム134 は ほとんど残ってない。
 残っているのは ほぼ全部セシウム137.
0943132人目の素数さん
垢版 |
2020/08/17(月) 17:57:48.48ID:h5k/PqLF
原子数=放射能/崩壊定数=放射能/(log2/半減期)=放射能*半減期*log2だから放射能が同じなら原子数∝半減期だから>942の通りですね。
0944132人目の素数さん
垢版 |
2020/08/17(月) 19:09:24.12ID:h5k/PqLF
原子数=放射能/崩壊定数=放射能/(log2/半減期)=放射能*半減期*log2
原子数で1:1なら、
放射能は原子数*崩壊定数=原子数*log2/半減期 ∝ 原子数/半減期
なので、原子数で1:1のときに放射能が1/2になるのは>941で
>942の計算は放射能比でCs137とCs134の比が1:1の計算でどちらも正しいと思う。
0945132人目の素数さん
垢版 |
2020/08/17(月) 19:56:14.56ID:W/1NrXP0
>>942
これは総セシウム数が半分になる期間で総放射能が半分になる期間ではないと思う。
0946132人目の素数さん
垢版 |
2020/08/17(月) 20:22:13.00ID:f+AbBcsh
時刻0において、何が1:1なのか?
初期状態において、セシウム137由来の放射線量と、セシウム134由来の放射線量が1:1と解釈するのか、
セシウム137の物質量と、セシウム134の物質量が1:1と解釈するのかでは、自ずと別の問題になる。
>>938の補足は、後者だという意思表明だと思うが、いずれであろうとも、
初期状態の設定の違いとして現れるだけなので、回答を作成する場合には、本質的な差は無い。

それよりも、混合線源に対して、「半減期」なるものは定義できないのでは?
問題作成者に問題があるとおもうが。
0947132人目の素数さん
垢版 |
2020/08/17(月) 20:36:31.16ID:W/1NrXP0
フクイチから放出されたセシウム137、セシウム134のベクレル比が1:1とすると本日の残存セシウム総数とそれからの放射能は

> cesium_now()
$mol_ratio
[1] 0.7561325

$radioactivity_ratio
[1] 0.02628639

という値になった。

セシウム総数が半分になるのが27年ということなので9.4年めの今は75%残存は感覚としてもあってそう。
0948132人目の素数さん
垢版 |
2020/08/17(月) 20:37:50.71ID:W/1NrXP0
>>946
>混合線源に対して、「半減期」なるものは定義できないのでは
これはご指摘の通りです。初期状態の半分になる時間と考えてください。
0949132人目の素数さん
垢版 |
2020/08/21(金) 07:00:50.93ID:eKSCCB4p
>>900
「n」の正六角形
 原点−中心の距離がn,
 (外接円の半径R) = (一辺の長さ) = √n,
 面積が (3√3)/2・n,
 aが中心ならば -3a, -3ωa, -3ω~a も中心
0950132人目の素数さん
垢版 |
2020/08/21(金) 18:48:33.23ID:eKSCCB4p
>>900
自然数mに対して「m^2」の正六角形がある。
 中心は -m^2, -m^2・ω, -m^2・ω~
 (外接円の半径 R) = (一辺の長さ) = m,
 面積が (3√3)/2・m^2
0951132人目の素数さん
垢版 |
2020/08/23(日) 08:46:24.44ID:zNwHrz2Z
ボルトの頭ってなんで六角形が多いの?

五角形の方が良くない?
角が少ない分、角が潰れることが少ないし、
角度は90度未満で回せる点は六角形に劣らない。
0954132人目の素数さん
垢版 |
2020/08/23(日) 09:57:06.06ID:DAxswYAA
>>951
二角形がベストと言うことか
0956132人目の素数さん
垢版 |
2020/08/23(日) 11:22:29.04ID:Wi06ZCqF
平行な辺がないとレンチをきっちり当てるのに苦労しそうだな
レンチをはめることを考えると四角形より六角形の方がやりやすく、角をなめにくいことを考えると八角形以上より六角形の方が有利
ってことで六角形なんじゃないか?
0957132人目の素数さん
垢版 |
2020/08/23(日) 15:01:09.38ID:+zeUDUO0
>>955 これ自己解決しました。

閉領域 [0...n]× [0...i] に於ける格子点を考える。
P, Q, M を其々 直線: y= (i/n) * x の 下方にある/ 上方にある / 乗っている 格子点数とする。
(n/2, i/2) に対しての点対称性より P = Q, それと M= gcd(i,n)+1
領域の全格子点数は (n+1)(i+1) = P+M+Q = 2P + gcd(i,n)+1
∴ Σ[j=1,n] 2[ij/n] = 2( P + M -1-n ) = (n+1)(i+1) -gcd(i,n)-1 + 2(gcd(i,n) -n)
 = gcd(i,n) -n + i(i+n)
0958イナ ◆/7jUdUKiSM
垢版 |
2020/08/23(日) 23:30:47.99ID:Q5A4PXq6
>>933
>>951
ペンチやモンキーで挟むとき辺が平行じゃないと力が均等にかからないから回しにくい。よって正方形か正六角形か正八角形が考えられるが、正方形の場合ペンチを差しこむ体勢が2つしかない。となりのネジに障って回せなかったりコードに手が支えたりするなか確率の低いことをするとなんでネジを正方形にすんねんてなる。逆に正八角形だとネジとペンチの接触面が少なく力をじゅうぶんに伝えられず効率がわるい。
あいだをとって正六角形にするのがよいと考えられている。もちろん別の考えで別のかたちのネジを作ってもよいが、世間に認められるかどうかは別だ。
0959132人目の素数さん
垢版 |
2020/08/25(火) 08:56:31.72ID:7va++JjE
>>951
数学脳は真実を見ない典型だな
72°>60°
これに尽きる
0960132人目の素数さん
垢版 |
2020/08/25(火) 18:38:19.82ID:LqiSh/C2
なるほど
おもしろいですね
0962132人目の素数さん
垢版 |
2020/08/27(木) 13:45:54.80ID:hFWBTQEb
∫^1_0ln(x^2-x+1)/(x*(x-1))dxはどのようにして求められますか?
0964132人目の素数さん
垢版 |
2020/08/27(木) 15:07:50.44ID:17pR8ej1
組合せ論の天才ロバース(数学オリンピック3回金メダル)の本だけあって面白い問題がありました。

□□□□□□□□
□□□□□□□□
□□□□□□□□
□□□□□□□□
□□□□□□□□
□■□□□□□□
□□□□□□□□
□□□□□□□□

2^n × 2^nのチェス盤から1つのマス目だけ取り除いた欠損チェス盤は以下のL字牌で敷き詰められることを証明せよ。


□□
0966132人目の素数さん
垢版 |
2020/08/27(木) 15:48:00.29ID:MGNmMRXt
>>962
∫[0,1] log(x^2-x+1)/(x(x-1)) dx
と解釈すると、すぐに思いついたのは、 log(1+x) のテイラー展開より |x| < 1 のとき
log(1+x) = Σ[n=1,∞] (-1)^(n-1)x^n/n
で、 log(x^2-x+1)/(x*(x-1)) = log(1+x(x-1))/(x(x-1)) において
0 ≦ x ≦ 1 のとき |x(x-1)| < 1 だから、
log(1+x(x-1))/(x(x-1)) = Σ[n=1,∞] (-1)^(n-1)(x(x-1))^(n-1)/n
より、項別積分によって 0 ≦ x ≦ 1 における定積分が計算できる
ここで
∫[0,1] (x(x-1))^(n-1) dx = (-1)^(n-1) Β(n, n)
となるので、
∫[0,1] log(1+x(x-1))/(x(x-1)) dx = Σ[n=1,∞] Β(n, n)/n
= Σ[n=1,∞] (((n-1)!)^2/(n*(2n-1)!))
Wolfram大先生によると π^2/9 らしいが、どうやって証明するんだろう
https://www.wolframalpha.com/input/?i=%CE%A3%5Bn%3D1%2C%E2%88%9E%5D+%28%28%28n-1%29%21%29%5E2%2F%28n*%282n-1%29%21%29%29
0967132人目の素数さん
垢版 |
2020/08/27(木) 17:41:24.67ID:bnTgCcB8
とりあえず
Σ[n=0,∞]n!n!/(2n+2)!=3F2(1,1,1;2,3/2;1/4)
になってコレにClausen's formulaなるものを使うと
=(2F1(1/2,1/2;3/2;1/4))^2
になりコレに2F1(1/2,1/2;3/2;s^2)=(1/s)arcsin(s)を適用すると
=(2(π/6))^2=π^2/9
にはなった
0968132人目の素数さん
垢版 |
2020/08/27(木) 19:52:04.98ID:yuNusFyR
Σ[n=m,∞] n!n!/(2n+2)! = 3F2(1,m+1,m+1;m+2,m+3/2;1/4)
で m=0 の場合。

>>965
□□
□△
○△△□
○○□□
のように分解していく・・・・
0970132人目の素数さん
垢版 |
2020/08/27(木) 20:02:57.96ID:17pR8ej1
>>968

模範解答は実は非常に分かりやすいんです。
0971132人目の素数さん
垢版 |
2020/08/27(木) 20:08:22.16ID:17pR8ej1
帰納法でやるんですが、例えば、

2^1のときには明らかに敷き詰め可能です。

2^kのときOKと仮定します:

□□□□
□■□□
□□□□
□□□□



□□□□□□□□
□□□□□□□□
□□□□□□□□
□□□■■□□□
□□□□■□□□
□■□□□□□□
□□□□□□□□
□□□□□□□□

を4分割します:

□□□□
□□□□
□□□□
□□□■

□□□□
□□□□
□□□□
■□□□

□□□□
□■□□
□□□□
□□□□

■□□□
□□□□
□□□□
□□□□

帰納法の仮定によりこれらの4つの欠損チェス盤は敷き詰め可能です。

もう一度、2^(k+1)の下のチェス盤を見てください:

□□□□□□□□
□□□□□□□□
□□□□□□□□
□□□■■□□□
□□□□■□□□
□■□□□□□□
□□□□□□□□
□□□□□□□□
0972132人目の素数さん
垢版 |
2020/08/27(木) 20:09:26.66ID:17pR8ej1
中央の

■■
□■

黒い部分(欠損している部分)はL字牌で敷き詰められます。

以上です。
0973132人目の素数さん
垢版 |
2020/08/27(木) 22:05:49.33ID:9j7a53Tx
>>970
さんの持ってる模範解答はともかくとして
Clausen's identity
(2F1(a,b;a+b+1/2;z))^2
=3F2(2a,2b,a+b;2a+2b,a+b+1/2;z)
ってどうやって示すんだろう?
0974132人目の素数さん
垢版 |
2020/08/28(金) 00:17:31.39ID:y9rDl245
>>970
良いスナその模範解答キモチヨイ
0975132人目の素数さん
垢版 |
2020/08/28(金) 00:25:18.50ID:PuYYvvEm
次の漸化式で定められた数列の全ての項は平方数であることを示せ
a_1=a_2=a_3=1
a_(n+3)=-a_(n+2)+2*a_(n+1)+8*a_n
0976132人目の素数さん
垢版 |
2020/08/28(金) 01:08:42.13ID:HSbg+0jD
10
11
20
31
52
121
200
314
512

これ次の数字分かる人いる?
数学というより数学モチーフにしたクイズ
規則性はちゃんとあるけどね
0978132人目の素数さん
垢版 |
2020/08/28(金) 01:24:43.52ID:HSbg+0jD
>>977
正解
これ注意書きなく単に「次の数字は?」って出し方したら悪問かな
どう思う?
自分は別にいい気がするけど
0979132人目の素数さん
垢版 |
2020/08/28(金) 02:06:57.43ID:sIUBKqp2
>>975
力技だけど
4項間漸化式を特性方程式x^3+x^2-2x-8=0の根
α=2, β=(-3+√(-7))/2, γ=(-3-√(-7))/2
を使って
a_n=Aα^n+Bβ^n+Cγ^n
と形を決めて、初項から
A=2/7, B=-1/7, C=-1/7
と係数を決定して
a_n=(2^(n+1)-((-3+√(-7))/2)^n-((-3-√(-7))/2)^n)/7
となるが、これは次の数列
b_n=(((-1+√(-7))/2)^n-((-1-√(-7))/2)^n)/√(-7
を使って
a_n=(b_n)^2
と書けている
b_nは3項間漸化式
b_0=0, b_1=1 b_(n+2)=-b_(n+1)-2b_n
を満たす数列であり、すべて整数である
よってa_nは必ず平方数になる
0980132人目の素数さん
垢版 |
2020/08/28(金) 02:12:30.96ID:Q3hz88VV
>>975
p=(-1+√7i)/2, q= (-1-√7i)/2, bn=(p^n-q^n)/(p-q)
とおく
b1=1, b2=-1, b3=-1
である
b[n+2]=-b[n+1]-2bn
によりbnは全て整数
cn=bn^2とおくとcnは全て平方数である
c1=c2=c3=1
cn=(p^2n+q^2n+2^n)/(p-q)^2
p^2+q^2+2=-1
2p^2+2q^2+p^2q^2=-2
2p^2q^2=8
によりcnは漸化式
c[n+3]=-c[n+2]+2c[n+1]+8c[n]
を満たす
∴ an=cn
0981132人目の素数さん
垢版 |
2020/08/28(金) 02:27:13.37ID:sIUBKqp2
>>969
このarcsinの2乗のテイラー展開は神秘的だな
逆三角関数のwiki英語版には書いてるけど日本語版には書いてない
0982132人目の素数さん
垢版 |
2020/08/28(金) 04:14:46.67ID:Lqo6RwyU
>>969
この記事ではChu-Vandermondeの恒等式とやらを使って証明しているけど、
f(x) := (arcsin(x))^2 のテイラー展開は普通に微分係数を計算しても証明できるみたい

f(0) = 0
f'(x) = 2arcsin(x)/(1-x^2)^(1/2) より f'(0) = 0
f''(x) = 2(1/(1-x^2) + xarcsin(x)/(1-x^2)^(3/2)) より f''(0) = 2
ここで微分方程式
(1-x^2)f''(x) = xf'(x) + 2
が成り立つので、この両辺を n 階微分すると、
(1-x^2)f^(n+2)(x) = (2n+1)xf^(n+1)(x) + (n^2)*f^(n)(x) より
f^(n+2)(0) = (n^2)*f^(n)(0)
が成り立つ。
よって n が奇数なら f^(n)(0) = 0, n が偶数なら、 n = 2k のとき
f^(2k)(0) = (2^(2k-1))*((k-1)!)^2
となるので、 f(x) のテイラー展開が
f(x) = Σ[n=1,∞] ((2^(2n-1))*((n-1)!)^2/(2n)!) x^(2n)
と求まる。
ゆえに、
2(arcsin(x/2))^2 = Σ[n=1,∞] (((n-1)!)^2/(2n)!) x^(2n)
が成り立つ。

特に x = 1 とすれば、
Σ[n=1,∞] (((n-1)!)^2/(2n)!) = π^2/18
となるから、したがって
Σ[n=1,∞] (((n-1)!)^2/(n*(2n-1)!)) = π^2/9
が得られる。
0983132人目の素数さん
垢版 |
2020/08/28(金) 09:44:16.83ID:RJx6/e6A
>>979 より
 b_0 = 0, b_1 = 1, b_{n+2} = -b_{n+1} - 2b_n,

ここで
 b_n = (-√2)^n B_n, cosθ = 1/√8,
とおくと
 B_0 = 0, B_1 = 1/(-√2),
 B_{n+2} = 2cosθ・B_{n+1} - B_n,
これと sinθ の和積公式
 sin((n+2)θ) = 2cosθ・sin((n+1)θ) - sin(nθ),
を比べて
 B_n = B_1 sin(nθ)/sinθ = 1/(-√2) sin(nθ)/(sinθ)
 b_n = (-√2)^{n-1} sin(nθ)/sinθ,
ここで
n-1が奇数 ⇒ sin(nθ)/sinθ はcosθ=1/√8 の奇関数、
n-1が偶数 ⇒ sin(nθ)/sinθ は cosθ=1/√8 の偶関数。
∴ b_n は整数。

なお、
 sin(nθ)/(sinθ) = U_{n-1}(cosθ),
を第二種チェビシェフ多項式と云うらしい。
0984132人目の素数さん
垢版 |
2020/08/28(金) 09:52:46.84ID:sIUBKqp2
ついこの前も3項間漸化式をわざわざチェビシェフ多項式で解いてたレス見たわ

ところで偶然にも>>969のテイラー展開はチェビシェフ多項式を使った「チェビシェフ展開」と見れる説が浮上してる
0985132人目の素数さん
垢版 |
2020/08/28(金) 11:47:15.50ID:RJx6/e6A
>>983
β,γは特性方程式の因数 x^2 + 3x + 4 = 0 の根
これより漸化式
 a_{n+2} = - 3a_{n+1} - 4a_n + 2^{n+2},
を得る。ここで
 a_n = (2^n)(A_n + 2/7), cos(2θ) = -3/4,
とおくと
 A_1 = 3/14, A_2 = -1/28, A_3 = -9/56,
 A_{n+2} = 2cos(2θ)・A_{n+1} - A_n,
これと cos の和積公式
 cos(2(n+2)θ) = 2cos(2θ)・cos(2(n+1)θ) - cos(2nθ),
を比べて
 A_n = - (2/7)cos(2nθ),
 a_n = 2^{n-1}・(4/7){1-cos(2nθ)}
   = 2^{n-1}・(8/7){sin(nθ)}^2,
となるが、平方数かどうか分からぬ・・・・・
0987132人目の素数さん
垢版 |
2020/08/28(金) 12:40:36.01ID:Lqo6RwyU
線型回帰数列の一般項を求めるのに線型代数を使おうが母関数を使おうが自由ではあるが、
一般項を求めるだけでは解決しない問題にわざわざ複雑な方法を使う意味がわからない
別の方法で計算したところで全く役に立ってないし
0988132人目の素数さん
垢版 |
2020/08/28(金) 13:38:09.10ID:RJx6/e6A
>>968
盤を 1x1のマスの集まりと見て、■マスを b_0
  2x2 ブロックの集まりと見たとき、■を含むブロックを b_1
  4x4 ブロックの集まりと見たとき、■を含むブロックを b_2
  ・・・・
  2^n x 2^n ブロック全体、b_n
とする。
 b_0 ⊂ b_1 ⊂ ・・・・ ⊂ b_n
これらの差分は、辺長が2ベキであるn個のL字形である。辺長は
  (1,2), (2,4),・・・・, (2^{n-1},2^n)
つまり、盤面全体が、■マスとn個のL字形とに分割される。

これらのL字形は、>>968 のやり方で半サイズのL字形に分解してゆけば、最後には
 □
 □□
に至る。
0989132人目の素数さん
垢版 |
2020/08/28(金) 16:23:16.84ID:2EjgpYll
n^2 + 1個の相異なる整数からなる数列には、長さn+1の増加部分列があるか、あるいは長さn+1の減少部分列があることを証明せよ。
0990132人目の素数さん
垢版 |
2020/08/28(金) 16:31:54.40ID:jXffUYs/
>>964 >>971
面白い問題ですね。
区画を 2*2 の4区画に分割していき、
切り分ける際に交点が1つあるのがミソですね。

これ、パーツを変更して
    □□
ただの2マスの棒にしたら
成立しないんだよな。(面積が偶数になってしまうからスペースが残せない)
0992132人目の素数さん
垢版 |
2020/08/28(金) 16:54:09.24ID:jXffUYs/
>>964

□□■…
□□□…
□□□…


サイズが a^n × b^n のチェス盤がある。
1つのマス目だけ取り除いた欠損チェス盤が
以下の部品で敷き詰められる
ようなチェス盤は存在するか?

□□□

↑ 3マスの横棒
存在するならば、そのような自然数 a,b を求めよ。
0993132人目の素数さん
垢版 |
2020/08/28(金) 22:59:15.08ID:y9rDl245
>>990
たりめーじゃん
0994132人目の素数さん
垢版 |
2020/08/29(土) 03:30:24.84ID:nEvr3uHf
>>964 の追加問題

 ■の位置が決まれば、L字形の敷き詰め方は >>988 に限るか?

 頂点(i,j)については

 ○−△
 | |
 △ ○−△
 |   |
 ○−△−○

 ○ i+j=偶数
 △ i+j=奇数

>>987
 実数だけで解ける問題にわざわざ複素数を使う意味が分からぬ

次スレ
http://rio2016.5ch.net/test/read.cgi/math/1598637093/
0995132人目の素数さん
垢版 |
2020/08/29(土) 03:50:24.27ID:nEvr3uHf
>>983
 U_0 = 1, U_1(x) = 2x, U_2(x) = (2x)^2 -1, U_3(x) = (2x)^3 - 2(2x), …

 U_{n-1}(x) は 2x の整係数(n-1)次式。
∴ (√2)^{n-1} U_{n-1}(1/√8) は整数
0996132人目の素数さん
垢版 |
2020/08/29(土) 03:51:43.96ID:gICDV3If
>>994
>実数だけで解ける問題にわざわざ複素数を使う意味が分からぬ

そういうことはせめて一般項を具体的に書き下してから言ってね
何? sin(nθ) って?
それに>>987>>975の解決に一切役に立っていないことを批判しているわけだが
わからないのか?
0997132人目の素数さん
垢版 |
2020/08/29(土) 06:16:48.81ID:xlGB+YDQ
実数だけの問題を解くのに複素数を使うほうが楽な例はある
波動方程式とかね

そういうとき複素数を使うことに違和感はないけどな
0998132人目の素数さん
垢版 |
2020/08/29(土) 10:28:20.59ID:nEvr3uHf
>>996
cosθ = 1/√8,
すなわち
θ = arccos(1/√8) = (1/2)arccos(-3/4) = 1.2094292…
ですね。
よく見て「批判」しましょう。
10011001
垢版 |
Over 1000Thread
このスレッドは1000を超えました。
新しいスレッドを立ててください。
life time: 144日 0時間 34分 11秒
10021002
垢版 |
Over 1000Thread
5ちゃんねるの運営はプレミアム会員の皆さまに支えられています。
運営にご協力お願いいたします。


───────────────────
《プレミアム会員の主な特典》
★ 5ちゃんねる専用ブラウザからの広告除去
★ 5ちゃんねるの過去ログを取得
★ 書き込み規制の緩和
───────────────────

会員登録には個人情報は一切必要ありません。
月300円から匿名でご購入いただけます。

▼ プレミアム会員登録はこちら ▼
https://premium.5ch.net/

▼ 浪人ログインはこちら ▼
https://login.5ch.net/login.php
レス数が1000を超えています。これ以上書き込みはできません。

ニューススポーツなんでも実況